You are on page 1of 43

CONSTI TUTI ONAL LAW 2 ZPG & ASSOCI ATES

Bill of Rights

I. THE DUE PROCESS, EQUAL PROTECTION AND CONTRACT CLAUSES AS
LIMITATIONS ON POLICE POWER, EMINENT DOMAIN AND TAXATION

Art. III, Sec. 1. No person shall be deprived of life, liberty, or property
without due process of law, nor shall any person be denied the equal
protection of the laws.

Art. III, Sec. 10. No law impairing the obligation of contracts shall be
passed.


LIMITATIONS OF SOVEREIGNTY

Inherent in sovereignty, and therefore not even required to be conferred
by the Constitution, are the police, eminent domain, and taxation powers. The
Bill of Rights, notably the due process, equal protection and non-impairment
clauses, is a means of limiting the exercise of these powers by imposing on the
State the obligation to protect individual rights. The Bill of Rights is addressed to
the State, notably the government, telling it what it cannot do to the individual.


A. Due process - Procedural and Substantive

Civil Procedural Due Process

In civil cases, the SC laid down its elements in the case of Banco Espanol
Filipino v. Palanca:

a. Court with jurisdiction over the subject matter.

b. Court with jurisdiction over the party-defendant

c. Judgement rendered according to law.

d. Defendant given the oppotunity to be heard (requirement on notice
and hearing)




Criminal Due Process

In criminal cases, the elements were laid down in Vera v. People:

a. Accused is informed why he is proceeded against, and what charge
he must answer.

b. Judgment of conviction is based on evidence that is not tainted by
falsity, and after the defendant was heard.

If the prosecution produces the conviction based on untrue
evidence, then it is guilty of depriving the accused of due process. Thus false
testimony can be questioned by the accused regardless of the time that
lapsed.

c. Judgment according to law

d. Tribunal with jurisdiction


Administrative Due Process

In administrative proceedings, the elements were laid down in the case
of Ang Tibay v. CIR as the "seven cardinal primary rights" in justiciable cases
before administrative tribunals:

a. There must be a hearing, where a party may present evidence in
support of his case.
b. The tribunal must consider the evidence presented by a party.
c. While the tribunal has no duty to decide the case correctly, its
decision must be supported by evidence.
d. The evidence supporting the decision must be substantial.
Substantial evidence is such relevant evidence as a reasonable mind might
accept as adequate to support a conclusion.
e. The evidence must have been presented at the hearing or at least
contained in the record and known to the parties affected.
f. The tribunal must rely on its own independent consideration of
evidence, and not rely on the recommendation of a subordinate.
g. The decision must state the facts and the law in such a way that the
parties can know the issues involved and the reasons for the decision.


Substantive Due Process

Ynot vs. IAC, 148 SCRA 659 (1987)

F: Petitioners' 6 carabaos were confiscated by the police for having been
transported from Masbate to Iloilo in violation of EO 626-A. He brought an
action for replevin, challenging the consitutionality of the EO. The trial court
sustained the confiscation of the animals and declined to rule on the validity of
the law on the ground that it lacked authority to do so. Its decision was affirmed
by the IAC. Hence this petition for review.

HELD: (1) Under the provision granting the SC jurisdiction to "review, revise,
reverse, modify or affirm on appeal or certiorari, as the law or rules of court may
provide final judgments of lower courts" in all cases involving the
constitutionality of certain measures, lower courts can pass upon the validity of
a statute in the first instance.
(2) There is no doubt that by banning the slaughter of these animals
(except where there at least 7 yrs. old if male and 11 yrs old if female upon the
issuance of the necessary permit) the EO will be conserving those still fit for farm
work or breeding and preventing their improvident depletion. We do not see,
however, how the prohibition of the interprovincial transport of carabaos can
prevent their indiscriminate slaughter, considering that they can be killed any
where, w/ no less difficulty in on province than in another. Obviously, retaining
the carabao in one province will not prevent their slaughter there, any more
than moving them to another province will make it easier to kill them there. As
for the carabeef, the prohibition is made to apply to it as otherwise, so says the
EO, it could be easily circumsbcribed by simply killing the animal. Perhaps so.
However, if the movement of the live animals for the purpose of preventing
their slaughter cannot be prohibited, it should follow that there is no reason
either to prohibit their transfer as, not to be flippant, dead meat.
(3) In the instant case, the carabaos were arbitrarily confiscated by the
police station commander, were returned to the petitioner only after he had
filed a complaint for recovery and given a supersedeas bond w/c was ordered
confiscated upon his failure to produce the carabaos when ordered by the trial
court. The EO defined the prohibition, convicted the petitioner and immediately
imposed punishment, w/c was carried out forthright. The measures struck him
at once and pounced upon the petitioner w/o giving him a chance to be heard,
thus denying him elementary fair play.
(4) It is there authorized that the seized prop. shall "be distributed to
charitable institutions and other similar institutions as the Chairman of the
National Meat Inspection Commission may see fit, in the case of carabeef, and to
deserving farmers through dispersal as the Director of Animal Industry may see
fit in the case of carabaos." The phrase may see fit is an extremely generous and
dangerous condition, if condition it is. It is laden w/ perilous opportunities for
partiality and abuse, and even corruption. One searches in vain for the usual
standard and the reasonable guidelines, or better still, the limitations that the
said officers must observe when they make their distribution. VV.


B. Due Process and Police Power


Bautista v. Juinio, 127 SCRA 329 (1984)

Ban on Use of Heavy Cars on Week-ends and Holiday s Valid.

F: LOI 689 banned the use of vehicles w/ A and EH plates on week-ends and
holidays in view of the energy crisis. It excepted, however, those classified as S
(Service), T (Truck), DPL (Diplomatic), CC (Consular Corps), and TC (Tourist Cars).
The resps., Min. of Public Works, Transportation, issued memo. providing
penalties for viol. of the LOI, namely, fine, confiscation of vehicles, and
cancellation of registration. The petitioners brought suit questioning the validity
of the LOI on the ground that it was discriminatory and a denial of due process.
The resps. denied the petitioner's allegations and argued that the suit amounted
to a request for advisory opinion.

HELD: (1) Petitioners are owners of an 8 cylinder 1969 Buick and of a 6 cylinder
Willy's Kaiser Jeep. The enforcement of the LOI to them would deprive them of
prop. They, therefore, have standing to challenge the validity of the LOI.
(2) But the LOI cannot be declared void on its face. It has behind it the
presumption of validity. The necessity for evidence to rebut such presumption is
unavoidable. As underlying the questions of fact may condition the
constitutionality of legislation the presumption of validity must prevail in the
absence of some factual foundation of record overthrowing the statute. The
LOI is an energy conservation measure; it is an apporpriate response to a
problem.
(3) Nor does the LOI deny equal protection to the petitioners. W/in the
class to w/c the petitioner belongs the LOI operate equally and uniformly. That
the LOI does not include others does not render it invalid. The govt is not
required to adhere to a policy of "all or none."
(4) To the extent that the Land Transpo. Code does not authorize the
impounding of vehicles as a penalty, to that extent the memo. of the resps.
would be ultra vires. VV.

CONSTI TUTI ONAL LAW 2 ZPG & ASSOCI ATES



Velasco v. Villegas, 120 SCRA (1983)

Ordinance Prohibiting Barbershops from Rendering Massage Services Valid.

F: The ordinance was enacted for a two-fold purpose: (1) To enable the
City of Mla. to collect a fee for operating massage clinics separately from those
operating barber ships and (2) To prevent immorality w/c might probably arise
from the construction of separate rooms.

HELD: The SC has been most liberal in sustaining ordinances based on general
welfare clause. VV.

Cruz v. Paras, 123 SCRA 569 (1983)

F: The petitioners are operators or nightclubs in Bocaue, Bulacan. they
filed prohibition suits to stop the Mun. of Bocaue from enforcing an ordinance
prohibiting the operation of nightclubs, cabarets, and dance h alls in that mun.
or the renewal of licenses to operate them. The CFI upheld the validity of the
ordinance and dismissed the petition. Hence, this petition for certiorari.

HELD: A mun. corp. cannot prohibit the operation of nightclubs. Nightclubs may
be regulated but not prevented from carrying on their business. RA 938, as
orginally enacted, granted municipalities the power to regulate the
establishment, maintenance and operation of nightclubs and the like. While it is
true that on 5/21/54, the law was amended by RA 979 w/c purported to give
municipalities the power not only to regulate but likewise to prohibit the
operation of nightclubs, the fact is that the title of the law remained the same so
that the power granted to municipalities remains that of regulation, not
prohibition. To construe the amendatory act as granting mun. corporations the
power to prohibit the operation of nightclubs would be to construe it in a way
that it violates the constitutional provision that "every bill shall embrace only
one subject which shall be expressed in the title thereof." Moreover, the
recentyly-enacted LGC (BP 337) speaks simply of the power to regulate the
establishment, and operation of billiard pools, theatrical performances, circuses
and other forms of entertainment. Certiorari granted. VV.


C. Due Process and Eminent Domain

The taking by the State of private property in an expropriation
proceeding must be: (1) for public use, (2) with just compensation, and (3)
upon observance of due process.

Article III, Sec. 9. Private property shall not be take for public use
without just compensation.

Article XII, Sec. 18. The State may, in the interest of national welfare or
defense, establish and operate vital industries and, upon payment of just
compensation, transfer to public ownership utilities and other private
enterprises to be operated by the government.


1. Taking either for public use or public purpose.

Public Use

Public use is equivalent to public purpose. It is not confined merely to use
by the public at large (e.g. roads). It is enough that it serves a public purpose,
even if it benefit a large group of people short of the public in general (e.g.
expropriating property for the relocation of squatters).

Heirs of Juancho Ardona v. Reyes 123 SCRA 220

F: The Philippine Tourism Authority sought the expropriation of 282 Ha of
land in Barangay Malubog and Babag in Cebu City. upon deposit of an amount
equivalent to 10% of the value of the property, the CFI authorized the PTA to
take immediate possession of the property. The charter of the PTA authorizes it
to acquire through condemnation proceedings lands for tourist zone
development of a sports complex. The petitioners who are occupants of the
lands, filed a petition for certiorari in the SC. They contended that (1) the taking
was not for public use; (2) the land was covered by the land reform program;
and (3) expropriation would impair the obligation of contracts.

HELD: The concept of public use is not limited to traditional purposes for the
construction of roads, bridges, and the like. The idea that "public use" means
"use by the public" has been discarded. As long as the purpose of the taking is
public, then the power of eminent domain comes into play. It is accurate to state
then that at present whatever may be beneficially employed for the general
welfare satisfies the requirement of public use. The petititioners have not
shown that the area being developed is land reform area and that the affected
persons have been given emancipation patents and certificates of land transfer.
The contract clause has never been regarded as a barrier to the exercise of the
police power and likewise eminent domain. VV.

Sumulong v. Guerrero 154 SCRA 461 (1987)

F: On December 5, 1977, the National Housing Authority filed a complaint
for the expropriation of 25 hectares of land in Antipolo, Rizal pursuant to PD
1224 authorizing the expropriation of private lands for socialized housing.
Among those lands sought to be expropriated are the petitioners' lands. They
brought this suit in the SC challenging the constitutionality of PD 1224.

HELD: Petitioners contend that socialized housing for the purpose of
condemnation proceedings is not public use since it will benefit only a handful of
people. The "public use" requirement is an evolving concept influences by
changing conditions. Urban renewal or redevelopment and the construction of
low-cost housing is recognized as a public purpose, not only because of the
expanded concept of public use but also because of specific provisions in the
Constitution. Shortage in housing is a matter of state concern since it directly
and significantly affects public health, safety, the environment and, in sum, the
general welfare. Petitioners claim that there are vast areas of lands in Rizal
hundreds of hectares of which are owned by a few landowners only. Why should
the NHA pick their small lots? Expropriation is not confined to landed estates.
The test to be applied for a valid expropriation of private lands was the area of
the land and not the number of people who stood to be benefitted. The State
acting through the NHA is vested with broad discretion to designate the
property. The property owner may not interpose objections merely because in
their judgment some other property would have been more suitable. The
provisions on just compensation found in PD 1224, 1259, and 1313 are the same
provisions which were declared unconstitutional in EPZA v. Dulay (1987) for
being encroachments on judicial prerogatives. VV.


2. Just compensation must be judicially determined

Just Compensation

Just compensation is the fair and reasonable equivalent of the loss
sustained by the owner of the property due to the taking; it is the fair market
value of the property measured at the time of the taking, no matter how long
ago it was taken (e.g. the time of the taking was in the 1920's, the time of
payment was in the 1960's, in the Ministerio and Amigable cases, supra), and
using the conversion rates at the time of taking (because according to those
cases, Art. 1250 of the Civil Code applied only to contractual obligations).


EPZA v. Dulay 149 SCRA 305 (1987)

F: The San Antonio Development Corporation was the owner of a piece of
land in Lapu-Lapu City which the EPZA expropriated in 1979. The commissioners
appointed by the trial court recommended that the San Antonio Development
Corp. be paid P15.00 per square meter. EPZA filed a petition for certiorari,
arguing that under PD 1533 the compensation should be the fair and current
market value declared by the owner or the market value determined by the
assessor, whichever is lower.

HELD: The method of ascertaining just compensation under PD 1533 constitutes
impermissible encroachment on judicial prerogatives. Although the court
technically would still have the power to determine the just compensation for
the property, following the decree, its task would be relegated to simply stating
the lower value of the property as declared either by the owner or the assessor.
Just compensation means the value of the property at the time of the taking. It
means a fair and full equivalent for the loss sustained. All the facts as to the
condition of the property and its surroundings, its improvements and
capabilities should be considered. In this case, the tax declarations used as basis
for the just compensation were made long before the declaration of martial law
when the land was much cheaper. To peg the value of the lots on the basis of
those documents which are outdated would be arbitrary and confiscatory. VV.


Manotok v. NHA 150 SCRA 89 (1987)

F: Petitioners are the owners of two large estates known as the Tambunting
Estate and Sunog-Apog in Tondo, Manila, both of which were declared
expropriated in two decrees issued by President Marcos, PD 1669 and PD 1670.
The petitioners contend that the decrees violate their constitutional right to due
process and equal protection since by their mere passage their properties were
automatically expropriated and they were immediately deprived of the
ownership and possession thereof without being given the chance to oppose
such expropriation. The government on the other hand contends that the power
of eminent domain is inherent in the State and when the legislature or the
President through his law-making powers exercises this power, the public use
and public necessity of the expropriation and the fixing of the just compensation
become political in nature and the courts must respect the decision.

HELD: The challenged decrees are unfair in the procedures adopted and the
powers given to the NHA. The Tambunting subdivision is summarily proclaimed a
blighted area and directly expropriated by decree without the slightest
CONSTI TUTI ONAL LAW 2 ZPG & ASSOCI ATES

semblance of a hearing or any proceeding whatsoever. The expropriation is
instant and automatic to take effect immediately upon the signing of the decree.
No deposit before the taking is required. There is not provision for any interest
to be paid upon unpaid installments. Not only are the owners given absolutely
no opportunity to contest the expropriation, or question the amount of
payments fixed by the decree, but the decision of the NHA are expressly
declared beyond judicial review. PD 1669 and 1670 are declared
unconstitutional.

Teehankee, CJ, concurring: The judgment at bar now learly overturns the
majority ruling in JM Tuason v. LTA that the power of Congress to designate the
particular property to be taken adn how much may be condemned thereof must
be duly recognized, leaving only as a judicial question whether in the exercise of
such competence, the party adversely affected is the victim of partiality and
prejudice. The SC now rules that such singling out of properties does not
foreclose judicial scrutiny as to whether such expropriation by legislative act
transgresses the due process and equal protection and just compensation
guarantees of the Constitution. VV.



3. Due process must be observed

De Knecht v. Bautista 100 SCRA 660 (1980)

F: The plan to extend EDSA to Roxas Boulevard to be ultimately linked to the
Cavite Coastal Road Project, originally called for the expropriation of properties
along Cuneta Avenue in Pasay City. Later on, however, the Ministry of Public
Highways decided to make the proposed extension pass through Fernando Rein
and Del Pan Streets. Because of the protests of residents of the latter, the
Commission on Human Settlements recommended the reversion to the original
plan, but the Ministry argued the new route withh save the government P2
million. The government filed expropriation proceedings against the owners of
Fernando Rein and Del Pan streets, among whom was petitioner.

HELD: The choice of Fernando Rein and Del Pan streets is arbitrayr and should
not receive judicial aprpoval. The Human Settlements Commission concluded
that the cost factor is so minimal that it can be disregarded in making a choice
between the two lines. The factor of functionality strongly militates against the
choice of Fernando Rein and Del Pan streets, while the factor of social and
economic impact bears grievously on the residents of Cuneta Avenue. While the
issue would seem to boil down to a choice between people, on one hand, and
progress and development, on the other, it is to be remembered that progress
and development are carried out for the benefit of the people. VV.





Republic v. De Knecht, 182 SCRA 142 (1990)

F: De Knecht was one of the owners of several properties along the
Fernando Rein-Del Pan streets which the Government sought to expropriate to
give way to the extension of EDSA and the construction of drainage facilities. De
Knecht filed a case to restrain the Government from proceeding with the
expropriation. Her prayer was denied by the lower court but upon certiorari, the
SC reversed the lower court decision and granted the relief asked for by De
Knecht ruling that the expropriation was arbitrary. The case was remanded to
the lower court.
No further action was taken despite the SC decision until two years
later, in 1983, when the Government moved for the dismissal of the case on the
ground that the Legislature has since enacted BP 340 expropriating the same
properties for the same purpose. The lower court denied tthe motion. Appeal.

RULING: While it is true that said final judgment of this Curt on the subject
becomes the law of the case between the parties, it is equally true that the right
of petitioner to take private properties for public use upon payment of just
compensation is so provided in the Constitution and the laws. Such
expropriation proceeding may be undertaken by the petitioner not only by
voluntary negotiation with the land owners but also by taking appropriate court
action or by legislation.
When BP 340 was passed, it appears that it was based on supervening
events that occured after the 1980 decision of the SC on the De Knecht case was
rendered. The social impact factor which persuaded the Court to consider this
extension to be arbitrary had disappeared.
Moreover, the said decision is no obstacle to the legislative arm of the
Government in thereafter making its own independent assessment of the
circumstances then pravailing as to the propriety of undertaking the
expropriation of properties in question and thereafter by enacting the
corresponding legislation as it did in this case. The Court agrees in the wisdom
and necessity of enacting BP 340. Thus the anterior decision of the Court must
yield to the subsequent legislative fiat. Charo.






NAPOCOR v. Gutierrez, 193 SCRA 1 (1991)

F: For the construction of its 230 KV Mexico-Limay transmission lines,
Napocor's lines have to pass the lands belonging to respondents. Unsuccessful
with its negotiations for the acquisition of the right of way easements, Napocor
was constrained to file eminent domain proceedings.

ISSUE: W/N petitoner should be made to pay simple easement fee or full
compensation for the land traversed by its transmissin lines.

RULING: In RP v. PLDT, the SC ruled that "Normally, the power of eminent
domain results in the taking or appropriation of the title to, and possession of,
the expropriated property, but no cogent reason appears why said power may
not be availed of to impose only a burrden upon the owner of the condemned
property, without loss of title or possession. It is unquestionable that real
property may, through expropriation, be subjected to an easement of right of
way." In this case, the easement is definitely a taking under the power of
eminent domain. Considering the nature and effect of the installation of the
transmission lines, the limitations imposed by the NPC against the use of the
land (that no plant higher than 3 meters is allowed below the lines) for an
indefinite period deprives private respondents of ts ordinary use.
For these reasons, the owner of the property expropriated is entitled to a
just compensation which should neither be more nor less, whenever it is
possible to make the assessment, than the money equivalent of said property.
Just equiivalent has always been understood to be the just and complete
equivalent of the loss which the owner of the thing expropriated has to suffer by
reason of the expropriation. The price or value of the land and its character at
the time of taking by the Govt. are the criteria for determining just
cmpensation. Charo.


D. Equal Protection

Art. III, Sec. 1. No person shall be deprived of life, liberty or property
without due process of law, nor shall any person be denied the equal
protection of the laws.

Art. XIII, Sec. 1. The Congress shall give highest priority to the
enactment of measure that protect and enhance the right of all the people to
human dignity, reduce social, economic, and political inequalities and remove
cultural inequities by equitably diffusing wealth and political power for the
common good.
To this end, the State shall regulate the acquisition, ownership, use, and
disposition of property and its increments.


1. Economic equality


Art. XIII, Sec. 2. The promotion of social justice shall include the
commitment to create economic opportunities based on freedom of initiative
and self-reliance.

Art. XIII, Sec. 3. The State shall afford full protection to labor, local and
overseas, organized and unorganized, and promote full employment and
equality of employment opportunities for all.
It shall guarantee the rights of all workers to self-organization, collective
bargaining and negotiations, and peaceful concerted activities, including the
right to strike in accordance with law. They shall be entitle to security of
tenure, humane conditions of work, and living wage. They shall also
participate in policy and decision-making process affecting the rights and
benefits as may be provided by law.
The State shall promote the principle of shared responsibility between
workers and employers and the preferential use of voluntary modes in settling
disputes including conciliation, and shall enforce their mutual compliance
therewith to foster industrial peace.
The State shall regulate the relations between workers and employers,
recognizing the right of labor to its just share in the fruits of production and
the right of enterprises to reasonable returns on investments, and to
expansion and growth.


Art. XII, Sec. 10. The Congress shall, upon recommendation of the
economic and planning agency, when the national interest dictates, reserve to
citizens of the Philippines or to corporations or associations at least sixty per
centum of whose capital is owned by such citizens, or such higher percentage
as Congress may prescribe, certain areas of investments. The Congress shall
enact measures that will encourage the formation and operation enterprises
whose capital is wholly owned by Filipinos.
In the grant of rights, privileges and concessions covering the national
economy and patrimony, the State shall give preference to qualified Filipinos.
CONSTI TUTI ONAL LAW 2 ZPG & ASSOCI ATES

The State shall regulate and exercise authority over foreign investments
within its national jurisdiction and in accordance with its national goals and
priorities.

Id., Sec. 2. xxx
The State shall protect the nation's marine wealth in its archipelagic
waters, territorial sea, and exclusive economic zone, and reserve its use and
enjoyment exclusively to Filipino citizens.

Art. III, Sec. 11. Free access to the courts and quasi-judicial bodies and
adequate legal assistance shall not be denied to any person by reason of
poverty.

Art. VIII, Sec. 5. The Supreme Court shall have the following powers:
xxx
(5) Promulgate rules concerning the protection and enforcement of
constitutional rights, pleading, practice, and procedure in all courts, the
admission to the practice of law, the Integrated Bar, and legal assistance to the
underprivileged. Such rules shall provide a simplified and inexpensive
procedure for the speedy disposition of cases, shall be uniform for all courts of
the same grade, and shall not diminish, increase, or modify substantive rights.
Rights of procedure of special courts and quasi-judicial bodies shall remain
effective unless disapproved by the Supreme Court.


There are areas of economic activity which can be limited to Filipinos.
The Constitution itself acknowledges this in various places - exploitation of
marine wealth (Art. XII, Sec. 2 par. 2), certain areas of investment (Art. XII, Sec.
10), to name a few.

In Ichong v. Hernandez, 201 Phil. 1155 (1937), the SC upheld the validity
of the law which nationalized the retail trade. For the protection of the law can
be observed by the national interest.





Ichong v. Hernandez, 201 Phil. 1155 (1937)

But there are areas where aliens cannot be kept away for the simple
reason that they cannot be deprived of a common means of livelihood,
especially when they are admitted to the country as immigrants.

In Villegas v. Hiu Chiong Isai Po Ho, 86 SCRA 270 (1978), the SC invalidated
a city ordinance imposing a P500 permit fee for aliens who wish to engage in the
pursuit of an occupation. The SC noted that this violated the uniformity of
taxation, and deprived aliens of the right to earn a common livelihood.

Villegas v. Hiu Chiung Tsai Pao Ho 86 SCRA 270 (1978)

F: An ordinance of the City of Manila prohibited the employment of aliens in
any occupation or business unless they first secured a permit from the Mayor of
Manila and paid a fee of P500. Respondent, an alien, employed in Manila,
brought suit and obtained judgment from the CFI declaring the ordinance null
and void.

HELD: The ordinance is a tax measure. In imposing a flat rate of P500, it failed to
consider substantial differences in situations among aliens and for that reason
violates the rule on uniformity of taxation. It also lays down no guide for
granting/denying the permit and therefore permits the arbitrary exercise of
discretion by the Mayor. Finally, the ordinance denies aliens due process and the
equal protection of the laws. VV.

In Vera v. Cuevas, 90 SCRA 379 (1979), Sec. 169 of the NIRC requiring
manufacturers of skimmed milk (non-fat) to put on its label the warning that the
milk is harmful for infants, was struck down as unconstitutional on the ground
that it did not require the same labeling in the case of filled milk (coco-fat
added)

At that time of the decision thought, the law was already inoperative.

Vera v. Cuevas 90 SCRA 379 (1979)

F: Respondents are engaged in the manufacture and sale of filled milk
products. They brought an action in the CFI for a declaration of their rights in
respect of section 169 of the Tax Code. This provision required that "all
condensed skimmed milk in whatever form, from which the fatty part has been
removed totally or in part or put on sale in the Philippines shall be clearly and
legibly marked on its immediate containers with the words: This milk is not
suitable for nourishment for infants less than one year of age.

HELD: Sec. 169 of the Tax Code has been repealed by RA 344. At any rate, Sec.
169 applied only to skimmed milk and not to filled milk. Sec. 169 is being
enforced only against respondent manufacturers of filled milk but not against
manufacturers of skimmed milk, thus denying them the equal protection of the
laws. VV.


2. Political equality


Art. III, Sec. 18. (1) No person shall be detained solely by reason of his
political beliefs and aspirations.

Art. IX, C, Sec. 10. Bona fide candidates to public office shall be free
from any form of harassment and discrimination.


In Dumlao v. Comelec, 95 SCRA 392 (1980), the SC upheld the validity of
sec. 4 of Batas Blg. 52 disqualifying retired elective local officials who have
received retirement benefits and would have been 65 years old at the start of
the term. It does not violate equal protection, for it gives younger blood the
opportunity to run the local government.

Dumlao v. Comelec, 95 SCRA 392 (1980)

F: Sec. 4 of BP 52 provides in part that "any retired elective provincial, city ot
municipal official who has received payment of the retirement benefits to which
he is entitled under the law and who shall have been 65 years of age at the
commencement of the term of office to which he seeks to be elected, shall not
be qualified to run for the same elective local office from which he has retired."
Petitioner, Governor of Nueva Vizcaya, sued for prohibition to enjoin
enforcement of the law on the ground that it was contrary to the equal
protection and due process guarantee of the Constitution.

HELD: Dumlao has not been injured by the application of the provision. No
petition seeking his disqualification has been filed against him. His petition is a
mere request for advisory opinion. Nevertheless, because of public interest, the
question should be resolved. The purpose of the law is to allow the emergence
of younger blood in local governments and therefore, not invalid. The retired
employee in effect declares himself tired and unavailable for the same
government work. VV.


In Igot v. Comelec, 95 SCRA 392 (1980), however, the disqualification of
candidates convicted or simply charged with national security offenses was
struck down as unconstitutional, for violating the presumption of innocence and
thus ultimately the equal political protection.

Igot c. Comelec 95 SCRA 392 (1980)

F: Romeo Igot, as taxpayer, voter and member of the bar, and Alfredo
Salapantan Jr., as taxpayer and voter, sued for prohibition to enjoin enforcement
of BP 52, sec. 4 of which provides for the disqualification as candidate of any
person convicted of subversion, insurrection or rebellion or similar offenses.

HELD: Neither petitioner has been convicted nor charged with acts of disloyalty
nor disqualified from being candidates for local elective positions. They have no
personal or substantial interest at stake and therefore no locus standi. Neither
can they sue as taxpayers because the statute does not involve disbursement of
public funds. VV.


3. Social equality


Art. XIII, Sec. 1. The Congress shall give highest priority to the
enactment of measure that protect and enhance the right of all the people to
human dignity, reduce social, economic, and political inequalities and remove
cultural inequities by equitably diffusing wealth and political power for the
common good.
To this end, the State shall regulate the acquisition, ownership, use, and
disposition of property and its increments.
.


II. REQUIREMENTS OF FAIR PROCEDURE


A. Arrests, Searches and Seizures

Art. III, Sec. 2. The right of the people to be secure in their persons,
houses, papers and effects against unreasonable searches and seizures of
whatever nature and for any purpose, shall be inviolable, and no search
warrant or warrant of arrest shall issue except upon probable cause to be
determined personally by the judge after examination under oath or affirma-
tion of the complainant and the witnesses he may produce, and particularly
describing the place to be searched and the person or things to be seized.

CONSTI TUTI ONAL LAW 2 ZPG & ASSOCI ATES

Sec. 3. The privacy of communication and correspondence shall be
inviolable, except upon lawful order of the court, or when public safety or
order requires otherwise as prescribed by law. Any evidence obtained in
violation of this or the preceding section, shall be inadmissible for any purpose
in any proceeding.


1. Requirements for Search Warrants


Yee Sue Kuy v. Almeda, 70 Phil. 141 (1940)

F: By virtue of the sworn application of Almeda, the Chief agent of the Anti-
Usury Board, a SW was issued to search the store and premises of the petitioner,
accused of violating the Anti-Usury Law. Receipt books, PNs and other articles
were seized and retained in the possession of the Anti-Usury Board.

ISSUE: W/n the requirements for the issuance of valid SW were complied with.

RULING: YES. The applicant, Almeda, in his application, swore that "he made his
own personal investigation and ascertained that petitioner is lending money
without a license, charging usurious rates." The witness he presentted testified
before the judge and swore that he knew that YEE was lending without a license
because he personally investigated the victims who secured loans from the
petitioner. Their affidavits were sufficient for, thereunder, they could be held
liable for perjury. The existence of probable cause has been determined by the
justice of the peace before issuing the warrant complained of, as shown in the
warrant itself. Charo.


Pasion vda. de Garcia v. Locsin, 65 Phil. 689, (1938)

F: This is a petition for mandamus presented to secure the annulment of a
search warrant (SW) & 2 orders of the resp. judge, & the restoration of certain
documents alleged to have been illegally seized by an agent of the Anti-Usury
Board.
Almeda, an agent of the Anti-Usury Board, obtained from the justice of
the peace of Tarlac, Tarlac, a SW, commanding any officer of the law "to search
the person, house or store of the petitioner for certain books, lists, chits,
receipts, documents & other papers relating to her activities as userer."
On the same date, Almeda, accompanied by a captain of the PC, went
to the office of the petitioner, and after showing the SW to the petitioner's
bookeeper, Salas, & w/o the presence of the petitioner, who was ill and confined
at that time, proceeded w/ the execution thereof. Two packages of records & a
locked filing cabinet containing several papers and documents were seized by
Almeda and a receipt thereof issued by him to Salas. Separate criminal cases
were filed against petitioner. Petitioner demanded the return of the documents
seized. Bu motion, pet. challenged the legality of the SW and the devolution of
the documents demanded. By resolution, the resp. judge of CFI denied the
petitioner's motion for the reason that though the SW was illegal, there was a
waiver on the part of the petitioner.
The resolution of 10/5/37 & the order of 1/3/38 are sought, together
w/ the SW, to be nullified in these proceedings.

HELD: Freedom from unreasonable searches and seizures is declared a popular
right and for a SW to be valid, (1) it must be issued upon probable cause; (2) the
probable cause must be determined by the judge himself and not by the
applicant or another; (3) in the determination of probable cause, the judge must
examine, under oath or affirmation, the complainant and such witnesses as the
latter may produce; & (4) the warrant issued must particularly describe the place
to be searched and persons or things to be seized.
In the instant case, the existence of probable cause was determined not
by the judge himself but by the applicant. All that the judge did was to accept as
true the affidavit made by agent Almeda. It does not appear that he examined
the applicant and his witnesses, if any. Even accepting the description of the
prop. to be seized to be sufficient and on the assumption that the receipt issued
is sufficiently detailed w/in the meaning of the law, the prop. seized were not
delivered to the court w/c issued the warrant, as required by law. Instead, they
were turned over to the resp. provincial fiscal & used by him in building up cases
against petitioner. Considering that at the time the warrant was issued, there
was no case pending against the petitioner, the averment that the warrant was
issued primarily for exploration purposes is not w/o basis.

IS THERE A WAIVER? No express waiver.
IS THERE AN IMPLIED WAIVER? None.

To constitute a waiver of constitutional right, it must appear first that (1)
the right exists; (2) that the person involved had knowledge, actual or
constructive, of the existence of such right; (3) that said person had an actual
intention to relinquish the right.
It is true that the petitioner did not object to the legality of the search
when it was made. She could not have objected bec. she was sick & was not
present when the warrant was served upon Salas. Certainly, the constitutional
immunity from unreasonable searches and seizures, being a personal one,
cannot be waived by anyone except by the person whose rights are invaded or
one who is expressly authorized to do so in his or her behalf. The failure on the
part of the petitioner and her bookkeeper to resist or object to the execution of
the warrant does not constitute an implied waiver of the consti. right. It is but a
submission to the authority of the law. The delay in making the demand for the
return of the documents seized is not such as to result im implied. waiver.
RAM.


Burgos v. Chief of Staff 133 SCRA 800 (1984)

Illegal search of newspaper offices and press freedom

F: On the basis of two warrants issued by the RTC of QC, the offices of the
Metropolitan Mail and the We Forum were search and printing machines,
paraphernalia, motor vehicles and other articles used in the printing, publication
and distribution of the newspapers as well as papers and other literature seized
on the ground that they were used in the commission of the crime of
subversion. Petitioners brought and action to annul the warrants and compel the
return of the things seized.

HELD: Petitioners' thesis is impressed with merit. Probable cause for a search is
defined as such facts and circumstances which would lead a reasonably discreet
and prudent man to believe that an offense has been committed and that the
objects sought in connection with the offense are in the place sought to be
searched. When addressed to a newspaper publisher or editor, the application
for a warrant must contain a specification stating with particularity the alleged
subversive materials he has published or intending to publish. Broad statement
in the application is a mere conclusion of law and does not satisfy the
requirement of probable cause. Another factor that makes the search warrants
constitutionally objectionable is that they are in the nature of general warrants.
In Stanford v. State of Texas, the US SC declared this type of warrant void. VV.


Corro v. Lising 137 SCRA 341 (1985)

F: Respondent Judge issued a search warrant for the seizure of articles
allegedly used by petitioner in committing the crime of sedition. Seized were
printed copies of the Philippine Times, newspaper dummies, typewriters,
mimeographing machines and tape recorders, video machines and tapes. The
petitioner moved to quash the warrant but his motion was denied.

HELD: The statements made in the affidavits are mere conclusions of law and do
not satisfy the requirement of probable cause. The language used is all
embracing as to include all conceivable words and equipment of petitioner
regardless of whether they are legal or illegal. The search warrant under
consideration was in the nature of a general warrant which is objectionable. VV.


Bache & Co. (Phil.) v. Ruiz, 37 SCRA 823 (1971)

F: The Com. of Internal Revenue through Rev. Examiner de Leon filed an
application for a SW against Bache & Co. and its pres., Seggerman for violation of
the provisons of the NIRC. As Judge Ruiz was then conducting a hearing, the
deposition of de Leon and his witness, Logronio, was taken by the Dep. Clerk of
Court. The deposition was later read to the judge who asked the witness to take
an oath as to the truth of his statements. The judge then signed the SW and
accordingly issued the same.

ISSUE: W/n the requirements for the issuance of valid SW were complied with.

RULING: NO. The judge did not personally examine the complainant and his
witnesses. The judge did not have the opportunity to observe tthe demeanor of
the deponents and to propound initial and follow-up questions which his judicial
mind, on account of his training, was in the best position to conceive. This is
important in arriving at a sound inference on the all-importatnt question of w/n
there was probable cause. Charo.


Prudente v. Dayrit, 180 SCRA 69 (1990)

F: Judge Dayrit, upon applicatin of P/Maj. Dimagmaliw, supported by a
"Deposition of Witness," executed by P/Lt. Angeles, issued a search warrant for
the search and seizure of arms, ammunitions and explosives in the premises of
the PUP which were supposed to be in possession of Dr. Prudente. In enforcing
the warrant, 3 fragmentation grenades were found in the bathroom of the office
of Dr. Prudente.

ISSUE: W/n the searrch warrant was valid.

RULING: NO.
(1) The warant was not issued on the basis of personal knowledge of the
applicant and his witness. The probable cause required under the Constitution
for the issuance of a search warrant must be in connection with one specific
offense, and the judge must, before issuing the warrant, personally examine in
the form of searching questions and answers, in writing and under oath, the
CONSTI TUTI ONAL LAW 2 ZPG & ASSOCI ATES

complainant and any witnesses he may produce, on facts personally known to
them and attach to the record their sworn statements together with any
affidavit submitted. However, in the case at bar, Dimagmaliw merely stated in
his application that his knowledge was based "on gathered infrmation from
verified sources." The same holds true for the affidavit of Angeles.
Moreover, the judge did not examine Angeles in the form of searching
questions and answers. What appears on the record are leading questions
answereable by yes or no.

(2) As to the claim that the SW failed to particularly describe the place to
be searched, the SC ruled that the description of the place to be searched is
sufficient if the officer with the warrant can, with reasonable effort, ascertain
and identify the place intended to be searched. Here, the SW described the
place as PUP, with its address and specifically mentioned the offices of the
"Dept. of Military Science and Tactics on the ground floor and the Office of the
President at the 2nd floor and the other rooms in that floor." This is deemed
sufficient.

(3) There was also an issue as to w/n the SW was issued for one specific
offense. The warrrant was issued for violation of PD 1866 which punishes several
offenses. While there was failure to state the particular provision of the law
violated, the warrant itself qualified the description of the offense as "illegal
possession of firearms, etc." This suffices to cure the defect.
The fact that the word "etc." was added to refer to ammunitions and
explosives did not violate the rule on single offense, for notwithstanding that
possession of firearms, explosives and ammunitions are punished in different
sections of the PD, they are treated as belonging to a single specie. An exception
to the rule that a warant shall be issued for a single offense is when existing laws
prescribe a single punishment for various offenses. Charo.


Olaes v. People, 155 SCRA 486 (1987)

F: Petitioners claim that the SW issued by resp. judge was invalid. They also
question the extrajudicial confession taken from them without according them
the right to assistance of a counsel. The articles seized by virtue of the SW
consisting of dried marijuana were admitted as evidence for violation of RA 6425
or Dangerous Drugs Act.

RULING: While it is true that the caption of the SW states that it is in connection
with "the violation of RA 6425," it is clearly recited in the text thereof that "there
is probable cause to believe that Olaes of Olongapo City has in his possession
and control, marijuana dried stalks which are suject of the offense stated
above." Although the specific section of the law is not stated, there is no
question at all that the specific offense alleged to have been committed as basis
for determining probable cause is alleged. Furthermore, the SW specifically
described the place to be searched and the things to be seized.

As to the extrajudicial confessions of the accused, the same are deemed
inadmissible against them. In People V. Galit, the Court declared that: " At the
time the person is arrested, it shall be the duty of the arresting officer to inform
him of the reason for the arrrest and he must be shown the warrant of arrest, if
any; he shall be informed of his constitutional rights to remain silent and to
counsel, and that any statement he might make could be used against him. The
person arrested shall have the right to communicate with his lawyer, a relative,
or anyone he chooses by the most expedient means - by telephone if possible -
or by letter or messenger. It shall be the responsibility of the arresting officer to
see to it that this is accomplished. No custodial investigation shall be conducted
unless it be in the presence of counsel engaged by the person arrested, by any
person on his behalf, or appointed by the court upon petition either of the
detainee himself or by anyone on his behalf. The right to counsel may be waived
but the wiaver shall not be valid unless made with the assistance of counsel. Any
statement obtained in violation of the procedure herein laid down, whether
exculpatory or inculpatory, in whole or in part, shall be inadmissible in
evidence."

These requirements were even made stricter under the 1987 Constitution
which provides that the rights of a person under custodial investigation cannot
be waived except when made in writing and in the presence of counsel. Charo.


Presidential Anti-Dollar Salting Task Force v. CA, 171 SCRA 348 (1989)

F: The PASTF was created by virtue of PD 1936 to serve as the President's
arm called upon to combat the vice of dollar salting or the blackmarketing and
salting of foreign exchange.

ISSUE: W/N the PASTF is "such other officer as may be authorized by law" to
issue warrants under the 1973 Constitition.

RULING: NO. The Court, in reviewing the powers of the PASTF under its enabling
law, sees nothing that will reveal a legislative intendement to confer upon the
body, quasi-judicial responsiibilities relative to offenses punishable by PD 1883.
Its undertaking is simply to determine w/n probable cause exists to warrant the
filing of charges with the proper court, meaning to say, to conduct an inquiry
preliminary to a judicial recourse, and to recommend action of appropriate
authorities.
The Court agrees that PASTF exercises, or was meant to exercise,
prosecutorial powers, and on that ground, it cannot be said to be a neutral and
detached judge to determine the existence of probable cause for purposes of
arrest or search. Unlike a magistrate, a prosecutor is naturally interested in the
success of his case. Although his office "is to see to it that justice if done and not
necessarily to secure the conviction of the accused," he stands invariably, as the
accused's adversary and his accuser. To permit him to issue warrrants and
indeed, warrants of arrest, is to make him both judge and jury in his own right,
when he is neither. This makes to our mind and to that extent, PD 1636 as
amended by PD 2002, unconstitutional.
The "responsible officer" referred to under the Cosntitution is one not
only possessing the necessary skills and competence but more significantly, the
neutrality and independence comparable to the impartiality presumed of a
judicial officer.


Salazar v. Achacoso, 183 SCRA 145

F: Pursuant to the powers vested by PD 1920 and EO 1022, POEA
Administrator Achacoso ordered the closure of the recruitment agency of Horty
Salazar, having verified that she had no license to operate a recruitment agency.
He further ordered the seizure of the documents and paraphernalias, being used
or intended to be used as the means of commiting illegal recruitment. This
order was enforced on 26 January 1988. Petitioner filed this suit for
prohibition.

Issue: May the POEA (or the Sec. of Labor) validly issue warrants of serach and
seizure (or arrest ) under Art. 38 of the Labor Code?

HELD: NO.
The provisions of PD 1920 and EO 1022, now embodied in Art. 38 of the
Labor Code, are the dying vestiges of authoritarian rule in its twilights moments.
Under Art. III, Sec 2 of the 1987 Constitution, it is only judges and no other, who
may issue warrants of arrest and search. The exception is in cases of
deportation of illegal and undesirable aliens, whom the President of the
Commissioner of Immigration may order arrested, following a final order of
deportation, for the purpose of deportation. The Sec. of Labor , not being a
judge. may no longer issue search or arrest warrants. Hence, the authorities
must go through the judicial process. To that extent, we declare Art. 38, par. C
of the Labor Code, unconstitutional and of no force and effect.


a. Existence of probable cause.

Probable cause is such facts and circumstances as would reasonably make
a prudent man believe that a crime have been committed and that the
documents or things sought to be searched and seized are in the possession of
the person against whom the warrant is sought. Without probable cause, there
can be no valid search warrant.

In Stonehill v. Diokno, 20 SCRA 385 (1967), 42 search warrants
were issued for alleged violation of Central Bank Laws, the Tariff and Customs
Code, the NIRC, and the Revised Penal Code. The SC voided the warrants on the
ground that it was impossible for the judge to have found probable cause in view
of the number of laws alleged to have been violated by the petitioner. How
could he even know what particular provision of each law had been violated? If
he did not know this, how could it be determined if the person against whom
the warrant was issued was probably guilty thereof? In truth, this was a fishing
expedition, which violated the sanctity of domicile and privacy of
communications. To establish the requirement of probable cause, the rule is:
One crime, one warrant.

b. As determined by a judge

Under the 1987 Constitution, only a judge can issue a warrant; the
offensive and much abused phrase "and other responsible officer as may be
authorized by law" in the 1973 Constitution has been removed

c. After personally examining under oath or affirmation the complainant and
his witness.

The examination conducted by the judge takes the form of searching
questions.

The requirement that the judge must personally examine the complainant
and his witnesses means that the actual examination cannot be delegated to
someone else, like the clerk of court.

So said the Court in Bache and Co. (Phil) v. Ruiz, 37 SCRA 823 (1971). In
this case, when the BIR agent and his witnesses arrived in court in the middle of
a hearing, the judge suspended the hearing and directed the branch clerk to
examine and take the testimony of the witnesses in his chambers. After he was
through with the hearing, he went back to his chambers and finding that the
CONSTI TUTI ONAL LAW 2 ZPG & ASSOCI ATES

examination was finished, asked the BIR agent and his witnesses if they affirmed
what they what they testified to, after which he issued the search warrant in
question.

d. On the basis of their personal knowledge of the facts that they are
testifying to.

The determination of the reasonableness of the judicial warrant must be
based on the affidavit of one who has personal knowledge of the facts to which
he testifies. The testimony cannot be based on mere belief. Neither can it be
based on a report. Otherwise, the warrant is void.

Thus, in Burgos v. Chief of Staff, 133 SCRA 800 (1984), reiterating the 1937
case of Rodriguez v. Villamiel, the testimony based on a military report that the
newspaper We Forum was used for subversive were held to be not a personal
knowledge and so was inadmissible.

Likewise, in Corro v. Lising, 137 SCRA 541 (1985), the testimony based on
investigation reports that certain items in the Philippine Times were subversive
were held to be not personal knowledge, and thus the search warrant issued
was not valid.

e. The search warrant must describe particularly describe the place to be
searched and the things to be seized.

Failure to state with particularity the place to be searched and items to be
seized makes the warrant used for fishing evidence (a general warrant) which is
void.

In Burgos v. Chief of Staff, the description which read "subversive
documents, leaflets, papers to promote the objective of the Movement for a
Free Philippines, the Light a Fire Movement, and the April 6 Movement" were
held not to be particular descriptions, thus making the warrant a general
warrant.

In Corro v. Lising, the search and seizure of "printed copies and dummies
of Philippine Times, subversive documents, articles, printed matters, handbills,
leaflets, banners, and typewriters, tape recorders, etc." was again invalidated for
the description was not at all particular or specific, thus making the warrants
general warrants.

When it comes to printed matters, the offensive material need not be
set out in full. It is enough if it specifies the issues and the title of the articles.
The instruction to seize "subversive materials" is not valid because the
determination of whether a material is subversive or not is not for the police
officer to decide; no unfettered discretion must be granted to him.

The matter is different if goods were searched and seized because of their
intrinsic quality (as when they are stolen or smuggled), than if the goods were
searched for the ideas they contain (as when a "subversive newspaper is
sought). In the latter case, a more detailed description of the physical features
of the item is required to avoid delegating the appreciation of ideas, and thus
threaten free expression.

2. When Search May Be Made without a Warrant

(a) When search is made of moving vehicles

The reason is the person may escape easily if a warrant has to be applied
for the mean time. In the Tariff and Customs Code, customs agents are
specifically authorized to search and seize vehicles even without a warrant.

Checkpoints are valid in some instances depending on the purpose (e.g.
apprehend a suspected criminal) and the circumstances (e.g. probable cause
that the criminal is inside the car). There is no question that when a child has
been reported kidnapped in a community, the police can stop all cars and check
if the detained child is in any one of them.

(b) When search is an an incident to a valid arrest.

Rule 126, Sec. 12. Search incident to lawful arrest.-- A person lawfully
arrested may be searched for dangerous weapons or anything which may be
used as proof of the commission of an offense, without a search warrant.
(Rules of Court.)

A person arrested may be searched for dangerous weapons or anything
that proves the commission of the offense. It follows that the search can only be
made within the area of control of the arrested person, and within the time of
the arrest.

In Nolasco v. Cruz Pano, 139 SCRA 152 (1985); Milagros Roque and
Cynthia Nolasco were arrested at the intersection of Mayon and Margal Streets
in QC at 11:30 a.m., having been wanted as high officers of the CPP. At 12:00
noon, Roque's apartment located 2 blocks away, was searched and some
documents seized. The SC at first held that the search was valid even if the
warrant issued was void for failing to describe with particularity the things to be
seized, because it was an incident of a valid arrest.

But after the EDSA revolution, the reconstituted SC granted the motion
for reconsideration and held that just because there was a valid arrest did not
mean that the search was likewise valid. To be valid, the search must be
"incidental" to the arrest, that is, not separated by time or place from the
arrest. If the basis for allowing incidental searches is looked into, one can see
that this situation is not one involving a valid incidental search.

The law allows the arresting officer to search a person validly arrested (by
frisking him for instance) because (a) a weapon held by the arrested person may
be turned against his captor and (b) he may destroy the proof of the crime, if the
arrested officer has to first apply for a search warrant from a judge.

If, in the Nolasco case, the search was conducted 30 minutes after the
arrest, there is no longer any danger that the captured may turn against the
captor; and if the documents in the apartment were 2 blocks away, the search
would no longer be justified since there is no way for Roque to go back to the
apartment and destroy the documents, having been arrested already.


Nolasco v. Cruz Pao 139 SCRA 152 (1985)

F: Milagros Aguilar-Roque was arrested together with Cynthia Nolasco by
the Constabulary Security Group. Milagrso had been wanted as a high ranking
officer of the CPP. The arrest took place at 11:30 a.m. of August 6, 1984. At noon
of the same day, her premises were searched and 428 documents, a portable
typewriter and 2 boxes were seized. Earlier that day, Judge Cruz Pao issued a
search warrant for rebellion against Milagros. On the basis of the documents
seized, charges of subversion and rebellion were filed but the fiscal's office
merely charged her and Nolasco with illegal possession of subversive materials.
Milagros asked for suppression of the evidence on the ground that it was
illegally obtained. The search warrant described the things to be seized as
"Documents, papers and other records of the CPP, NPA and NDF, xxx".

HELD: The search warrant is void because it fails to describe with particularity
the things to be seized. It does not specify what the subversive books and
instructions are and what the manuals not otherwise available to the public
contain to make them subversive. There is absent a definite guideline as to what
items might lawfully be seized, thus giving the officers discretion regarding what
articles they should seize. It is thus in the nature of a general warrant. But the
seizure of the articles could be justified as an incident of a valid arrest. It is a
general rule that, as an incident of an arrest, the place of premises where the
arrest was made can also be searched without a search warrant.

(c) When things seized are within plain view of a searching party

Roan v. Gonzales, 145 SCRA 687 (1986)

F: The challenged SW was issued by the resp. judge on 5/10/84. The
petitioner's house was searched 2 days later but none of the articles listed in the
warrant was discovered. The officers conducting the search found 1 colt
Magnum revolver & 18 live bullets w/c they confiscated. They are now the
bases of the charge against the petitioner.

RULING: Search warrant issued by resp. judge is hereby declared null and void
and accordingly set aside.

The petitioner claims that no depositions were taken by the resp. judge in
accordance w/ Rule 126, Sec. 4 of the ROC, but this is not entirely true.
Depositions were taken of the complainant's 2 witnesses in addition to the
affidavit executed by them. It is correct to say, however, that the complainant
himself was not subjected to a similar interrogation.
By his own accounts, all that resp. judge did was question Capt. Quillosa
on the contents of his affidavit only "to ascertain among others, if he knew and
understood the same," and only bec. "the application was not yet subscribed
and sworn to." The suggestion is that he would not have asked any questions at
all if the affidavit had already been completed when it was submitted to him. In
any case, he did not ask his own searching questions. He limited himself to the
contents of the affidavit. He did not take the applicant's deposition in writing
and attach them to the record, together w/ the affidavit presented to him. Such
written deposition is necessary in order that the Judge may be able to properly
determine the existence or non-existence of the probable cause, to hold liable
for perjury the person giving it if it will be found later that his declarations are
false. (Mata v. Bayona.)
The applicant was asking for the issuance of the SW on the basis of mere
hearsay and not of info. personally known to him. His application, standing
alone, was insufficient to justify the issuance of the warrant sought. It was,
therefore, necessary for the witnesses themselves, by their own personal info.,
to establish the applicant's claims.
Even assuming then that it would have suffied to take the deposition only
of the witnesses and not of the applicant himself, there is still the question of
the sufficiency of their depositions.
CONSTI TUTI ONAL LAW 2 ZPG & ASSOCI ATES

A study of the deposition taken from witnesess Esmael Morada and Jesus
Tohilida, who both claimed to be "intelligence informers," shows that they were
in the main a mere restatement of their allegations in their affidavits, except
that they were made in the form of answers to the questions put to them by the
resp. judge.
One may well wonder why it did not occur to the resp. judge to ask how
the witness could be so certain even as to the caliber of the guns, or how far he
was from the window, or whether it was on the first floor or second floor, or
why his presence was not noticed at all, or if the acts related were really done
openly, in the full view of the witnesses, considering that these acts were against
the law. These would have been judicious questions but they were injudiciously
omitted. Instead, the declaration of the witnesses were readily accepted and
the warrant sought was issued forthwith.

SOL-GEN ARGUES THAT THE PETITIONER WAIVED WHATEVER DEFECT WHEN
THE PETITIONER VOLUNTARILY SUBMITTED TO THE SEARCH AND MANIFESTED
HIS CONFORMITY IN WRITING.

We do not agree. What we see here is pressure exerted by the military
authorities, who practically coerced the petitioner to sign the supposed waiver
as guaranty against a possible challenge later to the validity of the search they
were conducting.

Malum Prohibitum.-- It does not follow that bec. an offense is malum
prohibitum, the subject thereof is necessarily illegal per se. Motive is immaterial
in mala prohibita, but the subjects of this kind of offense may not be summarily
seized simply bec. they are prohibited. A SW is still necessary.

Motion to Quash.-- Petitioner should have, before coming to the SC, filed
a motion to quash the search warrant by the resp. judge. But as we said and did
in Burgos, "this procedural flaw notwithstanding, we take cognizance of this
petition in view of the seriousness and urgency of the consitutional issues
raised." RAM.


Pita v. CA, 178 SCRA 362 (1989)

F: Pursuant to the Anti-Smut Campaign of Mayor Ramon Bagatsng,
policemen seized and confiscated from dealers, distributors, newsstand owners
and peddlers along Manila sidewalks, magazines, publications and other reading
materials believed to be obscene, pornographic, and indecent and later burned
the seized materials in public. Among the publications seized and later burned
was "Pinoy Playboy" magazines published and co-edited by plaintiff Leo Pita.
After his injunctive relief was dismissed by the RTC and his appeal rejected by
CA, he seeks review with SC, invoking the guaranty against unreasonable
searches and seizure.

Issue: W/N the search and seizure was illegal

HELD: YES.
It is basic that searches and seizure may be done only through a judicial
warrant , otherwise, they become unreasonable and subject to challenge. In
Burgos v Chief of Staff (133 SCRA 800) , the SC countermanded the orders of the
RTC authorizing the serach of the premises WE Forum and Metropolitan Mail,
two Metro Manila Dailies, by reason of a defective warrant. There is a greater
reason in this case to reprobate the questioned raid, in the complete absence of
a warrant, valid or invalid. The fact that the instant case involves an obscenity
rap makes it no different from Burgos, a political case, because speech is speech,
whether political or "obscene".
The authorities must apply for the issuance of the a search warrant from
the judge , if in their opinion, an obscenity rap is in order. They must convince
the court that the materials sought to be seized are "obscene" and pose a clear
and present danger of an evil substantive enough to warrant State interference
and action. The judge must determine WON the same are indeed "obscene":
the question is to be resolved on a case-to-case basis and on the judge's sound
discretion. If probable cause exist, a search warrant will issue.


(d) Stop and Frisk

Posadas v. CA, 188 SCRA 288 (1990)

F: Patrolmans Ungab and Umpar, both members of the INP of the Davao
Metrodiscom assigned w/ the Intelligence Task Force, were conducting a
surveillance along Magallanes, St., Davao City. While they were w/in the
premises of the Rizal Memorial Colleges, they spotted petitioner carrying a
"buri" bag & they noticed him to be acting suspiciously. They approached the
petitioner and identified themselves as members of the INP. Petitioner
attempted to flee but was stopped by the 2. They then checked the "buri" bag
of the petitioner where they found 1 caliber .38 Smith & Wesson revolver, w/ 2
rounds of live ammunition for a .38 cal. gun, a smoke grenade, & 2 live
ammunition for a .22 cal. gun. Petitioner was brought to the police station for
further investigation. He was prosecuted for illegal possession of firearms and
ammunitions in the RTC of Davao City wherein after a plea of not guilty, and trial
on the merits, a decision was rendered finding petitioner guilty. The CA affirmed
the appealed decision in toto.
Hence, the petition for review, the main thrust of w/c is that there
being no lawful arrest or search and seizure, the items w/c were confiscated
from the possession of the petitioner are inadmissible in evidence against him.
The Sol-Gen argues that under Sec. 12, R 136 of ROC, a person lawfully
arrested may be searched for dangerous weapons or anything (w/c may be)
used as proof of a commission of an offense, w/o a SW.

HELD: From Sec. 5, R 113, ROC, it is clear that an arrest w/o a warrant may be
effected by a peace officer or private person, among others, when in his
presence the person to be arrested has committed, is actually committing, or is
attempting to commit an offense, or when an offense has in fact, just been
committed, & he has personal knowledge of the facts indicating that the person
arrested has committed it.
At the time the peace officers identified themselves and apprehended the
petitioner as he attempted to flee, they did not know that he had committed, or
was actually committing, the offense. They just suspected that he was hiding
something in the buri bag. They did not know what its contents were. The said
circumstances did not justify an arrest w/o a warrant.
However, there are many instances where a warrant & seizure can be
effected w/o necessarily being preceded by an arrest, foremost of w/c is the
'stop & search' w/o a SW at military or police checkpoints, the constitutionality
of w/c has been upheld by this Court in Valmonte v. de Villa.
As bet. a warrantless search and seizure (S & S) conducted at military or
police checkpoints and the search thereof in the case at bar, there is no question
that, indeed, the latter is more reasonable considering that, unlike in the former,
it was effected on the basis of a probable cause. The probable cause is that
when the petitioner acted suspiciously and attempted to flee w/ the buri bag,
there was a probable cause that he was concealing something illegal in the bag
and it was the right and duty of the police officers to inspect the same.
It is too much indeed to require the police officers to search the bag in the
possession of the petitioner only after they shall have obtained a SW for the
purpose. Such an exercise may prove to be useless, futile and much too late.
As the Sol-Gen said:

"The assailed S & S may still be justified as akin to
a 'stop and frisk' situation whose object is either to
determine the identity of suspicious individuals or to
maintain the status quo momentarily while the police
officers seeks to obtain more info. ... The US SC held in
Terry v. Ohio that "a police officer may in appropriate
circumstances & in an appropriate manner approach a
person for the purpose of investigating possible criminal
behaviour even though there is no probable cause to
make an arrest." In such a situation, it is reasonable for an
officer rather than simply to shrug his shoulder and allow
a crime to occur, to stop a suspicious individual briefly in
order to determine his identity or maintaing the status
quo while obtaining more info."

PETITION DENIED. RAM.


(e) When there is a valid express waiver made voluntarily and intelligently.

Waiver cannot be implied from the fact that the person consented or did
not object to the search, for it many happen that he did so only out of respect
for the authorities. The waiver must be expressly made.


People v. De lara

F: After a surveillance conducted, a buy-bust operation was conducted by
the police, as a consequence of which, accused was arrested. The accused
already pocketed the marked money and handed two foils to the police when he
sensed the presence of police operatives. He tried to retrieve the two foils but
he was prevented from doing so. He tried to escape by running inside his
house. The police pursued him and were able to subdue him. The accused
admitted that he kept prohibited drugs in his house. He even showed the
arresting officers a blue plastic bag containing prohibited drugs. The team,
together with the accused, proceeded to WPD headquarters for investigation.
During the investigation, accused was apprised of his constitutional rights to
remain silent and to have the assistance of counsel. When appellant was asked
to give a written statement, he refused to do so pending arrival of his lawyer.
Accused contends that his arrest and the seizure of the bag containing
prohibited drugs was null and void. He also contends that he was not assisted
by counsel during custodial investigation, where he was forced to sign the
photocopy of the marked money, the Receipt of Property Seized, and the
Booking and Information Sheet.

ISSUE: Whether or not the arrest of the accused and the seizure of the plastic
bag were valid.

CONSTI TUTI ONAL LAW 2 ZPG & ASSOCI ATES

RULING: YES. The accused was caught in flagrante as a result of a buy-bust
operation. There was no need for a warrant. The policemen were not only
authorized but were also under obligation to apprehend the drug pusher even
without a warrant. The policemens entry into the house of the accused without
a search warrant was in hot-pursuit of a person caught committing an offense in
flagrante. The arrest that followed the hot-pursuit was valid. The seizure of the
plastic bag was the result of the accuseds arrest inside the house. A
contemporaneous search may be conducted upon the person of the arrestee
and the immediate vicinity where the arrest was made.

ISSUE: Whether the documents signed by the accused during the investigation
were admissible in evidence.

RULING: NO. There was no showing that accused was then assisted by counsel
nor his waiver thereto put into writing. (The rejection of these evidence would
not affect the conviction of the accused in view of the abundance of other
evidence establishing his guilt.) Bam.


People v. de Gracia, 233 SCRA 716 (July 6, 1994)

F: The incidents involved in this case took place at the height of the coup
d'etat staged in December, 1989. Accused-appellant Rolando de Gracia was
charged in two separate informations for illegal possession of ammunition and
explosives in furtherance of rebellion, and for attempted homicide. Appellant
was convicted for illegal possession of firearms in furtherance of rebellion, but
was acquitted of attempted homicide.
Surveillance was undertaken by the military along EDSA because of
intelligence reports about a coup. Members of the team were
engaged by rebels in gunfire killing one member of the team. A searching team
raided the Eurocar Sales Office. They were able to find and confiscate six cartons
of M-16 ammunition, five bundles of C-4 dynamites, M-shells of different
calibers, and "molotov" bombs inside one of the rooms belonging to a certain
Col. Matillano. De Gracia was seen inside the office of Col. Matillano, holding a
C-4 and suspiciously peeping through a door. The team arrested appellant. They
were then made to sign an inventory, written in Tagalog, of the explosives and
ammunition confiscated by the raiding team. No search warrant was secured by
the raiding team. Accused was found guilty of illegal possession of firearms.
That judgment of conviction is now challenged before us in this appeal.

Issue: Whether or not there was a valid search and seizure in this case.

Ruling: YES
It is admitted that the military operatives who raided the Eurocar Sales
Office were not armed with a search warrant at that time. The raid was actually
precipitated by intelligence reports that said office was being used as
headquarters by the RAM. Prior to the raid, there was a surveillance conducted
on the premises wherein the surveillance team was fired at by a group of men
coming from the Eurocar building. When the military operatives raided the
place, the occupants thereof refused to open the door despite requests for them
to do so, thereby compelling the former to break into the office. The Eurocar
Sales Office is obviously not a gun store and it is definitely not an armory or
arsenal which are the usual depositories for explosives and ammunition. It is
primarily and solely engaged in the sale of automobiles. The presence of an
unusual quantity of high-powered firearms and explosives could not be
justifiably or even colorably explained. In addition, there was general chaos and
disorder at that time because of simultaneous and intense firing within the
vicinity of the office and in the nearby Camp Aguinaldo which was under attack
by rebel forces. The courts in the surrounding areas were obviously closed and,
for that matter, the building and houses therein were deserted.

Under the foregoing circumstances, it is our considered opinion that the
instant case falls under one of the exceptions to the prohibition against a
warrantless search. In the first place, the military operatives, taking into account
the facts obtaining in this case, had reasonable ground to believe that a crime
was being committed. There was consequently more than sufficient probable
cause to warrant their action. Furthermore, under the situation then prevailing,
the raiding team had no opportunity to apply for and secure a search warrant
from the courts. Under such urgency and exigency of the moment, a search
warrant could lawfully be dispensed with.
There are two separate statutes penalizing different offenses with
discrete penalties. The Revised Penal Code treats rebellion as a crime apart from
murder, homicide, arson, or other offenses, such as illegal possession of
firearms, that might conceivably be committed in the course of a rebellion.
Presidential Decree No. 1866 defines and punishes, as a specific offense, the
crime of illegal possession of firearms committed in the course or as part of a
rebellion.
Subject to the presence of the requisite elements in each case, unlawful
possession of an unlicensed firearm in furtherance of rebellion may give rise to
separate prosecutions for a violation of Section 1 of Presidential Decree No.
1866, and also a violation of Articles 134 and 135 of the Revised Penal Code on
rebellion. Double jeopardy in this case cannot be invoked because the first is an
offense punished by a special law while the second is a felony punished by the
Revised Penal Code, 24 with variant elements.
Presidential Decree No. 1866 imposes the death penalty where the illegal
possession of firearms and ammunition is committed in furtherance of rebellion.
At the time the offense charged in this case was committed under the
governance of that law, the imposition of the death penalty was proscribed by
the Constitution. Consequently, appellant De Gracia could only be sentenced to
serve the penalty of reclusion perpetua which was correctly meted out by the
trial court, albeit with an erroneous recommendation in connection therewith.

3. Constitutionality of checkpoints and "areal target zonings."

Valmonte v. De Villa, 170 SCRA 256 (1989)

F: On 1/20/87, the NCRDC was activated w/ the mission of conducting
security operations w/in its area or responsibility and peripheral areas, for the
purpose of establishing an effective territorial defense, maintaining peace and
order, and providing an atmosphere conducive to the social, economic and
political dev't of the NCR. As part of its duty to maitain peace and order, the
NCRDC installed checkpoints in various parts of Valenzuela and MM.
Petitioners aver that, bec. of the institution of said checkpoints, the
Valenzuela residents are worried of being harassed and of their safety being
placed at the arbitrary, capricious and whimsical disposition of the military
manning the checkpoints, considering that their cars and vehicles are being
subjected to regular searches and check-ups, especially at night or at dawn, w/o
a SW and/ or court order. Their alleged fear for their safety increased when
Benjamin Parpon, was gaunned down allegedly in cold blood by members of the
NCRDC for ignoring and/ or continuing to speed off inspite of warning shots fired
in the air.

HELD: Petitioner's concern for their safety and apprehension at being harassed
by the military manning the checkpoints are not sufficient grounds to declare
the checkpoints per se, illegal. No proof has been presented before the Court to
show that, in the course of their routine checks, the military, indeed, committed
specific violations of petitioners' rights against unlawful search and seizure of
other rights.
The constitutional right against unreasonable searches and seizures is a
personal right invocable only by those whose rights have been infringed, or
threatened to be infringed.
Not all searches and seizures are prohibited. Those w/c are reasonable
are not forbidden.
The setting up of the questioned checkpoints may be considered as a
security measure to enable the NCRDC to pursue its mission of establishing
effective territorial defense and maintaining peace and order for the benfit of
the public. Checkpoints may not also be regarded as measures to thwart plots
to destabilize the govt, in the interest of public security.
Between the inherent right of the state to protect its existence and
promote public welfare and an individual's right against a warrantless search w/c
is, however, reasonably conducted, the former should prevail.
True, the manning of checkpoints by the military is susceptible of abuse
by the military in the same manner that all governmental power is susceptible of
abuse. But, at the cost of occasional inconveninece, discomfort and even
irritation to the citizen, the checkpoints during these abnormal times, when
conducted w/in reasonable limits, are part of the price we pay for an orderly
society and a peaceful community.


Guazon v. De Villa, 181 SCRA 623 (1990)

F: This is a petition for prohibition w/ prel. inj. to prohibit the military and
police officers represented by public respondents from conducting "areal target
zonings" or "saturation drives" in MM.
The 41 petitioners state that they are all of legal age, bona fide
residents of MM and Taxpayers and leaders in their respective communities.
Accdg. to the petitioners, the "areal target zonings" or "saturation
drives" are in critical areas pinpointed by the military and police as places where
the subversives are hiding. Petitioners claim that the saturation drives follow a
common pattern of human rights abuses.
Respondents stress 2 points. First, the resps. have legal authority to
conduct saturation drives. And, second, they allege that the accusations of the
petitioners about a deliberate disregard for human rights, are total lies.
Resps. cite Art. VII, Sec. 17 of the Const.:"The Pres. shall have control of
all the executive departments, bureaus and offices. He shall ensure that the
laws are faithfully executed."
They also cite sec. 18.:"The Pres. shall be the Commander-in-chief of all
AFP and whenever it becomes necessary, he may call out such armed forces to
prevent or suppress lawless violence, invasion or rebellion. xxx

HELD: The Court believes it is highly probable that some violations were actually
committed. This is so inspite of the alleged pleas of barangay officials for the
thousands of residents"to submit themselves voluntarily for character and
personal verification." However, the remedy is not to stop all police actions,
including the essential and legitimate ones. We see nothing wrong in police
making their presence visibly felt in troubled areas. Police cannot respond to
riots or violent demonstration if they do not move in sufficient numbers. A show
of force is sometimes necesary as long as the rights of the people are protected
and not violated. A blanket prohibition such as that sought by the petitioners
CONSTI TUTI ONAL LAW 2 ZPG & ASSOCI ATES

would limit all police actions to one on one confrontation where search warrants
and warrants of arrest against specific individuals are easily procured. Anarchy
may reign if the military and the police decide to sit down in their offices bec. all
concerted drives where a show of force is present are totally prohibited.
The remedy is not an original action for prohibition brought through a
TP's suit. Where not one victim complains, and not one violator is properly
charged, the problem is not initially for the SC. It is basically one for the
executive departments and for the trial courts.
Under the circumstances of this TP's suit, there is no erring soldier or
policeman whom we can order prosecuted. In the absence of clear facts
ascertained through an orderly procedure, no permanent relief can be given at
this time. Further investigation of the petitioners' charges and a hard look by
admin. officials at the policy implications of the prayed for blanket prohibition
are also warranted.
In the meantime, and in the face of a prima facie showing that some
abuses were probably committed and could be committed during future police
actions, we have to temporarily restrain the alleged baning on walls, the kicking
in of doors, the herding of half-naked men to assembly areas for examination of
tattoo marks, the violation of residences even if these are humble shanties of
squatters, and other alleged acts w/c are shocking to the conscience.


4. Wire Tapping


Gaanan v. IAC, 145 SCRA 112 (1986)

F: Complainant Atty. Pintor and Montebon offered to withdraw the
complaint for direct assault they filed against Laconico after demanding P8,000
from him. This demand was heard by Atty. Gaanan through a telephone
extension as requested by Laconico so as to personally hear the proposed
conditions for the settlement. Atty. Pintor was subsequently arrested in an
entrapment operation upon receipt of the money. Since Atty. Gaanan listened
to the telephone conversation without complainant's consent, complainant
charged Gaanan and Laconico with violation of the Anti- Wiretapping Act (RA
4200).

ISSUE: W/N an extension telephone is among the prohibited devices in Sec. 1 of
RA 4200 such that iuts use to overhear a private conversation would constitute
an unlawful interception of communication between 2 parties using a telephone
line.

HELD: NO
An extension tel. cannot be placed in the same category as a dictaphone,
dictagraph, or other devices enumerated in Sec. 1 of the law as the use thereof
cannot be considered as "tapping" the wire or cable of a telephone line. This
section refers to instruments whose installation or presence cannot be
presumed by the party or parties being overheard because, by their very nature,
they are not of common usage and their purpose is precisely for tapping,
intercepting, or recording a tel. conversation. The tel. extension in this case was
not installed for that purpose. It just happened to be there for ordinary office
use.
Furthermore, it is a general rule that penal statutes must be construed
strictly in favor of the accused. Thus in the case of doubt as in this case, on WON
an extension tel. is included in the phrase "device or arrangement" the penal
statute must be construed as not including an extension tel.
A perusal of the Senate Congressional Record shows that our lawmakers
intended to discourage, through punishment, persons suchj as government
authorities or representatives of organized groups from installing devices in
order to gather evidence for use in court or to intimidate, blackmail or gain some
unwarranted advantage over the tel. users. Consequently, the mere act of
listening , in order to be punishable must stricly be with the use of the
enumerated devices in RA 4200 or others of similar nature.

5. What may be seized

Rule 126, sec. 2. Personal property to be seized.-- A search warrant may
be issued for the search and seizure of the following personal property:
(a) Subject matter of the offense;
(b) Stolen or embezzled and other proceeds or fruits of the offense; and
(c) Used or intended to be used as a means of committing an offense.
(Rules of Court.)


6. Exclusionary Rule

Art. III, Sec. 3. xxx
(2) Any evidence obtained in violation of this (privacy of communication
and correspondence) or the preceding section (unreasonable searches and
seizures) shall be inadmissible for any purpose in any proceeding.

One of the remedies of one who was victimized by an illegal search is to
ask for the suppression of the things seized and the evidence illegally taken.

The exclusionary rule prohibits the use of any evidence obtained in
violation of secs. 2 and 3 (1) of Art. III for "any purpose" and in "any
proceeding". The evidence is absolutely useless. This has not always been the
case.

In Moncado v. People's Court (1948), the SC, following the U.S. case of
Wolf V. Colorado, rules that evidence illegally obtained is not necessarily
excluded if is otherwise admissible under the rules of evidence. In such case, the
evidence admitted, without prejudice to any criminal, civil or administrative
liability of the officer who illegally seized it. In other words, the admissibility of
the evidence is not effected by the illegality of the means by which it was
acquired.

It was in Stonehill v. Diokno, supra, following the U.S. case of Maop v.
Ohio 1969, when the exclusionary rule was first adopted in the Philippines, the
SC noting that the total suppression of the thing seized is the only effective
means of ensuring the constitutional right which it seeks to preserve. The Court
noted, the insufficiency of the other remedies (e.g. action for damages, criminal
punishment, resistance), especially in the Philippines where violations were
committed by those in power and were thus equipped with the pardoning
power to water down the gravity of the other penalties imposed to violators of
those constitutional rights.

The victim may or may not get back the thing seized, depending on
whether it is contraband or not. It the thing is contraband, it would not be
returned, and only its suppression can be asked for. But if the thing is legal, the
party can ask for its return, even if no criminal prosecution has yet been filed, as
in the Stonehill case.

Stonehill v. Diokno, 20 SCRA 383 (1967)

F: Upon application of the officers of the govt (resp. prosecutors), several
judges (resp. judges) issued a total of 42 search warrants against petitioners &/
or the corporations of w/c they were officers, directed to any peace officer, to
search the perons named and/ or the premises of their offices, warehouses,
and/ or residences, and to seize several personal prop. as the "subject of the
offense; stolen or embezelled or the fruits of the offense," or "used or intended
to be used as the means of committing the offense" as violation of CB Laws,
Tariff and Customs Laws (TCC), NIRC and the RPC."
Alleging that the aforementioned search warrants are null & void, said
petitioners filed w/ the SC this orig. action for certiorari, prohibition, mandamus
& injunction. The writ was partially lifted or dissolved, insofar as the papers,
documents, and things seized from the officers of the corporations; but the
injunction was maintained as regards those found & seized in the residences of
petitioners.

ISSUES: (1) With respect to those found & seized in the offices of the
corporations, w/n petitioners have cause of action to assail the validity of the
contested warrants.
(2) In connection w/ those found & seized in the residences of
petitioners, w/n the search warrants in question and the searches and seizures
made under the authority thereof are valid.
(3) If the answer in no. 2 is no, w/n said documents, papers and things
may be used in evidence against petitioners.

HELD: (1) No. Petitioners have no cause of action to assail the legality of the
contested warrants and the seizure made in pursuance thereof bec. said
corporations have their respective personalities, separate and distinct from the
personality of petitioners. The legality of a seizure can be contested only by the
party whose rights have been impaired thereby and that the objection to an
unlawful search and seizure is purely personal and cannot be avalied of by 3rd
parties.

(2) No. Two points must be stressed in connection w/ Art. III, Sec. 2 of
the Consti: (a) that no warrant shall issue but upon probable cause to be
determined by the judge in the manner set forth therein; & (b) that the warrant
shall particularly describe the things to be seized.
None of these requirements has been complied w/. It was stated that the
natural and juridical persons has committed a violation of CB laws, TCC, NIRC &
RPC. No specific offense had been alleged in said applications. The averments
thereof w/ respect to the offense committed were abstract. As a consequence,
it was impossible for the judges who issued the warrants to have found the
existence of a probable cause, for the same presupposes the introduction of
competent proof that the party against whom it is sought has performed
particular acts, or committed specific omissions, violating a given provision of
our criminal laws.
General search warrants are outlawed bec. they place the sanctity of the
domicile and the privacy of communication and correspondence at the mercy of
the whims, caprice or passion of peace officers.
The warrants sanctioned the seizure of all records of the petitioners and
the aforementioned corporations, whatever their nature, thus openly
contravening the explicit command of our Bill of Rights-- that the things to be
seized be particularly described-- as well as tending to defeat its major
objective: the elimination of general warrants. RAM.

CONSTI TUTI ONAL LAW 2 ZPG & ASSOCI ATES

7. Civil Action for Damages

A civil case for damages can also be filed pursuant to Article 32 of the Civil
Code.

In Aberca v. Ver, the SC held that even if the privilege of the writ is
suspended, the court can nevertheless entertain an action not only against the
task force but even against the top ranking officials who ordered the seizure, to
recover damages for the illegal searches and seizures made in a despotic
manner. By so doing, one can indirectly inquire into the validity of the
suspension of the privilege.


8. Search and Seizure by Private Persons

People v. Marti, 193 SCRA 57 (1991)

F: Before delivery of appellant's box to the Bureau of Customs and/ or
Bureau of Posts, Mr. Job Reyes (proprietor) & husband of Anita Reyes, following
standard operating procedure, opened the boxes for final inspection. When he
opened appellant's box, a peculiar order emitted therefrom. His curiosity
aroused, he squeezed one of the bundles allegedly containing gloves and felt
dried leaves inside. Opening one of the bundles, he pulled out a cellophane
wrapper protruding from the opening of one of the gloves. He made an opening
on one of the cellophane wrappers and took several grams of the contents
thereof.
Job Reyes reported the incident to the NBI and requested a laboratory
examination of the samples he extracted from the cellophane wrapper.
It turned out that the dried leaves were marijuana flowering tops as
certified by the forensic chemist of the Narcotics Section of the NBI.
Thereafter, an information was filed against appellant for violation of
RA 6425.

APPELANT CONTENDS that the evidence subject of the imputed offense had
been obtained in violation of his consti. rights against unreasonable searches
and seizures and privacy of communication and therefore argues that the same
should be held inadmissible in evidence.

The case at bar assumes a peculiar character since the evidence sought
to be excluded was primarily discovered and obtained by a private person,
acting in a private capacity and w/o the intervention and participation of state
authorities.

ISSUE: May an act of a private individual, allegedly in violation of appellant's
constitutional rights, be invoked against the state?

HELD: We hold in the negative. In the absence of governmental interference,
the liberties guaranteed by the Consti. cannot be invoked against the State. This
constitutional right refers to the immunity of one's person, whether citizen or
alien, from interference by govt. xxx (Villanueva v. Querubin.)
The contraband in the case at bar having come into possession of the govt
w/o the latter transgressing appellant's rights against unreasonable searches
and seizures (S & S), the Court sees no cogent reason why the same should not
be admitted against him.
Appellant, however, would like this Court to believe that NBI agents made
an illegal search and seizure of the evidence later on used in prosecuting the
case. The arguments of appellant stands to fall on its own weight, or the lack of
it.
First, the factual considerations of the case at bar readily foreclose the
proposition that NBI agents conducted an illegal S & S of the prohibited
merchandise. Records of the case clearly indicate that it was Mr. Job Reyes, the
proprietor of the forwarding agency, who made the search/ inspection. Such
inspection was reasonable and a SOP on the part of Mr. Reyes as a
precautionary measure bef. delivery of packages to the Bureau of Customs or
Bureau of Posts.
Second, the mere presence of the NBI agents did not convert the
reasonable search effected by Reyes into a warrantless S & S proscribed by the
Consti. Merely to observe and look at that w/c is plain sight is not search.
Having observed that w/c is open, where no trespass has been committed in aid
thereof, is not search.
That the Bill of Rights embodied in the Consti. is not meant to be invoked
against acts of private individuals finds support in the deliberations of the Con
Com.: " xxx The Bill of Rights governs the relationship between the individual
and the state. Its concern is not the relation between individuals, between a
private individual and other individuals. xxx" (Sponsorship speech of
Commissioner Bernas.)
The constitutional proscription against unlawful S & S therefore applies as
a restraint directed only against the govt and its agencies tasked w/ the
enforcement of the law. Thus, it could only be invoked against the State to
whom the restraint against arbitrary and unreasonable exercise of power is
imposed.
It the search is made at the behest or inititiation of the proprietor of a
private establishment for its own and private purposes, as in the case at bar, and
w/o the intervention of police authorities, the right against unreasonable S & S
cannot be invoked for only the act of private individuals, not law enforcers, is
involved. In sum, the protection against unreasonable S & S cannot be extended
to acts committed by private individuals so as to bring it w/in the ambit of
alleged unlawful intrusion by the govt.

9. In the issuance of warrants of ARREST, as distinguished from SEARCH
warrants, the judge may rely simply on fiscal's certification as to probable
cause

Compare Rule 112, Sec. 6 (on warrants of arrest) with Rule 126, Sec. 4 (on search
warrants.)

Rule 112, Sec. 6. When warrant of arrest may issue.-- (a) By the
Regional Trial Court.-- Upon the filing of an information, the Regional Trial
Court may issue a warrant for the arrest of the accused.
(b) By the Municipal Trial Court.-- If the municipal trial judge
conducting the preliminary investigation is satisfied after an examination in
writing and under oath of the complainant and his witnesses in the form of
searching questions and answers, that a probable cause exists and that there is
a necessity of placing the respondent under immediate custody in order not to
frustrate the ends of justice, he shall issue a warrant of arrest. (Rules of
Court.)


Rule 126, Sec. 4. Examination of complainant; record.-- The judge must,
before issuing the warrant, personally examine in the form of searching
questions and answers, in writing and under oath the complainant and any
witnesses he may produce on facts personally known to them and attach to
the records their sworn statements together with any affidavits submitted.
The requirement in the case of warrants of arrest is relaxed in that the judge
can rely on the certification of the fiscal that the latter has conducted the
preliminary investigation and has found probable cause on the part of the
accused. The judge can issue the warrant on the basis of the information filed
by the fiscal and the certification of probable cause.

The SC has allowed this practice in Amarga v. Abbas, 98 Phil. 739 (1956),
noting that it has been practice long settled and that a judge can issue an order
to arrest on the basis of the certificate.

Of course, if the judge is in doubt, he can always ask the fiscal to submit
the records of the preliminary investigation, so he could determine for himself if,
on the basis of the affidavits, there exists probable cause. It he is satisfied with
the affidavits, he need not summon the affiants.

Amarga v. Abbas, 98 Phil. 739 (1956)

F: Municipal Judge Samulde conducted a preliminary investigation (PI) of
Arangale upon a complaint for robbery filed by complainant Magbanua, alleging
that Arangale harvested palay from a portion of her land directly adjoining
Arangales land. After the PI, Samulde transmitted the records of the case to
Provincial Fiscal Salvani with his finding that there is prima facie evidence of
robbery as charged in the complaint. Fiscal Salvani returned the records to
Judge Samulde on the ground that the transmittal of the records was
premature because Judge Samulde failed to include the warrant of arrest (WA)
against the accused. Judge Samulde sent the records back to Fiscal Salvani
stating that although he found that a probable cause existed, he did not believe
that Arangale should be arrested.
Fiscal Salvani filed a mandamus case against Judge Samulde to compel
him to issue a WA. RTC dismissed the petition on the ground that the fiscal had
not shown that he has a clear, legal right to the performance of the act to be
required of the judge and that the latter had an imperative duty to perform it.
Neverhteless, Judge Samulde was ordered to issue a WA in accordance with Sec.
5, Rule 112 of the 1985 Rules of Court.

ISSUE: Whether it is mandatory for the investigating judge to issue a WA of the
accused in view of his finding, after conducting a PI, that there exists prima facie
evidence that the accused commited the crime charged.

HELD: THE PURPOSE OF A PRELIMINARY INVESTIGATION DOES NOT
CONTEMPLATE THE ISSUANCE OF A WA BY THE INVESTIGATING JUDGE OR
OFFICER.

Under Rule 112 of the 1985 ROC, a PI is conducted on the basis of
affidavits to determine whether or not there is sufficient ground to hold the
accused for trial. To determine whether a WA should issue, the investigating
judge must have examined in writing and under oath the complainant and his
wirtnesses by searching questions and answers; he must be satisfied that a
probable cause exists; and there must be a need to place the accused under
immediate custody in order not to frustrate the ends of justice. It is not
CONSTI TUTI ONAL LAW 2 ZPG & ASSOCI ATES

obligatory, but merely discretionary, upon the investigating judge to issue a WA,
for the determination of whether it is necessary to arrest the accused in order
not to frustrate the ends of justice, is left to his sound judgment or discretion.
The fiscal should, instead, have filed an information immediately so that
the RTC may issue a warrant for the arrest of the accused. Bam.


Beltran v. Makasiar, 167 SCRA 393 (1988)

F: The Pres. of the Phils. filed a complaint for libel against the petitioners,
who were publisher and columnist of the Philippine Star, based on the following
statement in Beltran's column of Oct. 12, 1987 entitled "The Nervous Officials of
the Aquino Administration:" "If you will recall, during the Aug. 29 coup attempt,
the Pres. hid under her bed, while the firing was going on-- perhaps the first
Commander-in-Chief of the AFP to have to do so."
Instead of submitting his counter-affidavit, Beltran moved to dismiss
the complaint. The fiscal deniend his motion after finding a prima facie case
against the petitioners and filed the case in court w/c thereafter issued warrants
of arrest against the petitioners. The petitioners filed a petition for certiorari
and prohibition.

HELD: The addition of the word "personally" after the word "determined" (Art.
III, Sec. 2) and the deletion of the grant of authority by the 1973 Consti. to issue
warrants to "other responsible officer as may be authorized by law," has
apparently convinced petitioner Beltran that the Consti. now requires the judge
to personally examine the complainant and his witnesses in his determination of
probable cause for the issuance of warrants of arrest. This is not an accurate
interpretation. What the Consti. underscores is the exclusive and personal
responsibility of the issuing judge to satisfy himself of the existence of probable
cause. In satisfying himself of the existence of probable cause for the issuance
of a warrant of arrest, the judge is not required to personally examine the
complainant and his witnesses. Following established doctrine and procedure,
he shall: (1) personally evaluate the report and the supporting documents
submitted by the fiscal regarding the existence of probable cause and, on the
basis thereof, issue a warrant of arrest; or (2) if on the basis thereof he finds no
probable cause, he may disregard the fiscal's report and require the submission
of supporting affidavits of witnesses to aid him in arriving at a conclusion as to
the existence of probable cause. Sound policy dictates this procedure,
otherwise judges would be unduly laden w/ the preliminary examination and
investigation of criminal complaints instead of concentrating on hearing and
deciding cases filed before their courts.
xxx

10. When arrest may be made without a warrant

Rule 113, Sec. 5. Arrest without warrant; when lawful.-- A peace officer
or a private person may, without a warrant, arrest a person:
(a) When, in his presence, the person to be arrested has committed, is
actually committing, or is attempting to commit an offense;
(b) When an offense, has in fact just been committed, and he has
personal knowledge of facts indicating that the person to be arrested has
committed it;
(c) When the person to be arrested is a prisoner who has escaped from
a penal establishment of place where he is serving final judgment or
temporarily confined while his case is pending, or has escaped while being
transferred from one confinement to another.
In cases falling under paragraphs (a) and (b) hereof, the person arrested
without a warrant shall be forthwith delivered to the nearest police station or
jail, and he shall be proceeded against in accordance with Rule 112, Section. 7.
(Rules of Court.)


Rule 113, sec. 5 talks of "citizen arrests", cases where an arrest can be
made either by the peace officer or a private person without need of a warrant.
The key element in the first case is that the offense was committed "in his
presence". The key element in the second case is that he has "personal
knowledge".

Thus, in People v. Burgos, 144 SCRA 1 (1986), the arrest made by the
constabulary without a warrant of a farmer on the basis of information that he
was a subversive was held unconstitutional, since there was no personal
knowledge of the offense itself.

The gun and subversive documents found by the officer and admitted by
the former to be his were likewise held inadmissible because the admission
violated the Miranda rule.

a. Strict enforcement of rule

People v. Burgos, 144 SCRA 1 (1986)

F: On the basis of info. given by Cesar Masamlok, the appellant was arrested
while plowing his farm in Tiguman, Davao del Sur, on May 13, 1982, on charges
of illegal possession of firearm in furtherance of subversion. A .38 caliber
revolver was found buried under his house. Subversive documents were also
seized from a place near his house. Two arresting officers testified that the
appellant had readily admitted ownership of the gun and the documents. The
appellant was found guilty of the charge and sentenced to 20 years of reclusion
temporal, as minimum, to reclusion perpetua, as maximum, and the gun and
documents were ordered confiscated.

HELD: (1) Under R 113, Sec. 5 (a), the arresting officer must have personal
knowledge that the crime has been committed, is being committed, or is about
to be committed, in order to justify an arrest w/o a warrant. The offense must
also be committed in his presence or w/in his view. There is no such personal
knowledge in this case. Hence the arrest of the appellant was illegal.
(2) Consequently, the incidental search and seizure were likewise illegal
and the firearm and document are inadmissible in evidence.
(3) The prosecution argues that the appellant admitted ownership of the
gun and claims that it was he who pointed to the place where the subversive
documents were hidden. However, as the appellant was not informed of his
constitutional rights at that time, his admission is inadmissible under [Art. III,
Sec. 12 (1).] It is true that 6 days later he executed a confession before the fiscal
w/ the assistance of counsel, but it was then already too late.
(4) As the remaining evidence against the appellant is the testimony of
Cesar M. and it is uncorroborated and unreliable, the appellant should be
acquitted, but the gun and the subversive documents must be confiscated.

People v. Rodriguez, 232 SCRA 498 (April 25, 1989)

F: Pat. Marvin Pajilan received a phone call from the desk officer of Sub-
Station I, namely, Michael Orbeta, who informed him that a person named 'Alyas
Allan' was selling marijuana at No. 8199 Constancia St., Makati, Metro Manila
and requested that said person be apprehended. Acting on this phone call of
desk officer Michael Orbeta, a team of policemen posted themselves about 10
to 15 meters from the house located at 8199 Constancia St., Makati. They saw a
tricycle with 3 persons on board, a driver and 2 passengers, stop in front of the
house at 8199 Constancia St. They also saw a male person come out of the said
house and approach and talk to the driver of the tricycle. After a while they saw
the male person go back to the house and a little later come back and hand to
the tricycle driver 'a suspicious stuff of a cigarette, a marijuana cigarette', they
further saw the tricycle driver in turn give something to the male person. Pat.
Pajilan together with his companions approached the male person and the
tricycle driver and after introducing themselves as police officers, they asked the
male person, the tricycle driver and his 2 passengers to bring out the contents of
their pockets, which the male person, the driver and the passengers of the
tricycle did. The male person brought out from his pockets 2 small plastic bags
containing suspected marijuana leaves. The tricycle driver brought out from his
right front pocket 3 sticks of suspected marijuana cigarettes. Nothing illegal was
found in the pockets of the 2 passengers of the tricycle.
The appellant contends that the police officers had no personal
knowledge that he was indeed handing marijuana to Enrico Bacod as they were
10-15 meters away from the alleged sale transaction. The arrest therefore was
not valid as the requirements for a warrantless arrest were not complied with.

Issue: Was the warantless arrest valid?

Ruling: YES.
The warrantless arrest made by the law enforcers was valid since it falls
under the provisions of Rule 113, Sec. 5(a) of the Rules of Court which provides:
Sec. 5. Arrest without warrant; when lawful. A peace officer or a private
person may, without a warrant, arrest a person:
(a) When, in his presence, the person to be arrested has committed, is
actually committing, or is attempting to commit an offense;

Having caught the appellant in flagrante as a result of the buy-bust
operation, the policemen were not only authorized but were also under
obligation to apprehend the drug pusher even without a warrant of arrest. The
police officers were tipped off by an informer about the illegal trade of the
accused. The exact location where this trading in drugs was taking place was
given to them. The 'suspicious stuff' taken from the accused were confirmed to
be marijuana after tests were conducted on them. The attendant circumstances
taking place before their eyes led the police officers to reasonably conclude that
an offense was actually being committed.


b. Exceptions to strict enforcement

(1) "Continuous" crimes of subversion

Umil v. Ramos, 187 SCRA 311 (1990)

These are 8 petitions for habeas corpus (HC) filed bef. the Court. The
Court finds that the persons detained have not been illegally arrested nor
arbitrarily deprived of their constitutional right to liberty and that the
circumstances attending these cases do not warrant their release on HC.
An arrest w/o a warrant, under Sec. 5, pars. (a) and (b) of Rule 113, ROC,
as amended is justified when the person arrested is caught in flagrante delicto,
viz., in the act of committing an offense; or when an offense has just been
CONSTI TUTI ONAL LAW 2 ZPG & ASSOCI ATES

committed and the person making the arrest has personal knowledge of the
facts indicating that the person arrested has committed it.
The persons in whose behalf these petitions for HC have been filed had
freshly committed or were actually committing an offense, when apprehended,
so that their arrests, w/o warrant were clearly justified, and that they are,
further detained by virtue of valid informations filed against them in court.

I

In Umil v. Ramos, RIOU-CAPCOM received confidential info. about a
member of the NPA-Sparrow unit being treated for a gunshot wound at the St.
Agnes Hospital in Roosevelt Ave., Q.C. It was found that the wounded person,
who was listed in the hospital records as Ronnie Javelon, is actually Rolando
Dural, a member of the NPA liquidation squad, responsible for the killing of 2
CAPCOM soldiers the day before. Dural was then transferred to the Regional
Medical Services of the CAPCOM.
Upon positive identification by an eyewitness, Dural was referred to the
Caloocan City Fiscal who conducted an inquest and thereafter filed w/ the RTC-
Caloocan City an info. charging Dural w/ the crime of "Double Murder w/ Assault
upon agents of persons in authority."
The petition for HC, insofar as Umil & Villanueva are concerned, is now
moot and academic and is accordingly dismissed, since the writ does not lie in
favor of an accused in a crim. case, who has been released on bail.
As to Dural, he was not arrested while in the act of shooting the 2
soldiers. Nor was he arrested after the commission of said offense for his arrest
came a day after the shooting incident. However, Dural was arrested for being a
member of the NPA, an outlawed subversive organization. Subversion being a
continuing offense, the arrest of Dural w/o warrant is justified as it can be said
that he was committing an offense when arrested.
The arrest of persons involved in rebellion whether as its fighting armed
elements, or for committing non-violent acts but in furtherance of rebellion, is
more an act of capturing them in the course of an armed conflict, to quell the
rebellion, than for the purpose of immediately prosecuting them in court for a
statutory offense. The arrest, therefore, need not follow the usual procedure in
the prosecution of offenses w/c requires the determination by a judge of the
existence of probable cause bef. the issuance of a judicial warrant and the
granting of bail if the offense is bailable. Obviously, the absence of a judicial
warrant is no legal impediment to arresting or capturing persons committing
overt acts of violence against govt forces, or any other milder acts but equally in
pursuance of the rebellious movement. xxx (Garcia-Padilla v. Enrile.)
Dural was found guilty of the charge and is now serving the sentence
imposed upon him by the trial court. Thus, the writ of HC is no longer available

II

The arrest of Amelia Roque and Wilfredo Buenaobra, w/o warrant is also
justified.

In view of the revelations made by Rogelio Ramos, a former NPA, the
Constantino house in Marikina Heights was placed under military surveillance
and on 8/12/88, pursuant to a search warrant , a search of the house was
conducted at 5 PM by CISC-NCD & CSG. In the course of the search were found
several firearms, regular power supply, antennae, speaker and subversive
documents.
When confronted, R. Constantino (RC) could not produce any permit or
authority to possess the firearms, ammunition, radio, etc. He was brought to CIS
HQ for investigation. When questioned, he refused to give a written statement,
although he admitted that he was a staff member of the executive of the NUFC
and a ranking member of the International Dept. of the CPP.
At about 8 PM, same day, Buenaobra arrived at RC's house. When
arrested, he readily submitted to the military agents that he is a regular member
of the CPP/ NPA and that he went to the place to deliver letters to "Ka Mong,"
referring to RC and other members of the rebel group. Also found in
Buenaobra's possession was a piece of paper containing the jumbled tel. no. of
Florida Roque, sister of Amelia Roque, aka. "Ka Nelia." They went to the address
on 8/13/88 and arrived at the place about 11 AM. After identifying themselves
as military agents and after seeking permission to search the place, w/c was
granted, the military agents conducted a search in the presence of the
occupants of the house and the barangay captain of the place.
The military found the place to be another safehouse of the NUFC/ CPP.
They found firearms, subversive documents, ledgers, journals, vouchers, among
others. Amelia admitted ownership of the documents seized.
Roque was brought to the Caloocan City Fiscal for inquest after w/c an
info. charging her w/ viol. of PD 1866 was filed. Another info. for viol. of the
Anti-Subversion Act was filed against Roque and also to Buenaobra.
A petition for HC was filed bef. this Court on behalf of Roque and
Buenaobra. At the hearing, Buenaobra manifested his desire to stay in the PC-
INP stockade at Camp Crame, Q.C. Accordingly, the petition for HC on his behalf
is now moot and academic.

III

Anonuevo v. Ramos.

The arrest of Domingo Anonuevo (A) and Ramon Casiple (C) w/o warrant
is justified.
At about 7:30 PM on 8/13/88, A and C arrived at the house of RC w/c was
still under surveillance. The military noticed bulging objects on their waist lines.
When frisked, the agents found them to be loaded guns. They were asked to
show their permit or license to possess or carry firearms and ammunitions but
they could not produce any. Hence, they were brought to PC HQ for
investigation.
At the PC stockade, A was identified as "Ka Ted," and C as "Ka Totoy" of
the CPP by their former comrades.
On 8/15/88, an info. charging them w/ viol. of PD 1866 was filed bef. RTC-
Pasig. On 8/24/88, a petition for HC was filed bef. this Court.

HELD: The petitioner's claim that they were unlawfully arrested bec. there was
no previous warrant, is w/o merit. The records show that they were carrying
unlicensed firearms and ammunitions in their person when apprehended.
There is also no merit in the contention that the info. filed against them
are null and void for want of prel. inv. The filing of an info., w/o a prel. inv.,
having been first conducted, is sanctioned by Rule 112, Sec. 7, ROC.
Petitioners refused to sign a waiver of the provisions of Art. 125, RPC. Nor
did petitioners ask for prel. inv. after the informations had been filed against
them in court.

IV

Ocaya v. Aguirre.

On 5/12/88, agents of the PC Intelligence and Investigation Division of
Rizal PC-INP Command, armed w/ a search warrant, conducted a search of a
house located at Marikina Green Heights, believed to be occupied by Benito
Tiamson, head of the CPP-NPA. In the course of the search, Ocaya arrived in a
car driven by Danny Rivera. Subversive documents and several rounds of
ammunitions for a .45 cal. pistol were found in Vicky Ocaya's car. They were
brought to the PC HQ for investigation, when O. could not produce any permit or
authorization to possess the ammunition, an info. charging her w/ viol. of PD
1866 was filed w/ RTC-Pasig. Rivera was released from custody.
On 5/17/88, a petition for HC was filed on behalf of these 2.

HELD: Vicky O. was arrested in flagrante delicto so that her arrest w/o warrant is
justified. No. prel. inv. was conducted bec. she was arrested w/o a warrant and
she refused to waive the provisions of Art. 125 of the RPC, pursuant to R112,
Sec. 7, ROC.

V

The petitioners Ocaya, Anonuevo, Casiple and Roque claim that the
firearms, ammunitions and subversive documents alleged to have been found in
their possession, when arrested, did not belong to them, but were planted by
the military to justify their illegal arrest.
The petitioners, however, have not introduced any evidence to support
their claim. On the other hand, no evil motive or ill will on the part of the
arresting officers that could cause the said officers in these cases to accuse the
petitioners falsely, has been shown.
As pointed out by the Sol-Gen, the arrest of the petitioners is not a
product of a witch hunt or a fishing expedition, but the result of an in-depth
surveillance of NPA safehouses pointed no less than by former comrades of the
petitioners.
VI

Espiritu v. Lim.

Deogracias Espititu is the Gen. Sec. of PISTON. Petitioner claims that at
about 5 AM of 11/23/88, while he was sleeping in his home located at Sta. Mesa,
Mla., he was awakened by his sister who told him that a group of persons
wanted to hire his jeepney. When he went down to talk to them, he was
immediately put under arrest. When he asked for the warrant, the men bodily
lifted him and placed him in their owner type jeepney. He demanded that his
sister be allowed to accompany him, but the men did not accede to his request.
An info. charging him w/ viol. of Art. 142, RPC (Inciting to sedition) was
filed against him.
In the afternoon of 11/22/88, during a press-con at the NPC "Deogracias
E. through tri-media was heard urging all drivers and operators to go on
nationwide strike on 11/23/88 xxx."
Policemen waited for petitioners outside the NPC in order to investigate
him, but he gave the lawmen his slip. He was next seen at about 5 PM at a
gathering of drivers and sympathizers, where he was heard as saying,
"Bukas tuloy and welga natin ... hanggang sa magkagulo na."
Since the arrest of the petitioner w/o warrant was in accordance w/ the
provisions of R 113, Sec. 5 (b), ROC, and the petitioner is detained by virtue of a
valid info. filed w/ the competent court, he may not be released on HC.

VII

Nazareno v. Station Commander.

CONSTI TUTI ONAL LAW 2 ZPG & ASSOCI ATES

At about 8:30 AM of 12/14/88, one Romulo Bunye II was killed by a group
of men in Alabang, Muntinglupa, MM. One of the suspects in the killing was
Ramil Regala who was arrested by the police on 12/28/88. Upon questioning,
Regala pointed to Nazareno as one of his companions in the killing of Bunye II.
In view thereof, the officers, w/o warrant, picked up Nazareno and brought him
to the police HQ for questioning.
xxx
On 2/1/89, the presiding judge of the RTC-Binan, Laguna, issued a
resolution denying the petition for HC, it appearing that said Narciso Nazareno is
in the custody of the respondents by reason of an info. filed against him w/ the
RTC-Mkti., MM.

HELD: The arrest of Nazareno was effected by the police w/o warrant pursuant
to Sec. 5 (b), R 113, ROC, after he was positively implicated by his co-accused;
and after investigation by the police.
The obligation of an agent of authority to make an arrest by reason of a
crime, does not presuppose as a necessary requisite for the fulfillment thereof,
the indubitable existence of a crime. For the detention to be perfectly legal, it is
sufficient that the agent or person in authority making the arrest has reasonably
sufficient grounds to believe the existence of an act having the characteristic of a
crime and that the same grounds exist to beleive that the person sought to be
detained participated therein." (Peo. v. Ancheta.)




VIII

In all the petitions here considered, criminal charges have been filed in
the proper courts against the petitioners. The rule is that if a person alleged to
be restrained of his liberty is in the custody of an officer under process issued by
a court or judge, and that the court or judge had jurisdiction to issue the process
or make the order, or if such person is charged before any court, the writ of HC
will not be allowed. (Sec. 4, R 102, ROC.)

On the Ilagan Doctrine.

As the Court sees it, re-examination or re-appraisal, w/ a view to its
abandonment, of the Ilagan case doctrine is not the answer. The answer and
the better practice would be, not to limit the function of HC to a mere inquiry as
to w/n the court w/c issued the process, judgement or order of commitment, or
bef. whom the detained person is charged, had jurisdiction or not to issue the
process, judgment or order or to take cognizance of the case, but rahter, as the
court itself stated in Morales, Jr. v. Enrile, in all petitions for HC, the court must
inquire into every phase and aspect of petitioner's detention-- "from the
moment petitioner was taken into custody up to the moment the court passes
upon the merits of the petition;" and "only after such a scrutiny can the court
satisfy itself that the due process clause of our Constitution in fact has been
satisfied."


Umil v. Ramos, 202 SCRA 251

PETITION SEEKING SEPARATE MOTIONS FOR RECONSIDERATION FROM THE
COURT'S DECISION PROMULGATED ON 9 JULY 1990

The decision (on July 9, 1990) did not rule that mere suspicion that one is
a CPP or NPA is a valid ground for his arrest w/o warrant.

We find no merit in the motions for reconsideration.

Rolando Dural.-- His arrest w/o warrant is justified as it can be said that,
w/in the contemplation of Sec. 5 (a), R 113, ROC, he was committing an offense,
when arrested, bec. Dural was arrested for being a member of the NPA, an
outlawed org., where membership is penalized, and for subversion w/c, like
rebellion is, under Garcia v. Padilla, a continuing crime.
Dural did not cease to be, or become less of a subversive, FOR PURPOSES
OF ARREST, simply bec. he was, at the time of arrest, confined in the St. Agnes
Hospital. Dural was identified as one of several persons who, the day before his
arrest, w/o warrant, had shot 2 CAPCOM policemen in their patrol car. Dural,
given another opportunity, would have shot or would shoot other policemen,
anywhere as agents or representative of an organized govt. It is in this sense
that subversion and rebellion are anchored on an ideological base w/c compels
the repetition of the same acts of lawlessness and violence until the overriding
objective of overthrowing organized govt is attained.
His arrest was based on "probable cause."
Sec. 5, R 113, ROC, requires 2 conditions for a valid arrest w/o warrant:
(1) the person to be arrested has just committed an offense and (2) the person
arresting has personal knowledge of facts indicating that the person to be
arrested is the one who committed the offense.
It has been ruled that personal knowledge of facts in arrests w/o warrant
must be based upon probable cause, w/c means on actual belief or reasonable
grounds of suspicion.
The grounds of suspicion are reasonable when, in the absence of actual
belief of the arresting officers, the suspicion that the person to be arrested is
probably guilty of committing the offense, is based on actual facts, i.e.,
supported by circumstances sufficiently strong in themselves to create the
probable cause of guilt of the person to be arrested. A reasonable suspicion
therefore must be founded on probable cause, coupled w/ good faith on the part
of the peace officers making the arrest.
Said confidential info. received by the arresting officers, to the effect that
an NPA was being treated for a gunshot wound was based on actual facts and
supported by circumstances sufficiently to engender a belief that an NPA
member was truly in said hospital. The actual facts supported by circumstances
are: (1) the day bef., or on 1/31/88, 2 CAPCOM soldiers were actually killed in
Bagong Bo., Caloocan City by 5 "sparrows" including Dural; (2) a wounded
person listed in the hospital records as "Ronnie Javelon" was actually then being
treated in said hospital for for a gunshot wound; (3) "Ronnie Javelon" and his
address entered in the hospital records were fictitious and the wounded man
was in reality Dural.

On good faith.-- The peace officers who arrested Dural are deemed to
have conducted the same in good faith, considering that law enforcers are
presumed to regularly perform their official duties.
A few days after Dural's arrest, an info. charging him w/ Double murder
w/ assault against agents of persons in authority was filed in RTC-Caloocan City.
He was placed under judicial custody. On 8/31/88, he was convicted and
sentenced to reclusion perpetua.

As to A. Roque., W. Buenaobra, D. Anonuevo, R.. Casiple & V. Ocaya, their
arrests, w/o warrant, are also justified. They were searched pursuant to a
warrant issued by a court of law and were found w/ unlicensed firearms,
explosives and/ or ammunitions on their persons. They were, therefore, caught
in flagrante delicto w/c justified their outright arrest w/o warrant under Sec. 5
(a), R113, ROC. A few days after their arrests, informations were filed in court
against said petitioners placing them w/in judicial custody and disposition.
Buenaobra's petition is moot bec. he had chosen to remain in detention.
The reason which compelled the military agents to make the arrests w/o
warrant was the info. given to the military that 2 safehouses (one occupied by
RC and the other by Benito Tiamson) were being used by the CPP/ NPA for their
operations, w/ info. as to their exact location and the names of RC and BT as
residents and occupants thereof.
And at the time of the actual arrests, the following circumstances
surrounded said arrests (of Roque, Buenaobra, Anonuevo and Casiple), w/c
confirmed the belief of the military that the info. they had received was true and
the persons to be arrested were probably guilty of the commission of certain
crimes: first, the search warrant was duly issued to effect the search of the
Constantino safehouse; second, found in the safehouse was a person named RC,
who admitted that he was a ranking member of the CPP, and found in his
possession were unlicensed firearms and communist equipment; third, at the
time of their arrests, in their possession were unlicensed firearms, ammunitions,
and/ or subversive documents, and they admitted ownership thereof as well as
their membership in the CPP/ NPA. And then shortyly after their arrests, they
were positively identified by their former comrades as CPP/ NPA members.
An arrest is in the nature of an administrative measure. The power to
arrest w/o warrant is w/o limitation as long as the requirements of Sec. 5, R 113
are met. This rule is founded on an overwhelming public interest in peace and
order in our community.
"xxx The legality of the detention does not depend upon the fact of the
crime, but xxx upon the nature of the deed, wherefrom such characterization
may reasonably be inferred by the officer or functionary to whom the law at the
moment leaves the decision for the urgent purpose of suspending the liberty of
the citizen." (US v. Sanchez.)

ESPIRITU was arrested w/o warrant, not for subversive or any "continuing
offense," but for uttering the words "Bukas tuloy ang welga natin xxx hanggang
sa magkagulo na" w/c in the perception of the arresting officers, was inciting to
sedition.

Many persons differ as to the validity of such perception and regard the
language as falling w/in free speech guaranteed by the Consti. But, the authority
of the peace officers to make the arrest, w/o warrant, at the time the words
were uttered, or soon thereafter, is still another thing. In the balancing of
authority and freedom, w/o obviously becomes difficult at times, the court, has
in this case, tilted the scale in favor of authority but only for purposes of the
arrest (not conviction.)

Supervening events made this case moot and academic. for E. had bef.
arraignment asked the court a quo for re-investigation, the peace officers did
not appear. Case against E. has been provisionally dismissed and his bail
cancelled.

NAZARENO'S ARREST.-- Although the killing of Bunye II occured on
12/14/88, while Nazareno's arrest w/o warrant was made only on 12/28/88 or
14 days later, teh arrest falls under Sec. 5 (b), R113, since it was only on
12/28/88 that the police authorities came to know that Nazareno was probably
one of those guilty in the killing of Bunye II and the arrest had to be made
promptly, even w/o a warrant (after the police were alerted) and despite the
lapse of 14 days to prevent possible flight.
CONSTI TUTI ONAL LAW 2 ZPG & ASSOCI ATES

Nazareno has since been convicted by the court a quo for murder and
sentenced to reclusion perpetua.

ADMISSIBLITY OF EXTRA-JUDICIAL ADMISSION.

These admissions strengthen the Court's perception that truly the
grounds upon w/c the arresting officers based their arrests w/o warrant, are
supported by probable cause, i.e., that the persons arrested were probably
guilty of the commission of certain offenses, in compliance w/ Sec. 5, R 113,
ROC. To note these admissions, on the other hand, is not to rule that the
persons arrested are already guilty of the offenses upon w/c their warrantless
arrests were predicated. The task of determining the guilt or innocence of
persons arrested w/o warrant is not proper in a petition for HC. It pertains to
the trial of the case on the merits.


(2) Illegal Possession of guns or drugs


People v. Linsangan, 195 SCRA 784

F: Accused Linsangan was arrested after a buy-bust operation. The two
marked ten-peso bill were retrieved from him. He was asked to sign his name
on the two marked bills. The ten handrolled cigarette sticks confiscated from
the accused were submitted for examination. After finding these positive for
marijuana, a case was filed for violation of the Dangerous Drugs Law. Linsangan
denied the charge. The trial court found Linsangan guilty. Upon appeal, one of
the assertions of Linsangan was that the trial court erred in not holding that
when the policemen required him to initial the marked bills, they violated his
constitutional right to counsel, to remain silent, and not to incriminate himself
while under custodial investigation.

ISSUE: WHETHER OR NOT THERE WAS A VIOLATION OF THE ACCUSEDS
CONSTITUTIONAL RIGHTS WHEN HE WAS MADE TO SIGN THE MARKED BILLS.

HELD: Although the accused was not assisted by counsel when he initialed the
P10-bills that the police found tucked in his waist, his right against self-
incrimination was not violated for his possession of the marked bills did not
constitute a crime; the subject of the prosecution was his act of selling
marijuana cigarettes. His conviction was not based on the presence of his initials
on the marked bills, but on the fact that the trial court believed the testimony of
the policemen that they arrested him while he was actually engaged in the
selling marijuana cigarettes to a member of the arresting party. The trial court
gave more credence to their categorical declarations than to the appellants
denials. That is as it should be for as law enforcers, they are presumed to have
performed their official duties in a regular manner. Their task of apprehending
persons engaged in the deadly drug trade is difficult enough without legal and
procedural technicalities to make it doubly so.


11. Immunity from arrest of members of Congress

Art. VI, Sec. 11. A Senator or Member of the House shall, in all offenses
punishable by not more than six (6) years imprisonment (prision correcional),
be privileged from arrest while Congress is in session. xxx


B. Rights of Persons under custodial interrogation

Art. III, Sec. 12. (1) Any person under custodial investigation for the
commission of an offense, shall have the right to be informed of his right to
remain silent and to have competent and independent counsel preferably of
his own choice. If the person cannot afford the services of counsel, he must be
provided with one. These rights cannot be waived except in writing and in the
presence of counsel.
(2) No torture, force, violence, threat, intimidation, or any other means
which vitiate the free will shall be used against him. Secret detention places,
solitary, incommunicado, or other similar forms of detention are prohibited.
(3) Any confession or admission obtained in violation of this or sec. 17
hereof, shall be inadmissible in evidence against him.
(4) The law shall provide for penal and civil sanctions for violations of
this section, as well as compensation to and rehabilitation of victims of torture
or similar practices, and their families.


Source: Miranda v. Arizona, 384 U.S. 436 (1966)

According to Chief Justice Warren, when a defendant is thrust into an
unfamiliar atmosphere and run through menacing police interrogation
procedures, where compulsion is forcefully potential and his will is likely to be
subjugated, the officers must undertake to afford proper safeguards by the
reading of the "Miranda rights" at the outset of the investigation to ensure that
the statements made are truly the product of free choice.

Any person under custodial or police investigation has the right to be
informed of the following rights:

1. Right to remain silent

a) To make him aware of it.
b) To overcome the inherent pressure of the interrogating atmosphere
c) To show the individual that his interrogators are prepared to recognize
his privilege should he choose to invoke his right.

2. Right to be reminded that if he waives his right to remain silent, anything he
says can and will be used against him.

a) To warn him of the consequences of waiving his right to remain silent.
b) To make him aware that this is an adversary system, and that the
police are not acting in his interest.

3. Right to counsel before and during the interrogation

a) To mitigate the dangers of untrustworthiness in his testimony, since
the inherent pressures initially overcome by the right to remain silent may again
run unless coupled with the right to counsel.
b) To lessen the possibility of coercion by the police.

4. Right to be reminded that if he cannot afford counsel, then one will be
provided for him by the state.

a) To inform him that if he does not have counsel or cannot afford one, he
does not have to defend himself alone.
b) To inform him that his poverty is no reason why he should lose his
right to counsel.

(The reading of these rights is no less indispensable even if the person
arrested is a prominent Constitutional lawyer. Although he may already know
these rights, the purpose is not so much to inform him, as to assure him that his
interrogators are willing to respect his rights amidst the pressure of custodial
investigation.)

The reading of these rights is required during "custodial investigation".

A police investigation consists of 2 stages:

1) "General exploratory investigation" - when the investigation consists
merely of general questions to find out who might be the culprit, but without
being directed at anyone's guilt in particular. At this stage, the Miranda rule is
not yet applicable; otherwise, people who could otherwise explain their
innocence would be arrested.

2) "Custodial investigation" - when the investigation now focuses on the
guilt of a person such that he is no longer allowed to leave the premise. It is at
this stage that the Miranda ruling is necessary, since the purpose of the
interrogation is to evince evidence that can be used to prosecute the person.

For instance, when A, a policeman, sees X running with a stained knife
away from an apparently dead man, he can rung after X and having grabbed
him, ask him for an explanation as to what he saw without reading his Miranda
rights. But once A arrests X and starts interrogating him in the police precinct,
then his rights must now be read, for there can only be one purpose to the
questioning, and that is to elicit evidence to be used to prosecute him.

Mendoza, The Right to Counsel During Custodial Investigations, 2 Law Rev. No.
10, 2 (1988); 61 Phil. LJ 409

I. RIGHT TO COUNSEL WAS DEVELOPED AS PART OF PROTECTION AGAINST
INVOLUNTARY CONFESSIONS.

Since the introduction of the American accusatorial system of criminal
procedure in the Phils., the rule has been that involuntary confessions are
inadmissible in evidence against the accused.
The question is on whom the burden of proof is placed. The early rule
placed the burden of proving that the confession was voluntary and, therefore,
admissible in evidence, on the prosecution. (Sec. 4, Act No. 619.) It was held
that a confession not shown to have been voluntarily given could be objected to
at any stage of the proceedings, even for the first time on appeal in the SC.
Act No. 619 was later repealed by the Admin. code of 1916, w/c placed
the burden of proof on the accused to show that his confession was involuntary.
Under the new rule, it was sufficient that the confession was given under
conditions w/c accredit prima facie its admissibility.
In 1953, a further change took place when the SC held in Peo. v. de los
Santos that "A confession, to be repudiated, must not only be proved to have
CONSTI TUTI ONAL LAW 2 ZPG & ASSOCI ATES

been obtained by force and violence, but also that it is false or untrue, for the
law rejects the confession when, by force or violence or intimidation, the
accused is compelled against his will to tell a falsehood, not even when such
force and violence he is compelled to tell the truth. In the later case of Peo. v.
Villanueva, the Court stated "the admissibility of that kind of evidence depends
not on the supposed illegal manner in w/c it is obtained but on the truth or
falsity of the facts or admission contained therein.
The illegality of the means used in obtaining evidence does not affect its
admissibility (Moncado v. People's Court.)

THE EFFECT OF THE EXCLUSIONARY RULE IN SEARCH AND SEIZURE CASES

The adoption in 1967 of the exclusionary rule in search and seizure cases
(Stonehill v. Diokno) worked a parallel in the law of confession. W/o expressly
overruling its decision in de los Santos and Villanueva, the Court, in Peo. v. Urro,
went back to the former rule that involuntary or coerced confessions, regardless
of their truth, are null and void. xxx Involuntary or coerced confessions
obtained by law, w/c proscribes the use of such cruel and inhuman methods to
secure confessions. xxx
Indeed, in the US, it is said that an "unconstitutional coercion will render
inadmissible even the most unquestionably true inculpatory statements." xxx
This is not bec. such confessions are unlikely to be true but bec. the methods
used to extract them offend an underlying principle in the enforcement of our
criminal law: that ours is an accusatorial and not an inquisitorial system -- a
system in w/c the State must establish guilt by evidence independently and
freely secured and not by coercion prove its charge against an accused out of his
own mouth xxx." (Rogers v. Richmond, J. Frankfurter.)

THE MIRANDA RULE

The prosecution may not use statements, whether exculpatory or
inculpatory, stemming from custodial interrogation of the def. unless it
demonstrates the use of procedural safeguards effective to secure the privilege
against self-incrimination. By custodial interrogation, we mean questioning
initiated by law enforcement officers after a person has been taken to custody or
otherwise deprived of his freedom of action in any significant way. xxx

II. IN TURN, MIRANDA WARNINGS WERE DEVISED AS MEANS OF SECURING THE
RIGHT TO COUNSEL.

Miranda v. Arizona requires certain warnings to be given by police
interrogators bef. a person in custody may be interrogated, w/c have been
adopted by the Phil. SC:
1. The person in custody must be informed in clear and unequivocal
terms that he has a right to remain silent. The purpose is to apprise him of his
privilege not to be compelled to incriminate himself, to overcome the inherent
pressures of the interrogation atmosphere, and to assure the individual that his
interrogators are prepared to recognize his privilege, should he choose to
exercise it.
2. The person in custody must be warned that anything he will say can
and wilol be used against him. This warning is intended to make him aware not
only of the privilege but also of the consequences of foregoing it.
3. Since the circumstances surrounding in-custody interrogation can
operate very quickly to overbear the will of one merely made aware of his
privilege by his interrogators, it is indispensable that he has the assistance of
counsel.


THE CUSTODIAL PHASE OF INTERROGATION

At what stage of the police interrogation must the warnings be given?
The Consti. does not state at what stage of the interrogation process they must
be made. but in Miranda, the court specified that it is only at the custodial
phase of the interrogation that its ruling applied. As the Court indicated in
Escobedo v. Illinois, it is only after the investigation ceases to be a general
inquiry into an unsolved crime and begins to focus on a particular suspect, the
suspect is taken into custody, and the police carries out a process of
interrogation that leads itself to eliciting incriminating statements that the rule
begins to operate.

In Gamboa v. Cruz, the accused was arrested, w/o a warrant, for
vagrancy. He was taken to police precint no. 2 in Mla. The next day, he was
included in a police line-up of 5 detainees and was pointed to by the
complainant as a complanion of the main suspect on the basis of w/c the
accused was ordered to stay and sit in front of the complainant, while the latter
was interrogated. The accused was then charged w/ robbery. The accused
moved to dismiss the case against him on the ground that he had been denied
the assistance of counsel during the line-up. His motion was denied. Hence, this
petition for certiorari.

HELD: The right to counsel attaches only upon the start of an interrogation,
when the police officer starts to ask questions designed to elicit info. and/ or
confessions or admissions from the accused. As the police line-up in this case
was not part of the custodial inquest, the petitioner was not entitled to counsel
xxx.


III. WAIVER OF RIGHTS.

It is important to distinguish bet. the waiver of rights and the waiver of
warnings. The first can be made provided that the waiver is "voluntary, knowing
and intelligent" but the second cannot. As the warnings are the means of
insuring that the suspect is apprised of his rights so that any subsequent waiver
of his rights can be "voluntary, knowing and intelligent," it is obvious that there
can be no valid waiver of the warnings. A waiver of rights will not be presumed.

1. With respect to confessions obtained bef. Jan. 17, 1973, the rule that
the suspect must be warned that he has a right to remain silent and to have the
assistance of counsel does not apply. such confessions, even though presented
in evidence in a trial after the effectivity of the 1973 Consti., are admissible,
provided they are voluntary, using the traditional test of voluntariness.

2. With respect to confessions obtained after Jan. 17, 1973, but before
March 20, 1985, when the decision of Peo. v. Galit was handed down, the rule is
that the voluntariness of a waiver of the rights to silence and to counsel must be
determined on a case-to-case basis, taking into account the circumstances under
w/c the waiver was made.

3. With regard to confessions obtained after March 20, 1985 but before
Feb. 2, 1987, when the present Consti. took effect, the rule is that a waiver of
the rights to remain silent and to the assistance of counsel, to be valid, must be
made w/ the assistance of counsel.

4. With regard to confessions given after Feb. 2, 1987, the present Consti.
requires that the waiver to be valid, must be in writing and w/ the assistance of
counsel.

IX. THE EXLUSIONARY RULE.

Any confession or admission obtained in violation of this or Sec. 17 hereof
shall be inadmissible in evidence against him, the Consti. says. No distinction is
made bet. confession or admission. Although the previous Consti. spoke of
confessions only, I have argued that it was not so limited but that it also
embraced uncounselled statements. For "if a statement made wore in fact
exculpatory, it could ... never be used by the prosecution, in fact, statements
merely intended to be exculpatory by the defendant are often used to impeach
his testimony at trial or to demonstrate untruths in the statement given under
interrogation and thus to prove guilt by implication."

EXCEPTIONS TO THE EXCLUSIONARY RULE

The phrase "for any purpose in any proceeding" conveys the idea that the
rule excluding evidence illegally obtained is absolute. No similar phraseology is
used in the exclusionary rule implementing the Miranda rule. Does this mean
there can be instances, where uncounselled statements may nevertheless be
admissible in evidence, albeit, for a limited purpose?

In Harris v. US, it was held that although a confession obtained w/o
complying w/ the Miranda rule was inadmissible for the purpose of establishing
in chief the confessor's guilt, it may nevertheless be presented in evidence to
impeach his credit. Petitioner, as a def., in a prosecution for selling heroin,
claimed that what he had sold to a police officer was baking powder, as part of
the scheme to defraud the purchaser xxx The shield provided by Miranda
cannot be perverted into a license to use perjury by way of a defense, free from
the risk of confrontation w/ prior inconsistent utterance

In New York v. Quarles, the SC created a "public safety" exception to the
Miranda rule. xxx. "There is public safety exception to the requirement that
Miranda warnings be given before a suspect's answers may be admitted in
evidence." It held that the warnings were not themselves Constitutional rights
but merely "prophylactic" measures to insure the right against self-
incrimination. The Court noted the cost imposed on the public by the rule,
namely, that the giving of warnings might deter suspects from answering
questions and this might lead in turn to fewer convictions. It then ruled that the
social cost is higher when the giving of warnings might deter suspects from
answering questions than are necessary to avert an immediate threat to public
safety. When answers are not actually coerced, this social cost outweights the
need for Miranda safeguards. In such exigent circumstances, police officers
must not be made to choose bet. giving the warnings at the risk that public
safety will be endangered and withholding the warnings at the risk that
probative evidence will be excluded.


People v. Bolanos, 211 SCRA 262

F: Bolanos was convicted for Murder. The victim, Pagdalian was found
dead, sustaining stab wounds. When the policemen inquired about the
circumstances of the incident, they were informed that the deceased was with
two companions, on the previous night. The accused was apprehended. In the
CONSTI TUTI ONAL LAW 2 ZPG & ASSOCI ATES

vehicle where the accused boarded, on his way to the Police Station, Bolanos
allegedly admitted that he killed Pagdalian because he was abusive.

ISSUE: Whether or not the admission in the jeep was admissible in evidence.

HELD: The trial court, in admitting the extra-judicial confession of the accused in
evidence, violated his Constitutional right to be informed, to remain silent and to
have a counsel of his choice, while already in police custody. Since the extra-
judicial confession was the only basis for the conviction of the accused, the trial
cousts judgment was reversed. Bam.


People v. Bandula, 232 SCRA 566

F: After he and his wife were individually hogtied and their house
ransacked, Atty. Garay was found dead with 3 gunshot wounds . For his
death and the loss of their things on the occasion thereof, Bandula,
Sidigo, Dionanao, and Ejan were charged in court for robbery with
homicide. On the basis of the extrajudicial confessions (EJC) allegedly
made by Bandula and Dionanao during their custodial investigation
which the court found to "have all the qualities and have complied with
all the requirements of an admissible confession, it appearing from the
confession that acussed were informed of their rights under the law
regarding custodial investigation and were duly represented by Counsel
(Atty. Zerna)", it disregarded the defenses interposed by the accused
and convicted Bandula. The 3 other accused were acquitted for
"insufficiency of evidence".

Issue: W/N the extrajudicial confession of Bandula conformed with
the constitutional requisites for its validity, hence admissible in
evidence.

HELD: NO
From the records, it can be gleaned that when accused Bandula
and Dionanao were investigated immediately after their arrest, they
had no counsel present. If at all, counsel came in only a day after the
custodial investigation with respect to Dionanao, and 2 weeks later with
respect to Bandula. And counsel who supposedly assisted both accused
was Atty. Zerna, the Municipal Attorney of Tanjay, whose interest is
admittedly adverse to the accused and who is not an independent
counsel. On top of this, there are telltale signs that violence was used
against the accused. Certainly, these are blatant violations of of Sec.
12, Art III of the 1987 Constitution which protects the rights of the
accused during custodial investigation. Suzette.

1. Miranda rule not applicable to confessions executed before January 17,
1973


2. Not applicable to res gestae statements

People v. Dy, 158 SCRA 111 (1988)
Res gestae (a Latin phrase meaning "things done") is an exception to the rule
against Hearsay evidence. Res gestae is based on the belief that because certain
statements are made naturally, spontaneously and without deliberation during
the course of an event, they leave little room for
misunderstanding/misinterpretation upon hearing by someone else( i.e. by the
witness who will later repeat the statement to the court) and thus the courts
believe that such statements carry a high degree of credibility. Evidence which
can be admitted into evidence as Res gestae fall into three headings:

Words or phrases which either form part of, or explain a physical act,
Exclamations which are so spontaneous as to belie concoction, and
Statements which are evidence as to someone's state of mind.

3. Not applicable to statements given in administrative investigations

People v. Ayson, 175 SCRA 216 (1989)

It should at once be apparent that there are two (2) rights, or
sets of rights, dealt with in the section, namely:

1) the right against self-incrimination i.e., the right of a person not to be
compelled to be a witness against himself set out in the first sentence, which is
a verbatim reproduction of Section 18, Article III of the 1935 Constitution, and is
similar to that accorded by the Fifth Amendment of the American Constitution,
and
2) the right of a person in custodial interrogation, i.e., the rights of every
suspect "under investigation for the commission of an offense."

Parenthetically, the 1987 Constitution indicates much more clearly the
individuality and disparateness of these rights. It has placed the rights in
separate sections. The right against self- incrimination, "No person shall be
compelled to be a witness against himself," is now embodied in Section 17,
Article III of the 1987 Constitution. The rights of a person in custodial
interrogation, which have been made more explicit, are now contained in
Section 12 of the same Article III.

Right Against Self-Incrimination

The first right, against self-incrimination, mentioned in Section 20, Article
IV of the 1973 Constitution, is accorded to every person who gives evidence,
whether voluntarily or under compulsion of subpoena, in any civil, criminal, or
administrative proceeding. The right is NOT to "be compelled to be a witness
against himself." It prescribes an "option of refusal to answer incriminating
questions and not a prohibition of inquiry." It simply secures to a witness,
whether he be a party or not, the right to refuse to answer any particular
incriminatory question, i.e., one the answer to which has a tendency to
incriminate him for some crime. However, the right can be claimed only when
the specific question, incriminatory in character, is actually put to the witness. It
cannot be claimed at any other time. It does not give a witness the right to
disregard a subpoena, to decline to appear before the court at the time
appointed.

The right against self-incrimination is not self-executing or automatically
operational. It must be claimed. It follows that the right may be waived,
expressly, or impliedly, as by a failure to claim it at the appropriate time.

Rights in Custodial Interrogation

Section 20, Article IV of the 1973 Constitution also treats of a second
right, or better said, group of rights. These rights apply to persons "under
investigation for the commission of an offense," i.e., "suspects" under
investigation by police authorities; and this is what makes these rights different
from that embodied in the first sentence, that against self-incrimination which,
as aforestated, indiscriminately applies to any person testifying in any
proceeding, civil, criminal, or administrative.

This provision granting explicit rights to persons under investigation for an
offense was not in the 1935 Constitution. It is avowedly derived from the
decision of the U.S. Supreme Court in Miranda v. Arizona, a decision described
as an "earthquake in the world of law enforcement."

Section 20 states that whenever any person is "under investigation for the
commission of an offense"--

1) he shall have the right to remain silent and to counsel, and to be
informed of such right,
2) no force, violence, threat, intimidation, or any other means which
vitiates the free will shall be used against him; and
3) any confession obtained in violation of these rights shall be
inadmissible in evidence.

Miranda rights

He must be warned prior to any questioning that he has the right to
remain silent, that anything he says can be used against him in a court of law,
that he has the right to the presence of an attorney, and that if he cannot afford
an attorney one will be appointed for him prior to any questioning if he so
desires. Opportunity to exercise those rights must be afforded to him
throughout the interrogation. After such warnings have been given, such
opportunity afforded him, the individual may knowingly and intelligently waive
these rights and agree to answer or make a statement. But unless and until such
warnings and waiver are demonstrated by the prosecution at the trial, no
evidence obtained as a result of interrogation can be used against him.

The objective is to prohibit "incommunicado interrogation of individuals
in a police-dominated atmosphere, resulting in self- incriminating statement
without full warnings of constitutional rights."

The rights above specified, to repeat, exist only in "custodial
interrogations," or "in-custody interrogation of accused persons." And, as this
Court has already stated, by custodial interrogation is meant "questioning
initiated by law enforcement officers after a person has been taken into custody
or otherwise deprived of his freedom of action in any significant way."

Rights of Defendant in Criminal Case As Regards Giving of Testimony

In fine, a person suspected of having committed a crime and subsequently
charged with its commission in court, has the following rights in the matter of his
testifying or producing evidence, to wit:

1) BEFORE THE CASE IS FILED IN COURT (or with the public prosecutor,
for preliminary investigation), but after having been taken into custody or
otherwise deprived of his liberty in some significant way, and on being
interrogated by the police: the continuing right to remain silent and to counsel,
and to be informed thereof, not to be subjected to force, violence, threat,
CONSTI TUTI ONAL LAW 2 ZPG & ASSOCI ATES

intimidation or any other means which vitiates the free will; and to have
evidence obtained in violation of these rights rejected; and

2) AFTER THE CASE IS FILED IN COURT

a) to refuse to be a witness;
b) not to have any prejudice whatsoever result to him by such
refusal;
c) to testify in his own behalf, subject to cross-examination by
the prosecution;
d) WHILE TESTIFYING, to refuse to answer a specific question
which tends to incriminate him for some crime other than that for
which he is then prosecuted.

It is clear from the undisputed facts of this case that Felipe Ramos was not
in any sense under custodial interrogation, as the term should be properly
understood, prior to and during the administrative inquiry into the discovered
irregularities in ticket sales in which he appeared to have had a hand. The
constitutional rights of a person under custodial interrogation under Section 20,
Article IV of the 1973 Constitution did not therefore come into play, were of no
relevance to the inquiry.

4. Custodial Phase of Investigation

Police Lineups

Gamboa v. Cruz June 27, 1988

Police line-up not part of custodial inquest

F: Petitioner was arrested for vagrancy in Manila. The following day, he was
included in a police line-up and was identified as one of the suspects in a
robbery case. He was later charged with robbery and charged. He moved to
dismiss the case on the ground that the conduct of the line-up, without the
assistance of counsel, was unconstitutional.

HELD: The police line-up was not part of the custodial inquest, hence, petitioner
was not yet entitled, at such stage, to counsel. VV.


US v. Wade, 388 US 218 (1967)

People v. Hatton, 210 SCRA 1

F: Algrame was stabbed at the back while walking with several companions
including Ongue who vaguely recognized the assailant, describing the latter as a
"mestizo." Two days later, Ongue was invited by the police to identify the
suspect in a police line- up. Hatton was pointed by Ongue as the assailant.
Hatton alleges that at the time that he was made to stand in the police line-up,
he was not assisted by counsel. Hence, his identification therein by Ongue is
inadmissble.

RULING: When the suspect was brought to the police station for indentification,
technically, he was not yet under custodial investigation. Thus, the right to
counsel does not yet apply.
However, there is every reason to doubt the regularity of the
identification of the suspect by the witness. During the proceedings in the police
station, Ongue identified Hatton not because he was certain that Hatton was
really the assailant but because he was the only mestizo in the station and
because he was pointed by the police as the suspect. This cannot be considered
as positive identification of the accused by the witness.

5. Tests of Validity of Waiver of Miranda Rights


No valid waiver.

People v. Caguioa 95 SCRA 2 (1980)

Right to counsel may be waived provided the waiver is voluntary, knowing and
intelligent

F: Respondent Paquito Yupo was accused of murder in the CFI of Bulacan.
The prosecution presented Corporal Conrado Roca of the Meycauayan Police
who identified a statement of the accused during a police interrogation and his
alleged waiver of the right to remain silent and to counsel. When Roca was
questioned on the incriminating answers in the statement, the defense
objected, contending that Yupo's statement was given without the assistance of
counsel. Respondent Judge sustained the objection on the ground that the right
to counsel cannot be waived.

HELD: The right to counsel during custodial interrogation may be waived
provided the waiver is made intelligently and voluntarily, with full understanding
of its consequences. In this case, the statement made only a perfunctory
opening question, after informing the suspect that he was under investigation,
that he had a right to counsel and that anything he said could be used for or
against him and after asking whether he was willing to answer questions and he
answered "yes." The statement was in Tagalog which the defendant, a native of
Samar, had not been shown to be fully acquainted with. The date of execution of
the statement before the municipal court was not indicated. The separate
statement signed by the defendant stating he was made to read the opening
statement containing the Miranda warnings and that they were explained to him
all the more engenders doubt as to whether the defendant was properly
informed of his right.

People v. Tampus 96 SCRA 624 (1980)

Public trial; waiver of right to counsel

F: Jose Tampus and Rodolfo Avila were prisoners at the National
Penitentiary in Muntinlupa, Rizal. On June 14, 1976, they attacked and killed
Celso Saminado, another prisoner. Afterwards, they surrendered to the prison
guard, saying "surrender po kami. Gumanti lang po kami." Two days later, they
gave extrajudicial confessions admitting the killing. They were accused of
murder and pleaded guilty. They took the witness stand and affirmed their
confessions. Tampus was sentenced to death while Avila to reclusion temporal.
Trial took place at the Penitentiary. On review, it was contended that Tampus
was denied the right to a public trial and to counsel.

HELD: The record does not show that the public was actually excluded from the
place where the trial was held or that the accused was prejudiced by the holding
of the trial there. Anyway, the right to public trial may be waived. In another
case where Avila was also a defendant, the SC directed that, for security reasons,
Avila's trial be held in the National Penitentiary. The accused was warned in
Tagalog that he had a right to remain silent and to counsel but despite this, he
was willing to answer questions of the police. There is no doubt that the
confession was voluntarily made. The truth is that shortly after the killing,
Tampus and Avila admitted their guilt. That spontaneous statement, elicited
without interrogation, was part of the res gestae and at the same time was a
voluntary confession of guilt. By means of that statement given freely on the spur
of the moment without any urging or suggestion, the two waived their right to
remain silent and to counsel.

People v. Poyos 143 SCRA 543 (1986)

No valid waiver of right to counsel and to silence

F: Poyos was convicted of the murder of a 77-year-old woman and sentence
to death. His conviction was based solely on his extrajudicial confession which
he disowned in court. The confession was given to the police and subscribed
before the clerk of court and contains a waiver.

HELD: It is doubtful, given the tenor of the question whether there was a
definite waiver by the suspect of his right to counsel. His answer was categorical
enough, to be sure, but the question itself was not since it spoke of a waiver only
"for the moment." As worded, the question suggested a tentativeness that
belied the suspect's supposed permanent foregoing of his right to counsel, if
indeed there was any waiver at all. Moreover, he was told that he could hire a
lawyer but not that one could be provided for him for free. VV.

Since Royo's conviction for murder was based on a written confession
showing that he was apprised of his right not only by the police but also by the
fiscal, but that he waived these rights, then the waiver found to be voluntary,
knowing and intelligent and thus admissible.


b. The Galit Rule (March 20, 1985 to Feb. 2, 1987)

It is not enough that the confession is voluntary, knowing and intelligent.
The waiver must be made in the presence of counsel. Waiver of the right to
counsel must be made with the assistance of counsel. This rule applied from
March 20, 1985 to February 2, 1987.

In People v. Galit, 135 SCRA 485 (1985), the SC, reiterating a dictum in
Morales v. Enrile, 121 SCRA 538 (1983), ruled that no custodial investigation
should be conducted unless it be in the presence of counsel, and that although
the right to counsel may be waived, the waiver should not be valid unless made
with the assistance of counsel.

In the Galit case, however, the adoption of the Morales obiter was also an
obiter. The confession in this case was traditionally involuntary, and so the SC
did not need the Morales obiter in order to disallow the confession.

Under the facts of the case, the accused Galit was convicted of robbery
with homicide on the basis of his confession, which was obtained through
torture. The NBI investigators covered Galit's face with a rag and then pushed it
into a toilet bowl full of human waste. It was only after they had broken his will
that he signed the confession and posed for pictures for re-enactment as
directed by the investigators.

CONSTI TUTI ONAL LAW 2 ZPG & ASSOCI ATES


People v. Galit 135 SCRA 465 (1985)

F: Defendant was convicted of robbery with homicide by the Circuit Criminal
Court. The principal prosecution witness testified that he heard the defendant
and his wife, who was the mother of the witness' wife, quarrelling the morning
after the crime. He said the defendant wanted to leave their house because he
and his companions had robbed "Aling Nene." The prosecution also presented
the extrajudicial confession of the defendant.

HELD: The confession of the defendant is inadmissible because it was obtained
through torture. The NBI investigators covered the defendant's face with a rag
and then pushed in into a toilet bowl full of human waste. It was only after they
had broken his will that the defendant signed the confession and posed for
pictures for reenactment as directed by the investigators. The defendant is from
Samar and there is no showing that he understood Tagalog. It was two weeks
after he executed the salaysay that his relatives were permitted to visit him. His
statement does not contain any waiver or right to counsel and yet during the
investigation he was not assisted by one. These constitute gross violations of his
right. The SC cited the case of Morales v. Ponce Enrile where it laid the
procedure in custodial investigations: No custodial investigation shall be
conducted unless it be in the precense of counsel engaged by the person
arrested, or by any person on his behalf, or appointed by the court upon petition
either of the detainee himself or of anyone on his behalf. The right to counsel
may be waived but the waiver shall not be valid unless made with the assistance
of counsel. Any statement obtained in violation of this, whether exculpatory or
inculpatory, in whole or in part, shall be inadmissible in evidence.


Whatever doubt as to the validity of the Galit rule, however, was laid to
rest by the SC in People v. Sison, 142 SCRA 219 (1986). The Court held that in
People v. Galit, which was decided en banc and concurred in by all the Justices
except one who took no part, the Court was out to rest all doubts regarding the
ruling in Morales v. Enrile, and embraced its ruling.

In this case, the prosecution sought to prove its charge of subversion
against Asis by means of her confession given in the hospital, in which she
admitted through a leading question, that she was a member of the NPA and
that she was wounded in the encounter. The SC upheld the trial court's decision
excluding the confession on the ground that the waiver of the Miranda rights
was made without the assistance of counsel.


People v. Sison 142 SCRA 219 (1986)

F: Jocelyn de Asis was accused of subversion. At the trial, the Fiscal offered
as evidence an extrajudicial confession given by her in the hospital. In that
confession, she admitted, through a leading question that she was a member of
the NPA. The trial court excluded the confession on the ground that the waiver
of Miranda rights was made without the assistance of counsel. The prosecution
contends that the ruling in Morales v. Ponce Enrile that the right to counsel may
be waived only with the assistance of counsel, was only a dictum.

HELD: In the case of People v. Galit, which was decided en banc and concurred
in by all Justices except one who took no part, the SC put to rest all doubts
regarding the ruling in Morales v. Ponce Enrile and Moncupa v. Enrile.



People v. Lim, 196 SCRA 809 (1991)

In People v. Nabaluna, 142 SCRA 446 (1986), Nabaluna et. al. were
convicted of robbery with homicide on the basis, among others, of extrajudicial
confessions taken in 1977. The confessions and the special counsel before
whom the confessions were signed prove that the Miranda warnings were given,
but these were not made in the presence of counsel. The SC, in allowing the
confession, ruled that the GAlit ruling could not have a retroactive effect,
especially since in this case the trial court decision was already rendered before
the SC pronouncement.



People v. Lasac 148 SCRA 624 (1987)

F: Appellant was convicted of parricide on the basis of a confession and
circumstantial evidence which the trial court found substantial to establish guilt.

HELD: The waiver by the appellant of his right to counsel was made without the
assistance of a counsel. The SC has held in Morales v. Ponce Enrile, People v.
Galit and People v. Sison (1986) that this requirement is mandatory. Any
statement obtained in violation of this procedure shall be inadmissible in
evidence. VV.

c. New rule on waiver (Feb. 2, 1987)

Art. III, Sec. 12 (1): Waiver must be in writing and made in the presence of
counsel

Art. III, Sec. 12. (1) Any person under investigation for the commission
of an offense shall have the right to be informed of his right to remain silent
and to have competent and independent counsel preferably of his choice. If
the person cannot afford the services of counsel, he must be provided with
one. These rights cannot be waived except in writing and in the presence of
counsel.


Under the new Constitution, any waiver must now be made (1) in writing,
and (2) in the presence of counsel.

6. The burden of proving voluntariness of waivers is on the prosecution

The burden to prove that there was a valid waiver of the Miranda warning
devolves upon the one seeking to present the confession, that is, on the
prosecution. This rule applies whether in the pre-Galit, Galit, or 1987 rule.

In People v. Jara, 144 SCRA 516 (1986), the SC noted that the stereotype
"advice" appearing in practically all extrajudicial confessions which are later
repudiated has assumed the nature of a legal form. Investigators automatically
type it together with "opo" as the answer, or ask the accused to sign it or even
copy it in their handwriting. Its tired punctilious, fixed and artificially stately
style does not create an impression of voluntariness or even understanding on
the part of the accused.

Whenever a Constitutional protection is waived by one entitled to that
protection, the presumption is always against the waiver. Thus, the prosecution
must prove with strongly convincing evidence that indeed the accused willingly
and voluntarily submitted his confession, and knowingly and deliberately
manifested that he was not interested in having a lawyer assist him during the
taking of that confession.


People v. Jara, 144 SCRA 516 (1986)

F: Appellants were found guilty of robbery with homicide for the killing and
robbery of Ampara vda. de Bantigue on June 9, 1978. In another case, two of the
appellants were found guilty of homicide for the killing on the same date of Luisa
Jara while Felicisimo Jara, the husband of the deceased, was found guilty of
parricide. Two of the appellants, Raymundo Vergara and Bernardo Bernadas,
made extrajudicial confessions implicating Jara as the mastermind. The
confessions were taken while the two were held incommunicado in the presence
of five policemen and after two weeks of detention.

HELD: The stereotyped "advice" of the Miranda rights appearing in practically all
extrajudicial confessions which are later repudiated assumed the nature of a
legal form or model. Its tired, punctilious, fixed and artificial style does not
create an impression of voluntariness or even understanding on the part of the
accused. The showing of a spontaneous, free and unconstrained giving up of a
right is missing. Whenever a protection given by the Constitution is waived by
the person entitled to that protection, the presumption is always against the
waiver. Consequently, the prosecution must prove with strong, convincing
evidence that indeed the accused willingly and voluntarily submitted his
confession and knowingly and deliberately manifested that he was not
interested in having a lawyer assist him during the taking of that confession.
That proof is missing in this case.

7. What may be waived: The right to remain silent and to counsel, but not
the right to be given "Miranda warnings"

The right to remain silent and to counsel, which are the effectuations of
the Miranda rights, can be waived.

What cannot be waived are:

1. The right to be given the Miranda warnings. (For how can one waive
what one does not know?)

2. The right to counsel when making the waiver of the right to remain
silent or to counsel.


8. Exclusionary rule

Art. III, Sec. 12. xxx
(3) Any confession or admission obtained in violation of this or Section
17 hereof shall be inadmissible in evidence against him.


Note than under [Art. III, Sec. 3(2)] the exclusionary rule reads: (any
evidence obtained in violation of this or the preceding section shall be
inadmissible "for any purpose in any proceeding."
CONSTI TUTI ONAL LAW 2 ZPG & ASSOCI ATES

There are two exceptions to the exclusionary rule. One, to impeach the
credibility of the accused. Two, public safety.

Impeach the credibility

The unwarned or uncounselled confession is not totally without use.
While it is not admissible to prove the guilt of the accused, it may be used
against him to impeach his credibility by showing that he is lying in court, so
ruled the U.S. Supreme Court in Harris v. New York, 401 U.S. 222 (1971).


Harris v. New York, 401 U.S. 222 (1971)

In this case, Harris was arrested for twice selling heroin to an undercover
police agent. He confessed to the crime during the police interrogation, but the
confession was uncounselled, and so it was held as inadmissible in evidence. But
when Harris took the witness stand, he testified that what he sold was baking
powder in order to defraud the police agent. The SC allowed the prosecution to
introduce the uncounselled statment to show that he was lying.

In justifying the admission of the testimony, Justice Burger said that it is
one thing to say that the government cannot make an affirmative use of the
evidence unlawfully obtained, and quite another to say that the defendant can
turn the illegal method by which the evidence in the possession of the
government was obtained to his own advantage, providing himself with a shield
against perjury and the contradiction of his untruths.

The reason, continued the Court is that the shield provided by the
Miranda rights cannot be perverted into a license to use perjury by way of a
defense, free from the risk of confrontation with prior inconsistent utterances.

Public Safety

Public Safety may justify the police in taking confessions without prior
warning. Thus ruled the U.S. Supreme Court in New York v. Quarles, 104 S. Ct.
2626 (1984).



New York v. Quarles, 104 S. Ct. 2626 (1984).

In the case, the Court excused the giving of the Miranda warning because
the public safety required that the weapon had to be located before it could be
used by the accused against those in the supermarket.

The criticism hurled against this ruling is that while the police may be
justified in forcing the assailant to say where the weapon is located, he is not
justified to present this in evidence in the subsequent criminal prosecution.


C. Right to bail

Art. III, Sec. 13. All persons, except those charged with offenses
punishable by reclusion perpetua when the evidence of guilt is strong, shall,
before conviction, be bailable by sufficient sureties, or be released on
recognizance as may be provided by law. the right to bail shall not be impaired
even when the privilege of the writ of habeas corpus is suspended. Excessive
bail shall not be required.

1. When right may be invoked

The right to bail is available from the very moment of arrest (which may
be before or after the filing of formal charges in court) up to the time of
conviction by final judgement (which means after appeal).

No charge need be filed formally before one can file for bail, so long as
one is under arrest. So ruled the SC in Heras Teehankee v. Rovica. 75 Phil.634
(1945).

The case was unique in that after the war, the People's Court Act
amended Art. 125 of the RPC to allow for a longer time to detain persons
because of the impossibility of filing charges within the reglementary period due
to the number of indictees.

Bail and Habeas Corpus

In the case of bail, there is an implicit recognition that the arrest and
detention, are valid, or that even if they were initially illegal, such illegality was
cured by the subsequent filing of a case in court. Thus, the prayer in bail is that
one be released temporarily from such valid detention, and this can be made
anytime after arrest.

In habeas corpus, the assumption is precisely that the arrest and
detention are illegal, so that the prayer is to be released permanently from such
illegal detention. When the privilege of the writ is suspended, the arrest and
detention remain illegal, but the remedy afforded by law to the victim is not
available. Under the 1987 Constitution, though the effect of the suspension has
been considerably lessened to the need to file a case within 72 hours from the
illegal arrest, otherwise the detainee is to be released.

The Constitution now provides, overruling Morales v. Enrile, that the
suspension of the privilege of the writ does not carry with it the suspension of
the right to bail. Habeas Corpus refers to illegal detention, while bail refers to
legal detention, or even detention that started as illegal but was cured by the
filing of a case in court.

2. When bail is a matter of right, when it is a matter of discretion

Bail is a matter of right in all cases not punishable by reclusion perpetua.

It is a matter of discretion in case the evidence of guilt is strong. In such a
case, according to People v. San Diego, 26 SCRA 522 (1966), the court's
discretion to grant bail must be exercised in the light of a summary of the
evidence presented by the prosecution. Thus, the order granting or refusing bail
must contain a summary of the evidence for the prosecution followed by the
conclusion on whether or not the evidence of guilt is strong.

The only time bail may be denied is when (a) the offense is punishable by
reclusion perpetua, and (b) the evidence of guilt is strong.

With the abolition of the death penalty (III, 20), and the automatic
commutation of a death sentence to reclusion perpetua, it is contended that
when the 1987 Constitution denies the right to bail in offenses punishable by
reclusion perpetua, it is meant to apply only to those crimes which were once
punishable by death. For if it includeds even those crimes which before and now
are really punishable by reclusion perpetua, it would go against the very spirit of
the Constitution.

People v. Donato, 196 SCRA 130 (1991)

3. Bail in courts-martial

Commendador v. De Villa, 200 SCRA 80 (1991)

4. Standards for fixing bail

Rule 114, Sec. 6. Amount of bail; guidelines.-- The judge who issed the
warrant or granted the application shall fix a reasonable amount of bail
considering primarily, but not limited to the following guidelines:
(a) Financial ability of the accused to give bail;
(b) Nature and circumstances of the offense;
(c) Penalty of the offense charged;
(d) Character and reputation of the accused;
(e) Age and health of the accused;
(f) The weight of the evidence against the accused;
(g) Probability of the accused appearing in trial;
(h) Forfeiture of other bonds;
(i) The fact that accused was a fugitive from justice when arrested; and
(j) The pendency of other cases in which the accused is under bond.
Excessive bail shall not be required.


Where the right to bail exists, it should not be rendered nugatory be
requiring a sum that is excessive, otherwise, it becomes "a promise to the ear to
be broken to the hope, a teasing illusion like a munificent bequest in a pauper's
will" (Jackson). Thus, said the SC in De la Camara v. Enage, 41 SCRA 1 (1971).

In this case, a bail of P1.195 million imposed against Mayor Camara for
charges of 12 murders and 12 frustrated murder was found excessive.

The SC laid down the following guidelines in fixing the amount of bail in
Villasenor v. Abano, 21 SCRA 312 (1967), later contained in sec. 6 of Rule 114.

1. Ability of the accused to give the bail.
2. Nature of the offense.
3. Penalty for the offense charged.
4. Character and reputation of the accused
5. Health of the accused.
6. Character and strength of the evidence.
7. Probability of the accused appearing in trial.
8. Forfeiture of other bonds.
9. Whether the accused was a fugitive from justice when arrested.
10. If the accused is under bond for appearance at trial in other cases.

Even when the accused has previously jumped bail, still he cannot be
denied bail. the remedy in this case is to increase the amount of the bail
(Siquiam v. Amparo).

5. Right to bail and right to travel abroad

CONSTI TUTI ONAL LAW 2 ZPG & ASSOCI ATES

Art. III, Sec. 6. The liberty of abode and of changing the same within the
limits prescribed by law shall not be impaired except upon lawful order of the
court. Neither shall the right to travel be impaired except in the interest of
national security, public safety, or public health, as may be provided by law.


In Manotoc v. Court of Appeals, 142 SCRA 149 (1986), the SC disallowed a
person released on bail to travel abroad for a business trip. The Court gave 2
reasons why bail operates only within the country.

One, the accused may be placed beyond the jurisdiction of the court if he
were allowed to leave the Philippines without sufficient reason, thus affecting
one of the conditions in the grant of bail, namely to have the accused available
whenever the court requires his presence.

Two, implicit in the bail is the agreement between the State and the
surety that the State will do nothing to make it difficult for the surety to arrest
the defendant upon order of the court. If the court thus allows his to leave, then
the State loses its right to order the forfeiture of the bond because it itself has
breached its obligation to the surety.

The case leaves the question of allowing an accused under bail to go
abroad for humanitarian reasons open-ended. This reason was not foreclosed
by the Court, which hinted that the accused could be allowed to leave if he had
"sufficient reason". What the Court found insufficient was the business trip.


Manotoc v. CA, 142 SCRA 149 (1986)

F: Petitioner is a principal stockholder of two corporations, in one of which
he was the president. The firms were placed under a management committee by
the SEC and petitioner was placed "on hold" by the Commission of Immigration.
Petitioner was charged with estafa. He later asked for permission to leave the
country for business reasons, but his request was denied by the courts. He filed
a petition for certiorari but his petition was also dismissed for lack of merit. He
appealed to the SC.

HELD: The condition imposed by Rule 114, sec. 1 upon the accused to make
himself available whenever the court requires his presence, operates as a valid
restriction on his right to travel. The constitutional right to travel is not absolute,
but is subject to lawful orders of the court. VV.


6. Waiver of the Right to Bail

People v. Donato, 198 SCRA 130 (1991)

D. Rights during trial

Art. III, Sec. 14. (1) No person shall be held to answer for a criminal
offense without due process of law.
In all criminal prosecutions, the accused shall be presumed innocent
until the contrary is proved, and shall enjoy the right to be heard by himself
and counsel, to be informed of the nature and cause of the accusation against
him, to have a speedy, impartial and public trial, to meet the witnesses face to
face, and to have compulsory process to secure the attendance of witnesses
and the production of evidence in his behalf. However, after arraignment, trial
may proceed notwithstanding the absence of the accused provided that he has
been duly notified and his failure to appear is unjustifiable.


1. Presumption of innocence

In People v. Dramayo, 42 SCRA 69 (1971), the SC noted that the
requirement of proof beyond reasonable doubt is a necessary corollary of the
constitutional right to be presumed innocent.


In Igot v. Comelec, 95 SCRA 392 (1980), a law disqualifying candidates
charged with national security offences was struck down as unconstitutional, for
violating the presumption against innocence.


In Alejandro v. Pepito, 96 SCRA 322 (1980), a judge who allowed the
accused to present his evidence ahead of the prosecution, over the objection of
the prosecution, after the acused admitted the killing but invoked self-defense,
was reversed by the SC on the ground that this change in the order of trial
violated the constitutional presumption of innocence which places the burden
proof on the prosecution.

This ruling was modified by Rule 119, sec. 3 (e) of the 1985 Rules of
Criminal Procedure which now reverses the order of trial when the defendant
admits the act but invokes a justifying or exempting circumstance.


People v. de Guzman, 231 SCRA 739

F: De Guzman, Castro and Catap were charged with murder for the
killing of an unidentified person on Nov. 16, 1994. Only De Guzman and
Castro were arrested and both pleaded not guilty. They were convicted
by the court mainly on the basis of the testimony of Adelia Angeles.
She positively identified the 2 accused as the persons who were with
Catap who maltreated an unidentified person whom they had tied to an
ipil-ipil tree and upon seeing her, she testified that they untied the man
and brought him towards the direction of the Pasig river which was only
3 houses away. This was further strengthened by the extrajudicial
confession (EJC) of accused Castro to Police Corporal Dominador
Cunanan that it was Catap who killed the victim and that he and de
Guzman acted only as look-outs.

Issue: W/N the constitutional presumption of innocenec of the
accused has been overcome.

HELD: YES
Though there is no direct evidence to link the 3 accused to the
killing of the unknown victim, the circumstantial evidence presented
satisfied Sec. 4, Rule 133 ROC namely: (1) there is more than one
circumstance; (2) the facts from which the inferences are derived are
proven; and (3) the combination of all the circumstances is such as to
produce conviction beyond resonable doubt.
With regard to the EJC of Accused Castro to Police Cpl. Cunanan, there is
no evidence that Cunanan had any motive to falsely testify against accused.
While it is true that accused's EJC was made without the advice and assistance of
counsel, hence inadmissible as evidence, it could be treated as a verbal
admission of the accused established through the testimonies of persons who
heard it or who conducted the investigatiuon of the accused (Peo v Molas 218
SCRA 473). Moreover in Peo v Alvarez, the court ruled that an extrajudicial
confession is admissible against a co-accused when it is used as a circumstantial
evidence to show the probability of the participation of said co-accused in the
crime committed.

2. Right to be heard personally or by counsel

Adequate legal assistance shall not be denied to any person by reason of
poverty (Art. III, Sec. 11.) No matter how educated one may be, he may not
know how to establsih his innocence for the simple reason that he does not
know the rules of evidence said the SC in People v. Holgado, 85 Phil 752 (1952).


People v. Holgado, 85 Phil 752 (1952)

F: Appellant Frisco Holgado was charged in the court of First Instance of
Romblon with slight illegal detention because he did "feloniously and without
justifiable motive, kidnap and detain one Artemia Fabreag in the house of
Antero Holgado for about eight hours thereby depriving said Artemia Fabreag of
her personal liberty." Accused, unaided by counsel, pleaded guilty and said that
he was instructed by Mr. Ocampo to do so.
Accused was convicted of a capital offense.
Since the accused-appellant pleaded guilty and no evidence appears to
have been presented by either party, the trial judge must have deduced the
capital offense from the facts pleaded in the information.

Ruling: Under the circumstances, particularly the qualified plea given by the
accused who was unaided by counsel, it was not prudent, to say the least, for
the trial court to render such a serious judgment finding the accused guilty of a
capital offense, and imposing upon him such a heavy penalty as ten years and
one day of prision mayor to twenty years, without absolute any evidence to
determine and clarify the true facts of the case.
The proceedings in the trial court are irregular from the beginning. It is
expressly provided in our rules of Court, Rule 112, section 3 (now Rule 116, Sec.
6), that:

If the defendant appears without attorney, he must be informed by the
court that it is his right to have attorney before being arraigned., and must be
asked if he desires the aid of attorney, the Court must assign attorney de oficio
to defend him. A reasonable time must be allowed for procuring attorney.

Not one of these duties had been complied with by the trial court.
One of the great principles of justice guaranteed by our Constitution is
that "no person shall be held to answer for a criminal offense without due
process of law", and that all accused "shall enjoy the right to be heard by himself
and counsel." In criminal cases there can be no fair hearing unless the accused
be given the opportunity to be heard by counsel. The right to be heard would be
of little avail if it does not include the right to be heard by counsel. Even the
most intelligent or educated man may have no skill in the science of the law,
particularly in the rules of procedure, and, without counsel, he may be convicted
not because he is guilty but because he does not know how to establish his
innocence. And this can happen more easily to persons who are ignorant or
uneducated. It is for this reason that the right to be assisted by counsel is
deemed so important that it has become a constitutional right and it is so
CONSTI TUTI ONAL LAW 2 ZPG & ASSOCI ATES

implemented that under our rules of procedure it is not enough for the Court to
apprise an accused of his right to have an attorney, it is not enough to ask him
whether he desires the aid of an attorney, but it is essential that the court
should assign one de oficio if he so desires and he is poor grant him a reasonable
time to procure an attorney of his own.


Q: What happens if the accused files a demurrer to the evidence of the
prosecution (on the ground that the prosecution failed to tender a case) and this
motion is denied -- could the defense still present its own evidence?

In Abriol v. Homeres, 84 Phil 525, (1949), the SC ruled in the affirmative,
contending that the right of the accused to present his evidence is a
constitutional right which cannot be defeated by the dismissal of the motion of
demurrer.

Filing of demurrer to evidence is a WAIVER of right to be heard (Rule 119, Sec.
15.)

Abriol v. Homeres, 84 Phil 525, (1949)

F: Fidel Abriol, together with six other persons, was accused of illegal
possession of firearms and ammunition. After the prosecution had presented its
evidence and rested its case, counsel for the defense moved to dismiss the case
on the ground of insufficiency of the evidence to prove the guilt of the accused.
After hearing the arguments for and against the motion for dismissal, the court
held the proofs sufficient to convict and denied said motion, whereupon counsel
for the defense offered to present evidence for the accused. The provincial fiscal
opposed the presentation of evidence by the defense, contending that the
present procedural practice and laws precluded the defense in criminal cases
from presenting any evidence after it had presented a motion for dismissal with
or without reservation and after said motion had been denied, and citing as
authority the case of United States vs. De la Cruz, 28 Phil., 279. His Honor Judge
S. C. Moscoso sustained the opposition of the provincial fiscal and, without
allowing the accused to present evidence in their defense, convicted all of them
and sentenced the herein petitioner to suffer seven years of imprisonment and
to pay a fine of P2,000.

Issue: Whether the accused should be allowed to present evidence after the
denial of their motion to dismiss on the ground of insufficiency of evidence of
the prosecution

Ruling: The accused should be allowed to present evidence.

1. The refusal of Judge Moscoso to allow the accused-petitioner to
present proofs in his defense after the denial of his motion for dismissal was a
palpable error which resulted in denying to the said accused the due process of
law guaranteed in the Bill of Rights embodied in the Constitution, it being
provided in Article II, section 1 (17), of the Constitution that in all criminal
prosecutions the accused shall enjoy the right to be heard by himself and
counsel and to have compulsory process to secure the attendance of witnesses
in his behalf. There is no law nor "procedural practice" under which the accused
may ever be denied the right to be heard before being sentenced.
Now that the Government cannot appeal in criminal cases if the
defendant would be placed thereby in double jeopardy (sec. 2, Rule 118), the
dismissal of the case for insufficiency of the evidence after the prosecution has
rested terminates the case then and there. But if the motion for dismissal is
denied, the court should proceed to hear the evidence for the defense before
entering judgment regardless of whether or not the defense had reserved its
right to present evidence in the event its motion for dismissal be denied. The
reason is that it is the constitutional right of the accused to be heard in his
defense before sentence is pronounced on him. Of course if the accused has no
evidence to present or expressly waives the right to present it, the court has no
alternative but to decide the case upon the evidence presented by the
prosecution alone.

2. The main question to decide is whether the writ of habeas corpus lies in
a case like the present.
We have already shown that there is no law or precedent which could be
invoked to place in doubt the right of the accused to be heard or to present
evidence in his defense before being sentenced. On the contrary, the provisions
of the Constitution hereinabove cited expressly and clearly guarantee to him
that right. Such constitutional right is inviolate. No court of justice under our
system of government has the power to deprive him of that right. If the accused
does not waive his right to be heard but on the contrary as in the instant case
invokes that rough, and the court denies it to him, that court no longer has
jurisdiction to proceed; it has no power to sentence the accused without hearing
him in his defense; and the sentence thus pronounced is void and may be
collaterally attacked in a habeas corpus proceeding.
Although the sentence against the petitioner is void for the reasons
hereinabove stated, he may be held under the custody of the law by being
detained or admitted to bail until the case against him is finally and lawfully
decided. The process against him in criminal case No. 1472 may stand should be
resumed from the stage at which it was vitiated by the trial court's denial of his
constitutional right to be heard. Up to the point when the prosecution rested,
the proceedings were valid and should be resumed from there.


People v. Donesa, 49 SCRA 281 (1973)

Grant of demurrer is equivalent to an acquittal

F: After prosecution presented its witnesses, the defense moved for
dismissal of the case on the ground of insufficiency of evidence. The judge
granted the motion.

Issue: Did such dismissal operate as an acquittal of the accused?

Ruling: YES
A dismissal ordered after the termination of the presentation of the
evidence for the prosecution has the force and effect of an acquittal. Since there
is a failure to prove the guilt of the accused, the case must be dismissed, and it
will be a bar to another prosecution for the same offense even though it was
ordered by the Court upon motion or with the express consent of the defendant,
in exactly the same way as a judgment of acquittal.

Rule 119, Sec. 15. Demurrer to evidence.-- After the prosecution has rested its
case, the court may dismiss the case on the ground of insufficiency of
evidence: (1) ont its own intitiative after givint the prosecution an
opportunity to be heard; or (2) on motion of the accused filed with proper
leave of court.
If the court denies the motion for dismissal, the accused may adduce
evidence in his defense. When the accused files such motion to dismiss
without express leave of court, he waives the right to present evidence and
submits the case for judgment on the basis of the evidence for the
prosecution. (Rules of Court.)

3. Right to free legal assistance

Art. III, Sec. 11. Free access to the courts and quasi-judicial bodies and
adequate legal assistance shall not be denied to any person by reason of
poverty.


People v. Rio, 201 SCRA 702 (1991)

F: On 29 December 1989, the accused-appellant Ricardo Rio, in two (2)
letters dated 14 December 1989, addressed to Division Clerk of Court Fermin J.
Garma and to Assistant Clerk of Court Tomasita M. Dris, manifested his intention
to withdraw the appeal due to his poverty.

Paraphrasing Mr. Justice Malcolm, "Two (2) of the basic privileges of the
accused in a criminal prosecution are the right to the assistance of counsel and
the right to a preliminary examination. President Mckinley made the first a part
of the Organic Law in his Instructions to the Commission by imposing the
inviolable rule that in all criminal prosecutions the accused 'shall enjoy the right
... to have assistance of counsel for the defense' ". Today said right is enshrined
in the 1987 Constitution for, as Judge Cooley says, this is "perhaps the privilege
most important to the person accused of crime."
"In criminal cases there can be no fair hearing unless the accused be given
an opportunity to be heard by counsel. The right to be heard would be of little
meaning if it does not include the right to be heard by counsel. Even the most
intelligent or educated man may have no skill in the science of the law, particu-
larly in the rules of procedure, and, without counsel, he may be convicted not
because he is guilty but because he does not know how to establish his
innocence. And this can happen more easily to persons who are ignorant or
uneducated. It is for this reason that the right to be assisted by counsel is
deemed so important that it has become a constitutional right and it is so
implemented that under our rules of procedure it is not enough for the Court to
apprise an accused of his right to have an attorney, it is not enough to ask him
whether he desires the aid of an attorney, but it is essential that the court
should assign one de oficio for him if he so desires and he is poor, or grant him a
reasonable time to procure an attorney of his own."
This right to a counsel de oficio does not cease upon the conviction of an
accused by a trial court. It continues, even during appeal, such that the duty of
the court to assign a counsel de oficio persists where an accused interposes an
intent to appeal. Even in a case, such as the one at bar, where the accused had
signified his intent to withdraw his appeal, the court is required to inquire into
the reason for the withdrawal. Where it finds the sole reason for the withdrawal
to be poverty, as in this case, the court must assign a counsel de oficio, for
despite such withdrawal, the duty to protect the rights of the accused subsists
and perhaps, with greater reason. After all, "those who have less in life must
have more in law." Justice should never be limited to those who have the
means. It is for everyone, whether rich or poor. Its scales should always be
balanced and should never equivocate or cogitate in order to favor one party
over another.
It is with this thought in mind that we charge clerks of court of trial courts
to be more circumspect with the duty imposed on them by law (Section 13, Rule
122 of the Rules of Court) so that courts will be above reproach and that never
CONSTI TUTI ONAL LAW 2 ZPG & ASSOCI ATES

(if possible) will an innocent person be sentenced for a crime he has not
committed nor the guilty allowed to go scot-free.
In this spirit, the Court ordered the appointment of a counsel de oficio for
the accused-appellant and for said counsel and the Solicitor General to file their
respective briefs, upon submission of which the case would be deemed
submitted for decision.

From the records of the case, it is established that the accused-
appellant was charged with the crime of rape in a verified complaint filed by
complainant Wilma Phua Rio, duly subscribed before 3rd Assistant Fiscal Rodolfo
M. Alejandro of the province of Rizal, which reads as follows:
That on or about the 24th day of March, 1984, in the Municipality of
Muntinlupa, Metro Manila, Philippines, a place within the jurisdiction of this
Honorable Court, the above-named accused, by means of force and intimidation
did then and there wilfully, unlawfully and feloniously have carnal knowledge of
the undersigned Wilma Phua against her will.
On 26 June 1985, at the arraignment, the accused-appellant, assisted by
Atty. Leonido Manalo of the Makati CLAO office, as counsel de oficio, entered a
plea of not guilty to the offense charged.
xxx
The trial court found the accused-appellant guilty of the crime of rape.

The theory of the defense at the trial level was grounded on alibi. The
accused claimed that at the time of the alleged commission of the crime of rape
he was in Romblon. This claim was corroborated by the accused's brother,
Amado Rio. However, this claim was, as aforestated, rebutted by the
prosecution's submission of the voter's affidavit executed by the accused in
Muntinlupa, Metro Manila on 31 March 1984 when appellant claimed he was in
Romblon.

HELD: On appeal, appellant's counsel de oficio changed the theory of the
defense. The new theory presented by counsel de oficio is that Wilma Phua
consented when accused-appellant had sexual intercourse with her on 24 March
1984. It was stressed by counsel de oficio that the rape occurred on 24 March
1984 and that, allegedly, it was the fourth time accused had abused
complainant. This allegation as well as the fact that complainant failed to lock
the door to the bathroom could only have been due to the fact that there was
consent. The charge was filed, according to defense counsel de oficio, only
because the complainant's mother caught them.
This theory of the defense on appeal that there had been consent from
the complainant, fails to generate doubt as to the accused's guilt, for it would be
an incredulous situation indeed to believe that one, so young and as yet
uninitiated to the ways of the world, would permit the occurrence of an
incestuous relationship with an uncle, a brother of her very own mother. The
Court notes the sudden swift in the theory of the defense from one of total
denial of the incident in question, by way of alibi, to one of participation, that is,
with the alleged consent of the complainant. This new version could only be
attributed by the Court to the fact that counsel on appeal is different from the
counsel in the trial court. Although the Solicitor General has suggested that this
sudden shift be interpreted as an afterthought by the accused or a desperate
effort to get himself acquitted, the Court deems it more likely that this shift was
caused by counsel de oficio's preparation of the appellant's brief without
examining the entire records of the case. If the appointed counsel for the
accused, on appeal, had read the records and transcripts of the case thoroughly,
he would not have changed the theory of the defense for such a shift can never
speak well of the credibility of the defense. Moreover, the rule in civil procedure,
which applies equally in criminal cases, is that a party may not shift his theory on
appeal. If the counsel de oficio had been more conscientious, he would have
known that the sudden shift would be violative of aforementioned procedural
rule and detrimental to the cause of the accused-appellant (his client).
The Court hereby admonishes members of the Bar to be more conscious of
their duties as advocates of their clients' causes, whether acting de parte or de
oficio, for "public interest requires that an attorney exert his best efforts and
ability in the prosecution or defense of his client's cause." Lawyers are an
indispensable part of the whole system of administering justice in this
jurisdiction. And a lawyer who performs that duty with diligence and candor not
only protects the interests of his client; he also serves the ends of justice, does
honor to the Bar and helps maintain the respect of the community to the legal
profession. This is so because the entrusted privilege to practice law carries with
it correlative duties not only to the client but also to the court, to the bar and to
the public.
While a lawyer is not supposed to know all the laws, he is expected to take
such reasonable precaution in the discharge of his duty to his client and for his
professional guidance as will not make him, who is sworn to uphold the law, a
transgressor of its precepts.
The fact that he merely volunteered his services or the circumstance that
he was a counsel de oficio neither diminishes nor alters the degree of
professional responsibility owed to his client. The ethics of the profession require
that counsel display warm zeal and great dedication to duty irrespective of the
client's capacity to pay him his fees. Any attempted presentation of a case
without adequate preparation distracts the administration of justice and
discredits the Bar.

4. Right to be informed of nature and cause of accusation

The arraignment in criminal prosecution is precisely intended to comply
with the right of the accused to be informed of the nature and cause of the
accusation against him. As noted in Vera v. People, procedural due process
requires that the accused must be informed why he is being prosecuted and
what charge he must meet.

Borja v. Mendoza, 77 SCRA 422 (1977)

No valid trial in absentia without arraignment

F: Petitioner was accused of slight physical injuries in the City Court of Cebu.
After one postponement due to petitioner's failure to appear, the case was
reset. Again, petitioner failed to appear, despite notice to his bondsman. The
court then allowed the prosecution to present evidence despite the fact that
petitioner had not been arraigned. After the offended party had testified and
presented documentary evidence, the court found petitioner guilty. The CFI
affirmed the decision. Hence, this petition for certiorari.

HELD: Respondent Judge committed a grave abuse of discretion and his decision
is void. Because petitioner was not arraigned, he was not informed of the nature
and cause of accusation against him. Arraignment is an indispensable
requirement in any criminal proceeding.

5. Right to speedy, impartial and public trial

(1) Speedy Trial

The right to a speedy trial means one that is free from vexatious and
oppressive delays. Its objective is to free the innocent person from anxiety and
expense of a court litigation, or otherwise, to have his guilt determined within
the shortest possible time, compatible with the presentation and consideration
of whatever legitimate defense the accused may interpose.

While reasonable delay may be allowed as determined on a case to case
basis, an unreasonable delay on the part of the prosecution to present its case,
thereby causing the threat of penal liability to remain hanging over the head of
the accused for an extended period of time, violates the right of the accused to a
speedy trial.

The remedy of the accused in this case is habeas corpus if he has been
restrained of his liberty, or certiorari, prohibition or mandamus for the final
dismissal of the case; and dismissal based on the denial of the right to speedy
trial amounts to an acquittal.

So said the SC in Acevedo v. Sarmiento, 36 SCRA 247 (1970), a case
involving the prosecution for damage to property through reckless imprudence
which had been pending for 6 years, the last step taken being the start of the
cross-examination of the complaining witness, who did not appear thereafter.
The SC ordered the case dismissed with prejudice, thus acquitting the accused.

(2) Public Trial

A public trial does not require that the entire public can witness the trial.
It is enough if it is conducted at a place where one's relatives and friends can be
accommodated and the public may know what is going on.

The right is not absolute. The court can order the public out of the trial
room in the interest of morality and order.

In Garcia v. Domingo, 52 SCRA 143 (1970), the SC dismissed the
contention of one party that the trial was conducted inside the chamber of the
judge on the ground that the objection came too late (the party only complained
after the 14th hearing) and that the place was agreed upon by the parties for
their mutual convenience (the judge's room was air conditioned).


Garcia v. Domingo, 52 SCRA 143 (1970)

The pivotal question in this petition for certiorari and prohibition, one which
thus far has remained unresolved, is the meaning to be accorded the
constitutional right to public trial.

Issue: Is the holding of trial in the chambers of the judge violative of the right to
a public trial?

Ruling: NO
The defendants in this case agreed that the hearings be held in the
chambers. On fourteen separate occasions this was the case and there was no
objection on their part. There was no evidence to substantiate the claim that
any other person was excluded from the chambers. It is thus evident that what
took place in the chambers of the city court judge was devoid of haste or
intentional secrecy.
The trial must be public. It possesses that character when anyone
interested in observing the manner a judge conducts the proceedings in his
courtroom may do so. There is to be no ban on such attendance. His being a
CONSTI TUTI ONAL LAW 2 ZPG & ASSOCI ATES

stranger to the litigants is of no moment. No relationship to the parties need be
shown. The thought that lies behind this safeguard is the belief that thereby the
accused is afforded further protection, that his trial is likely to be conducted with
regularity and not tainted with any impropriety. It is not amiss to recall that
Delegate Laurel in his terse summation the importance of this right singled out
its being a deterrence to arbitrariness. It is thus understandable why such a right
is deemed embraced in procedural due process. Where a trial takes place, as is
quite usual, in the courtroom and a calendar of what cases are to be heard is
posted, no problem arises. It the usual course of events that individuals desirous
of being present are free to do so. There is the well recognized exception though
that warrants the exclusion of the public where the evidence may be
characterized as "offensive to decency or public morals."
What did occasion difficulty in this suit was that for the convenience of
the parties, and of the city court Judge, it was in the latter's air-conditioned
chambers that the trial was held. Did that suffice to investigate the proceedings
as violative of this right? The answer must be in the negative. There is no
showing that the public was thereby excluded. It is to be admitted that the size
of the room allotted the Judge would reduce the number of those who could be
present. Such a fact though is not indicative of any transgression of this right.
Courtrooms are not of uniform dimensions. Some are smaller than others.
Moreover, as admitted by Justice Black in his masterly In re Oliver opinion, it
suffices to satisfy the requirement of a trial being public if the accused could
"have his friends, relatives and counsel present, no matter with what offense he
may be charged."
Then, too, reference may also be made to the undisputed fact at least
fourteen hearings had been held in chambers of the city court Judge, without
objection on the part of respondent policemen. xxx

(3) Impartial trial

One aspect of an impartial trial is a neutral magistrate who exercises cold
impartiality.

In Tumey v. Ohio, 273 U.S. 510 (1927), it was held that a town mayor who
was paid on the basis of the fine he imposes for every conviction for violation of
the drinking laws, could not be an impartial judge. Under such a situation, he
would be interested in convicting those he tries so he would earn more.

Another aspect of an impartial trial is an impartial tribunal bound by the
Bill of Rights and the strict rules of evidence and procedure.

In Olaguer v. Military Commission, 150 SCRA 144 (1987), the SC held that
a civilian cannot be tried by a military court (in connection with the Light a Fire
Movement) so long as the civil courts are open and operating, even during
Martial Law.

6. Right to confront witness

The purpose of this right is to enable the accused to test the credibility of
the witness. The best means of confrontation is the process of cross-
examination.

7. Right to secure attendance of witnesses (and the production of evidence in
his behalf)

There are various means available to the parties to compel the
attendance of witnesses and the production of documents and things needed in
the prosecution or defense of a case in an adversarial manner: subpoena and
subpoena duces tecum: depositions and other modes of discovery; perpetuation
of testimonies.

8. Trial in Absentia

Although the right to be present is not explicit in the provision, it is
inferrable from the phrase "trial may proceed notwithstanding the absence of
the accused"

This right to be present may, however, be waived by the accused. Rule
115, sec, 1(c), talks of 3 ways that the waiver may take place: (a) express waiver
pursuant to the stipulations set forth in his bail bond, unless his presence is
specifically ordered by the court for purposes of identification; (b) implied
waiver when the accused without any justifiable cause is absent at the trial on a
particular date of which he had notice; and (c) implied waiver when the accused
under custody who had been notified of the date of trial escapes.
In cases in which there have been a waiver of the right to be present,
whether expressed or implied, the trial may be held "in absentia". The
requisites of a valid trial in absentia are: (i) the accused has been arraigned; (ii)
he was duly notified of the hearing; and (iii) his failure to attend the trial is
unjustified.

There can be no valid trial in absentia unless the accused has been
arraigned, ruled the SC in Boria v. Mendoza, 77 SCRA 422 (1977), a case involving
a charge for slight physical injuries where the accused failed to appear and so
the trial court allowed the prosecution to present its evidence even if the
accused has not yet been arraigned. Arraignment is crucial because it informs
the accued of the nature and cause of the accusation against him. Conviction
without arraignment violates due process and ousts the court of its jurisdiction.

Boria v. Mendoza, 77 SCRA 422 (1977), supra.

HELD: The subsequent trial in absentia deprived petitioner of his right to be
heard by himself and counsel. The indispensable requirement for trial in
absentia is that it should come after arraignment. VV.


Waiver of the right to be present implies also waiver of the right to
present evidence. Thus, if the accused fails to attend trial (which presupposes
arraignment), without any justifiable cause, the prosecution can proceed with
the presentation of the evidence, and thereupon, the court may consider the
case submitted for decision. The court will decide the case on the basis only of
the prosecution's evidence. This does not violate the constitutional presumption
of innocence because it does not mean that the judgment of the trial court will
result in conviction.

So ruled the SC in People v. Salas, 143 SCRA 163 (1986), which further
ruled that trial in absentia applies even to capital cases.


People v. Salas 143 SCRA 163 (1986)

Trial in absentia applies even to capital cases

F: Mario Abong was originally charged with homicide in the CFI Cebu but
before he could be arraigned, the case was reinvestigated on motion of the
prosecution. As a result of the reinvestigation, an amended information was
filed, with no bail recommended, to which he pleaded not guilty. Trial
commenced but while it was in progress, the prisoner took advantage of the first
information filed and succeeded in deceiving the city court of Cebu into granting
him bail and ordering his release. The respondent Judge, learning of the trickery,
cancelled the illegal bail bond and ordered Abong's re-arrest. But he was gone.
Nonetheless, the prosecution moved that the hearing continue in accordance
with the constitutional provision authorizing trial in absentia. The respondent
Judge denied the motion and suspended all proceedings until the return of the
accused. Hence, this petition.

HELD: The doctrine laid down in People v. Avancea has been modified by Art.
IV, sec. 19 [now Art. III, sec. 14(2) of the 1987 Constitution] which allows trial in
absentia. The prisoner cannot by simply escaping thwart his continued
prosecution and possible eventual conviction provided only that (a) he has been
arraigned; (b) he has been duly notified of the trial; and (c) his failure to appear
is unjustified. The right to be present at one's trial may now be waived except
only at that stage where the prosecution intends to present witnesses who will
identify the accused. The defendant's escape will be considered a waiver of this
right and the inability of the court to notify him of the subsequent hearings will
not prevent it from continuing with his trial. VV.


Trial in absentia was introduced only in the 1973 Constitution to remedy a
situation in which criminal prosecution could not move because the accused has
either escaped or jumped bail.

In People v. Prieto, 84 SCRA 198 (1978), the SC ruled that trial in absentia
does not justify the accused to jump bail. Just because th Constitution allows
trial in absentia does not mean that the accused is now free to waive his right to
be present during the trial. If he does, he runds the risk of having his bail bond
forfeited.

Provision for trial in absentia not a justification for jumping bail

F: For repeated failure of the accused Dario Gamayon to appear, respondent
Judge declared the bail bond forfeited and required the bondsmen to produce
the accused within thirty days and to show cause why no judgment should be
rendered against them. However, on motion of defense counsel, who invoked
the last sentence of Art. IV, section 19 [now Art. III, sec. 14(2)] on trial in
absentia, respondent Judge reconsidered his order. He argued that "if trial could
be conducted after the accused has been arraigned and identified, the
conclusion is inescapable that issuing an order of forfeiture of the bail bond is
premature." The prosecution filed a petition for certiorari.

HELD: The innovation introduced by the present Constitution goes no further
than to enable a judge to continue with the trial even if the accused is not
present under the conditions therein specified. It does not give the accused the
right to jump bail. VV.

Gimenez v. Nazareno, 160 SCRA 1 (1988)

In trial in absentia accused waives the right to present evidence and confront
witnesses

CONSTI TUTI ONAL LAW 2 ZPG & ASSOCI ATES

F: Teodoro dela Vega Jr., together with five others, was charged with
murder. After arraignment, during which he pleaded not guilty, the case was set
for hearing on Sept. 18, 1973 but he escaped. He was tried in absentia. The trial
court rendered judgment dismissing the case against his co-accused but it held
in abeyance the proceedings against him in order to give him the chance to cross
examine the witnesses against him and present evidence. Hence, this petition
for certiorari.

HELD: Was the jurisdiction lost when the accused escaped from the custody of
the law and failed to appear during the trial? No. As we have consistently ruled,
jurisdiction once acquired is not lost upon the instance of parties but continues
until the case is terminated. The lower court was correct in proceeding with the
reception of evidence but it erred when is suspended the proceedings as to the
respondent. The court need not wait for the time until the accused finally
decides to appear. To allow this delay is to render ineffective the constitutional
provision on trial in absentia.

9. When presence of the accused is a DUTY

In People v. Avancena, 32 O.G. 713, the SC held that (a) the accused has
the right to be present during trial; (b) if he is in the custody of the law,
presence in all stage is likewise a duty during (i) arraignment, (ii) entering a plea,
and (iii) promulgation of judgment. This rule however has been modified.

As things stand, the following are the rules:

1. Generally, the accused has the right to be present at all stages the trial
(from arraignment to rendition of judgment).

2. If the accused is in the custody of the law, his presence during the trial
is a duty only if the court orders his presence to enable the prosecution
witnesses to identify him. (People v. Salas, infra. reiterating Aquino v. Military
Commiission, infra. modifying People v. Avancena, infra.)

3. Although the accused is not in the custody of the law (and more so if
he is in the custody of the law), his presence is required in the following cases:

a) Arraignment, regardless of the offense;

b) Entering a plea, regardless of whether the plea is guilty or
not guilty.

c) Promulgation of judgment, except that when the judgment
is for a light offense, he may be represented by his counsel or a
personal emissary.



a. Arraignment and plea, whether of innocence or of guilt

Rule 116, Sec. 1. Arraignment and plea; how made.--
xxx
(b) The accused must be present at the arraignment and must
personally enter his plea. Both arraignment and plea shall be made of record,
but a failure to enter of record shall not affect the validity of the proceedings.


b. During trial, for identification

People v. Salas, 143 SCRA 163 (1986), supra.

HELD: The right to be present at one's trial may now be waived except
only at that stage where the prosecution intends to present witnesses
who will identify the accused.

c. Promulgation of sentence, unless it is for a light offense, in which case
accused may appear by counsel, or a representative (Rule 120, Sec. 6.)

E. Priviledge against self incrimination

Art. III, Sec. 17. No person shall be compelled to be a witness against
himself.

Any confession or admission obtained in violation of section 17 hereof
shall be inadmissible in evidence against him. [Art. III, Sec. 12 (3)]

1. Scope of privilege: Compulsory Testimonial self-incrimination

The privilege covers only testimonial incrimination obtained
compulsorily. It refers therefore to the use of the mental process and the
communicative faculties, and not to a merely physical activity. If the act is
physical or mechanical, the accused can be compelled to allow or perform the
act, and the result can be used in evidence against him.

Thus the accused can be required to allow a sample of a substance taken
from his body (U.S. v. Tan Teng. 23, Phil. 145 (1912)).

F: This defendant was charged with the crime of rape. He was found guilty
of the charge. He appeals the decision on the ground that the lower court erred
in admitting the testimony of the physicians about having taken a certain
substance from the body of the accused while he was confined in jail and
regarding the chemical analysis made of the substance to demonstrate the
physical condition of the accused with reference to a venereal disease. It was
discovered that the rape victim was infected by venereal disease so that the
finding of venereal disease in the accused was material to his conviction.
Upon this information the defendant was arrested and taken to the
police station and stripped of his clothing and examined. The policeman who
examined the defendant swore from the venereal disease known as gonorrhea.
The policeman took a portion of the substance emitting from the body of the
defendant and turned it over to the Bureau of Science for the purpose of having
a scientific analysis made of the same. The result of the examination showed
that the defendant was suffering from gonorrhea.

Issue: Whether or not the information that the accused has gonorrhea may be
used against him

Ruling: YES. The accused was not compelled to make any admissions or answer
any questions, and the mere fact that an object found on his person was
examined: seems no more to infringe the rule invoked, than would the
introduction in evidence of stolen property taken from the person of a thief.
The substance was taken from the body of the defendant without his
objection, the examination was made by competent medical authority and the
result showed that the defendant was suffering from said disease. As was
suggested by Judge Lobingier, had the defendant been found with stolen
property upon his person, there certainly could have been no question had the
stolen property been taken for the purpose of using the same as evidence
against him. So also if the clothing which he wore, by reason of blood stains or
otherwise, had furnished evidence of the commission of a crime, there certainly
could have been no objection to taking such for the purpose of using the same
as proof. No one would think of even suggesting that stolen property and the
clothing in the case indicated, taken from the defendant, could not be used
against him as evidence, without violating the rule that a person shall not be
required to give testimony against himself.
But the prohibition of compelling a man in a criminal court to be a witness
against himself, is a prohibition of the use of physical or moral compulsion, to
extort communications from him, not an exclusion of his body as evidence,
when it may be material. The objection, in principle, would forbid a jury (court)
to look at a person and compare his features with a photograph in proof.
Moreover we are not considering how far a court would go in compelling a man
to exhibit himself, for when he is exhibited, whether voluntarily or by order,
even if the order goes too far, the evidence if material, is competent.
The prohibition contained in section 5 of the Philippine Bill that a person
shall not be compelled to be a witness against himself, is simply a prohibition
against legal process to extract from the defendant's own lips, against his will, an
admission of his guilt.
Mr. Wigmore, in his valuable work on evidence, in discussing the question
before us, said:
If, in other words, it (the rule) created inviolability not only for his
[physical control] in whatever form exercised, then it would be possible for a
guilty person to shut himself up in his house, with all the tools and indicia of his
crime, and defy the authority of the law to employ in evidence anything that
might be obtained by forcibly overthrowing his possession and compelling the
surrender of the evidential articles a clear reductio ad absurdum. In other
words, it is not merely compulsion that is the kernel of the privilege, . . . but
testimonial compulsion. (4 Wigmore, sec. 2263.)
The main purpose of the provision of the Philippine Bill is to prohibit
compulsory oral examination of prisoners before trial. or upon trial, for the
purpose of extorting unwilling confessions or declarations implicating them in
the commission of a crime. (People vs. Gardner, 144 N. Y., 119.)
The doctrine contended for by appellant would prohibit courts from
looking at the fact of a defendant even, for the purpose of disclosing his identity.
Such an application of the prohibition under discussion certainly could not be
permitted. Such an inspection of the bodily features by the court or by
witnesses, can not violate the privilege granted under the Philippine Bill,
because it does not call upon the accused as a witness it does not call upon the
defendant for his testimonial responsibility. Mr. Wigmore says that evidence
obtained in this way from the accused, is not testimony but his body his body
itself.


The accused can be ordered to expel the morphine from his mouth (U.S.
v. Ong Sio Hong 36 Phil 735, (1917)).

U.S. v. Ong Sio Hong 36 Phil 735, (1917)

Counsel for appellant raises the constitutional question that the accused
was compelled to be a witness against himself. The contention is that this was
the result of forcing the accused to discharge the morphine from his mouth. To
force a prohibited drug from the person of an accused is along the same line as
CONSTI TUTI ONAL LAW 2 ZPG & ASSOCI ATES

requiring him to exhibit himself before the court; or putting in evidence papers
and other articles taken from the room of an accused in his absence; or, as in the
Tan Teng case, taking a substance from the body of the accused to be used in
proving his guilt. It would be a forced construction of the paragraph of the
Philippine Bill of Rights in question to hold that any article, substance, or thing
taken from a person accused of crime could not be given in evidence. The main
purpose of this constitutional provision is to prohibit testimonial compulsion by
oral examination in order to extort unwilling confessions from prisoners
implicating them in the commission of a crime. (Harris vs. Coats [1885], 75 Ga.,
415.)


The accused can be made to take off her garments and shoes and be
photographed. (People v. Otadura, 96 Phil 244 (1950)).


A woman accused of adultery can be compelled to show her body for
physical investigation to see if she is pregnant (Villaflor v. Summers, 41 Phil. 62
(1920)). Viewed against present standards, however, it is possible that this
method of determining pregnancy would violate due process as being too
barbaric.

Villaflor v. Summers, 41 Phil. 62 (1920)

F: The facts are not dispute. In a criminal case pending before the Court of
First Instance of the city of Manila, Emeteria Villaflor and Florentino Souingco
are charged with the crime of adultery. The court ordered the defendant
Emeteria Villaflor, to submit her body to the examination of one or two
competent doctors to determine if she was pregnant or not. The accused
refused to obey the order on the ground that such examination of her person
was a violation of the constitutional provision relating to self-incrimination.
Thereupon she was found in contempt of court and was ordered to be
committed to Bilibid Prison until she should permit the medical examination
required by the court.

Issue: Whether the compelling of a woman to permit her body to be examined
by physicians to determine if she is pregnant, violates that portion of the
Philippine Bill of Rights

Ruling: The constitutional guaranty, that no person shall be compelled in any
criminal case to be a witness against himself, is limited to a prohibition against
compulsory testimonial self-incrimination. The corollary to the proposition is
that, an ocular inspection of the body of the accused is permissible. The proviso
is that torture of force shall be avoided. Whether facts fall within or without the
rule with its corollary and proviso must, of course, be decided as cases arise.
It is a reasonable presumption that in an examination by reputable and
disinterested physicians due care will be taken not to use violence and not to
embarass the patient any more than is absolutely necessary. Indeed, no
objection to the physical examination being made by the family doctor of the
accused or by doctor of the same sex can be seen.

The taking of footprint sample to see if it matches the ones found in the
scene of the crime is allowed (People v. Salas and People v. Sara).

However, making the accused take dictation to get a specimen of her
handwriting is not allowed, for this involves the use of the mental process.
[Bermudez v. Castillo, 64 Phil. 485 (1937).]

Bermudez v. Castillo, 64 Phil. 485 (1937)

F: In connection with this administrative case, said respondent filed, six
letters which, for purposes of identification, were marked as Exhibits 32, 34, 35,
36 and 37. He contends that said six letters are the complainant's, but the latter
denied it while she was testifying as a witness in rebuttal.
Respondent required complainant to copy the letters in her own
handwriting in the presence of the investigator. The complainant, refused
invoking her right not to incriminate herself. The investigator, upholding the
complainant, did not compel her to submit to the trial required, thereby denying
the respondent's petition.

Issue: Whether or not the complainant may be forced to make a copy of the
letters in her own handwriting

Ruling: No. It would violate her right against self- incrimination.
The constitution provides: "No person shall be compelled to be a witness
against himself." It should be noted that before it was attempted to require the
complainant to copy the six documents above-stated, she had sworn to tell the
truth before the investigator authorized to receive statements under oath, and
under said oath she asserted that the documents in question had not been
written by her. Were she compelled to write and were it proven by means of
what she might write later that said documents had really been written by her, it
would be impossible for her to evade prosecution for perjury.
The reason for the privilege appears evident. The purpose thereof is
positively to avoid and prohibit thereby the repetition and recurrence of the
certainly inhuman procedure of compelling a person, in a criminal or any other
case, to furnish the missing evidence necessary for his conviction. If such is its
purpose, then the evidence must be sought elsewhere; and if it is desired to
discover evidence in the person himself, then he must be promised and assured
at least absolute immunity by one authorized to do so legally, or he should be
asked, one for all, to furnish such evidence voluntarily without any condition.
This court is of the opinion that in order that the constitutional provision under
consideration may prove to be a real protection and not a dead letter, it must be
given a liberal and broad interpretation favorable to the person invoking it.
In view of the foregoing consideration and holding, as it is hereby held,
that the complainant is perfectly entitled to the privilege invoked by her, the
respondent's petition is denied.

Also requiring the accused to reenact the crime is not allowed, for this
also involves the mental process.


People v. Olvis, 154 SCRA 525

F: Villarojo, Cademas and Sorela were convicted in the lower court of
murder for the death of Bagon. Olvis, the alleged principal by inducement, was
acquitted. The three accused were convicted on the basis of the extrajudicial
confessions executed by them in the presence of a counsel summoned by the
NBI to handle appellants' case, and the reenactment done by them of the
circumstances surrounding the killing.

RULING: The extrajudicial confessions are inadmissible. They were made in the
presence of a counsel summoned by the NBI and not of appellants' own choice.
He cannot therefore be said to have been acting on behalf of the accused when
he lent his presence at the confession proceedings.
But the accused were denied their right to counsel not once but twice
when they were forced to re-enact the crime. Forced re-enactments like
uncounselled and coerced confessions come within the ban against self-
incrimination. This constitutional privilege has been defined as a protection
against testimonial compulsion but this has since been extended to any evidence
communicative in nature acquired under circumstances of duress. Essentially,
the right is meant to avoid and prohibit positively the repetition and recurrence
of the certainly inhuman procedure of compelling a person, in a criminal or any
other case, to furnish the missing evidence necessary for his conviction.

People v. Go, 237 SCRA 73

F: After a buy-bust operation accused were arrested by the police. Upon
the presentation of a search warrant, the house of the accused was searched,
and several prohibited drugs were seized. They were charged with and
convicted of violation of the Dangerous Drugs law. They contended that they
had not been shown a search warrant. In concluding that a search warrant had
been presented to the accused prior to the search, the trial court relied on a
document entitiled Certificate of Re-conduct of Search, signed by the accused.

ISSUE: Whether or not such document is admissible in evidence.

RULING: IT CANNOT BE ADMITTED IN ITS ENTIRETY.
The second paragraph of the Certification amounts to an implied
admission that shabu, the marked money, and shabu papaphernalia had been
found by the police authorities at the residence of the Go spouses and therefore,
subject to the control and custody of the accused (the spouses) and necessarily
in their possession. To this extent, the Certification is a declaration against the
interest and tacit admission of the crime charged. The second paragraph of the
Certification is a self-incriminatory statment made at a time when the spouses
were not assisted by counsel and under circumstances (in the course of or
immediately after the search of the residence and seizure of quantities of shabu)
which render intelligent waiver of their right against self-incrimination open to
serious doubt.
The Court considers that there is nothing to prevent admission of the
Certification to substantiate the fact that a search warrant issued by a judge
had been brought to the attention of the spouses in the course of the raid or
buy-bust operation carried out at their residence and that in the course thereof,
no force or intimidation had been exercised upon the spouses.
Notwithstanding such, the accused were convicted of the crime charged
against them.

2. In what proceedings available

The privilege is available in any proceedings, even outside the court, for
they may eventually lead to a criminal prosecution.

In Pascual v. Board of Medical Examiners, 28 SCRA 344 (1969), the SC held
that the privilege against self-incrimination extends to administrative
proceedings which possess a criminal or penal aspect. In this case, it was held
that a doctor who was being investigated by a medical board for alleged
malpractice and would lose his license if found guilty, could not be compelled to
take the witness stand without his consent.

Pascual v. Board of Medical Examiners, 28 SCRA 344 (1969)

CONSTI TUTI ONAL LAW 2 ZPG & ASSOCI ATES

F: Arsenio Pascual, Jr., petitioner-appellee, filed on February 1, 1965 with
the Court of First Instance of Manila an action for prohibition with prayer for
preliminary injunction against the Board of Medical Examiners, now respondent-
appellant. It was alleged therein that at the initial hearing of an administrative
case for alleged immorality, counsel for complainants announced that he would
present as his first witness herein petitioner- appellee, who was the respondent
in such malpractice charge. Thereupon, petitioner-appellee, through counsel,
made of record his objection, relying on the constitutional right to be exempt
from being a witness against himself. Respondent-appellant, the Board of
Examiners, took note of such a plea, at the same time stating that at the next
scheduled hearing, on February 12, 1965, petitioner-appellee would be called
upon to testify as such witness, unless in the meantime he could secure a
restraining order from a competent authority.
A decision was rendered by the lower court on August 2, 1965, finding
the claim of petitioner-appellee to be well-founded and prohibiting respondent
Board "from compelling the petitioner to act and testify as a witness for the
complainant in said investigation without his consent and against himself."

HELD: Petitioner could suffer the revocation of his license as a medical
practitioner, for some an even greater deprivation.
Why it should be thus is not difficult to discern. The constitutional
guarantee, along with other rights granted an accused, stands for a belief that
while crime should not go unpunished and that the truth must be revealed, such
desirable objectives should not be accomplished according to means or methods
offensive to the high sense of respect accorded the human personality. More
and more in line with the democratic creed, the deference accorded an
individual even those suspected of the most heinous crimes is given due weight.
To quote from Chief Justice Warren, "the constitutional foundation underlying
the privilege is the respect a government ... must accord to the dignity and
integrity of its citizens."
Thus according to Justice Douglas: "The Fifth Amendment in its Self-
Incrimination clause enables the citizen to create a zone of privacy which
government may not force to surrender to his detriment." So also with the
observation of the late Judge Frank who spoke of "a right to a private enclave
where he may lead a private life. That right is the hallmark of our democracy."
In the light of the above, it could thus clearly appear that no possible objection
could be legitimately raised against the correctness of the decision now on
appeal. We hold that in an administrative hearing against a medical practitioner
for alleged malpractice, respondent Board of Medical Examiners cannot,
consistently with the self-incrimination clause, compel the person proceeded
against to take the witness stand without his consent.

In Galman v. Pamaran, infra, the privilege was held to extend to fact-
finding investigation by an adhoc body.


Galman v. Pamaran, 138 SCRA 274 (1985)

A person can be compelled to testify provided he is given immunity co-extensive
with the privilege against self- incrimination

F: The respondents led by General Fabian Ver and Major General Prospero
Olivas testified before the Agrava Board looking into the killing of former
Senator Benigno Aquino. They were subsequently accused of murder in two
cases for the killing of Sen. Aquino and Rolando Galman. They were charged as
accessories in both. The prosecution offered in evidence the testimony of Ver
and Olivas before the Agrava Board, but on the latter's objections, the
Sandiganbayan excluded the testimony. The private and public prosecutions
filed petitions for certiorari.

HELD: The persons summoned to testify before the Agrava Board were "under
investigation for the commission of the offense" within the meaning of Art. III,
sec. 12. It is to be noted that the framers of the Constitution did not adopt the
Miranda reference to "custodial investigation." The subject matter dealt with
and the questioning before the Agrava Board indubitably evinced purposes
other than merely determining the surrounding facts and circumstances of the
assassination. The respondents were called to determine their probable
involvement in the crime. Yet they were not informed or at the very least
warned of their right to remain silent and that any statement given by them may
be used against them. The first portion of Sec. 5 of PD 1886 denied them the
right to remain silent, and gave power to the Board to punish refusal to testify.
The SC said it is not satisfied that when they testified they waived their
constitutional right not be compelled to be a witness against themselves, much
less their right to remain silent. The SC also said it cannot be contended that the
privilege against self- incrimination applies only to criminal prosecutions. Art. III,
sec. 17 of the Const. provides that "No person shall be compelled to be a witness
against himself."



Compare People v. Ayson, 175 SCRA 216 (1989), supra.

3. "Use and Fruit Immunity" v. "Transactional Immunity"

When the State requires testimony to be made before a board or body, it
has to grant immunity by means of law to the persons testifying, so as not to
violate their right against self-incriminatrion. This is the only way to reconcile
two conflicting values; public interest to get certain relevant information, say, to
legislation, that can only be supplied by the testimony of certain persons and the
highly primed constitutional right not to make a person a witness against
himself.

Through an immunity statute, the state in effect exchanges immunity for
the testimony of a witness. The problem concerns the extent of immunity that
the State must grant in order to protect the privilege against self-incrimination.

Transactional Immunity

In a transactional immunity, a person is given immunity from prosecution
of the crime in connection with which he gave his testimony. The immunity is
from the prosecution, not merely from the use of the testimony. Thus, even if
the guilt of the person testifying can be proven by independent means, he can
not be prosecuted anymore.

Use and Fruit Immunity

In a use and fruit immunity, a person is exempted from the use of his
testimony as well as the leads (fruits) that the testimony opened up in a criminal
prosecution arising from what he testified on. The immunity in this case is from
the testimony given. Thus, if the state can procure evidence, independent of the
testimony and its fruits, it can prosecute the person testifying nevertheless.


History in the United States

In Councilman v. Hitchcock (1892), the SC ruled that the only way to
respect the right against self-incrimination is to give transactional immunity;
anything less violates the constitutional right.

Thus, Congress in 1893 passed the Compulsory Testimony Act, providing
for transactional immunity.

In 1964, the U.S. SC in Murphy v. Waterfront Commission of New York
hinted that it was not really necessary to give transactional immunity in order to
protect the right against self incrimination.

This gave the U.S. Congress the cue to revise the Compulsory Testimony
Act and provide for a "use and fruit immunity".

With the validity of this limited immunity was raised, the SC in Castigas v.
U.S. and Zicarelli v. U.S. ruled that the right is amply protected by the use and
fruit immunity.


In the Philippines

There is no fixed rule in the Philippines. "Transactional immunity" can be
found in the following:

Art. XIII, Sec. 18. The Commission on Human Rights shall have the
following powers and functions
xxx
(8) Grant immunity from prosecution to any person whose testimony or
possession of documents or other evidence is necessary or convenient to
determine the truth in any investigation conducted by it or under its authority.

Use and Fruit Immunity

On the other hand, "use and fruit immunity can be found in P.D. 1886,
which created the Agrava Fact Finding Board, and which was the subject-matter
of Galman v. Pamaran, 138 SCRA 274 (1985).

In this case, Ver and other high-ranking AFP officials were made to testify
before the Agrava Board investigating the double murder of Sen. Aquino and
Galman. Under P.D.1886, every person summoned by the Board has to appear
and testify on pain of being held in contempt. Any testimony made, in turn, was
exempted from being "used" in a criminal prosecution. Despite this however, a
case was file against Ver in the Sandiganbayan, and one of the evidence
presented was the testimony he made before the Board. When objected to, the
Sandiganbayan sustained the objection. And so the matter was raised to the SC
on certiorari.

The SC held that the testimony could not be used in a subsequent
proceeding. it hinted that were it not for the provision in the decree
conmpelling attendance and testimony on pain of being held in contempt, the
accused could have invoked the right against self-incrimination. But since the
state needed the testimony, it gave them immunity and so now, the State must
honor its obligation and disallow the use of the testimony in the criminal
prosecution.
CONSTI TUTI ONAL LAW 2 ZPG & ASSOCI ATES


Galman v. Pamaran, 138 SCRA 274 (1985), supra.

HELD: Immunity statutes may be generally classified into two: one, which grants
"use immunity" and the other, which grants what is known as "transactional
immunity." The distinction between the two is: "Use immunity" prohibits use of
a witness' compelled testimony and its fruits in any manner in connection with
the criminal prosecution of the witness. On the other hand, "transactional
immunity" grants immunity to the witness from prosecution for an offense to
which his compelled testimony relates. PD 1886, sec. 5 grants merely immunity
from use of any statement given before the Board, but not immunity from
prosecution by reason or on the basis thereof. VV


4. Exclusionary rule

Art. III, Sec. 12. xxx
(3) Any confession or admission obtained in violationof this or Section
17 hereof shall be inadmissible in evidence against him.

The paradigmatic application of the exclusionary rule is a traditionally
coerced confession, and not so much on uncounselled statement. A fortiori,
testimony forced out of a person cannot be used in evidence against that
person.

5. Effect of denial of the privilege by court

When the privilege against self-incrimination is violated outside of court,
say, by the police, then the testimony, as already noted, is not admissible under
the exclusionary rule.

When the privilege is violated by the court itself, that is, by the judge, the
court is ousted of its jurisdiction, all its proceedings are null and void, and it is as
if no judgment has been rendered. A classic case is Chavez v. Court of Appeals,
34 SCRA 663 (1968).
Chavez v. Court of Appeals, 34 SCRA 663 (1968)

F: The thrust of petitioner's case presented in his original and
supplementary petitions invoking jurisdiction of this Court is that he is entitled,
on habeas corpus, to be freed from imprisonment upon the ground that in the
trial which resulted in his conviction he was denied his constitutional right not
to be compelled to testify against himself. There is his prayer, too, that, should
he fail in this, he be granted the alternative remedies of certiorari to strike down
the two resolutions of the Court of Appeals dismissing his appeal for failure to
file brief, and of mandamus to direct the said court to forward his appeal to this
Court for the reason that he was raising purely questions of law.
Accused Chavez was made to testify as a witness for the prosecution
without him being considered a state witness inspite of objections by his
counsel.
Roger Chavez was found guilty. The court had this to say: "Roger Chavez
does not offer any defense. As a matter of fact, his testimony as witness for the
prosecution establishes his guilt beyond reasonable doubt." The trial court
branded him "a self- confessed culprit".

Issue: Whether or not Chavez right against self-incrimination was violated

Ruling: YES
The right agianst self-incrimination is "not merely a formal technical rule
the enforcement of which is left to the discretion of the court"; it is mandatory;
it secures to a defendant a valuable and substantive right; it is fundamental to
our scheme of justice.
The constitutional proscription was established on broad grounds of
public policy and humanity; of policy because it would place the witness against
the strongest temptation to commit perjury, and of humanity because it would
be to extort a confession of truth by a kind of duress every species and degree of
which the law abhors.
Therefore, the court may not extract from a defendant's own lips and
against his will an admission of his guilt. Nor may a court as much as resort to
compulsory disclosure, directly or indirectly, of facts usable against him as a
confession of the crime or the tendency of which is to prove the commission of a
crime. Because, it is his right to forego testimony, to remain silent, unless he
chooses to take the witness stand with undiluted, unfettered exercise of his
own free, genuine will.
Compulsion as it is understood here does not necessarily connote the use
of violence; it may be the product of unintentional statements. Pressure which
operates to overbear his will, disable him from making a free and rational
choice, or impair his capacity for rational judgment would in our opinion be
sufficient. So is moral coercion "tending to force testimony from the unwilling
lips of the defendant."
Petitioner, as accused, occupies a different tier of protection from an
ordinary witness. Whereas an ordinary witness may be compelled to take the
witness stand and claim the privilege as each question requiring an incriminating
answer is shot at him, and accused may altogether refuse to take the witness
stand and refuse to answer any and all questions. For, in reality, the purpose of
calling an accused as a witness for the People would be to incriminate him.

xxx With all these, we have no hesitancy in saying that petitioner was
forced to testify to incriminate himself, in full breach of his constitutional right to
remain silent. It cannot be said now that he has waived his right. He did not
volunteer to take the stand and in his own defense; he did not offer himself as a
witness; on the contrary, he claimed the right upon being called to testify. If
petitioner nevertheless answered the questions inspite of his fear of being
accused of perjury or being put under contempt, this circumstance cannot be
counted against him. His testimony is not of his own choice. To him it was a case
of compelled submission. He was a cowed participant in proceedings before a
judge who possessed the power to put him under contempt had he chosen to
remain silent. Nor could he escape testifying. The court made it abundantly clear
that his testimony at least on direct examination would be taken right then and
thereon the first day of the trial.
The course which petitioner takes is correct. Habeas corpus is a high
prerogative writ. It is traditionally considered as an exceptional remedy to
release a person whose liberty is illegally restrained such as when the accused's
constitutional rights are disregarded. Such defect results in the absence or loss
of jurisdiction and therefore invalidates the trial and the consequent conviction
of the accused whose fundamental right was violated. That void judgment of
conviction may be challenged by collateral attack, which precisely is the function
of habeas corpus. Habeas corpus is proper to challenge a conviction where the
consitutional rights of the accused were violated.
A court which denies the accused of his constitutional rights is ousted of
its jurisdiction. The judgment of conviction pronounced by a court without
jurisdiction is void, and one imprisoned thereunder may obtain release of
habeas corpus.

Notes on the case: In this case, the accused Chavez was compelled by the
judge with the threat of being held in contempt to take the witness stand, in
spite of his objection that he had the right to remain silent and not to be a
witness against himself. And so he took the witness stand and was convicted by
qualified theft. He appealed but the lawyer failed to file the appellant's brief
and so the appeal was dismissed, the judgment became final and executory, and
he served his sentence. Years later, Chavez went to the SC on habeas corpus,
contending that his convictioin was void because it was rendered on the basis of
evidence obtained in the violation of his right against self- incrimination. The SC
granted the petition and released him.
Habeas Corpus, as shown by this case, is an extraordinary post-conviction,
mid-sentence, remedy. The petition for habeas corpus is such that it inquires
into all questions of illegal detention. When the judge compelled the accused to
take the witness stand, he was ousted of his jurisdiction and all subsequent
proceedings became void. Ultimately, the judgment of conviction and even the
sentence were likewise void, thus making the detention of Chavez illegal, and
thus actionable by habeas corpus.
The case also illustrates the difference between the ordinary witness and
the accused. A witness can be conmpelled to take the stand; he can only object
to the questions as they come, invoking his right against self-incrimination.
But in the case of the accused, he cannot even be made to take the
witness stand, for the only purpose of such is to incriminate him.
Of course, the moment the accused agrees to take the stand, he is
deemed to have waived his right, and must now thus submit himself to cross-
examination.

E. Right to an impartial tribunal and trial of civilians by military courts

Animas v. Minister of National Defense, 146 SCRA 406 (1986)

F: This petition challenges the jurisdiction of a military tribunal to try
twelve accused persons, only one of whom is in the military, for the offense
devoid of any national security or political complexion and committed long
before the proclamation of martial law,
The petitioners were charged with murder in connection with the
alleged killing of Yanson, a political leader,during the November 11 elections.
The accused were arrested almost a year later, on September 21, 1972
after martial law was proclaimed. It was only in 1974 that a "summary
preliminary investigation" was conducted by a PC captain belonging to the Judge
Advocate General Service. The petitioners were recommended for prosecution
before the Military Tribunal, considering that one of them, petitioner Sgt.
Rodolfo Animas is a military personnel. Thereafter, the Judge Advocate General
filed the corresponding charge sheet, but he modified the crime charged from
"Murder" to "Violation of Section 878 of the Revised Administrative Code" in
Relation to Section 2692 of the same Code and Presidential Decree No. 9, "
Illegal Possession of Firearms with Murder."
On February 16, 1978, the Minister of National Defense referred the
case to the Military Tribunal's Branch of the Judge Advocate General's Office
(JAGO) which in turn assigned the same to respondent Military Commission No.
27.

Issue: Whether or not Military Commission No. 27 is without jurisdiction over
the criminal case

Ruling: The military court is without jurisdiction.
We apply the rule in Rolando A. de Guzman v. Hon. Alejandro R.
Leopando, et al, (G.R. No. 62798, December 22, 1983 and March 13, 1984)
CONSTI TUTI ONAL LAW 2 ZPG & ASSOCI ATES

where the lone military personnel was ordered tried together with 19 civilians
accused before a civil court. It is also clear from the records that the acts for
which Sgt. Animas was charged had nothing to do with the performance of
official duty.
The crime for which the petitioners were charged was committed on
November 10, 1971 long before the proclamation of martial law. There was no
question about the case being prosecuted by civilian fiscals and tried by civil
courts at the time. Now that it is already late 1986, and martial law is a thing of
the past, hopefully never more to return, there is no more reason why a murder
committed in 1971 should still be retained, at this time, by a military tribunal.


Olaguer v. Military Commission No. 34, 150 SCRA 144

Military trial of civilians void even under Martial Law if the civil courts are open

F: Petitioners were found guilty of subversion by the respondent military
commission and sentenced to death. They filed a petition for habeas corpus,
certiorari, prohibition and mandamus before the SC, questioning the jurisdiction
of the military tribunal.

HELD: In Aquino v. Military Commission (1975), the SC held that "Martial law
creates and exception to the general rules of exclusive jurisdiction, and renders
offenses against the laws of war as well as those of a civil character, triable by
military tribunals.xxx" Due process, however demands that in all criminal cases
prosecutions, the accused shall be entitled to, among others, a trial. As
explained by Justice Teehankee in his dissenting opinion in Aquino v. Military
Commission supra: "Judicial power is vested by the Constitution exclusively in
the SC and insuch inferior courts as are established by law. Judicial power exists
only in the courts which have the exlcusive power to hear and determine those
matters which affect the life or liberty or property of a citizen." Since we are not
an enemy occupied territory and even on the premise that martial continues in
force, the military tribunals cannot try and exercise jurisdiction over civilians for
civil offenses committed by them which are properly cognizable by the civil
courts.
xxx
"The presiding officer at a court martial is not a judge whose objectivity
and independence are protected by tenure and undiminshed salary and
nurtured by the judicial tradition, but is a military officer. Substantially different
rules of evidence and procedure apply in military trials. Apart from these
differences, the suggestion of the possibility of influence on the actions of the
court-martial by the officer who convenes it, selects its members and the
counsel on both sides, and who usually has direct command and authority over
its members is a pervasive one in military laws, despite strenuous efforts to
eliminate the danger. VV.




Cruz v. Ponce-Enrile, 160 SCRA 702 (1988)

F: Habeas corpus proceedings were commenced in this Court on October 1,
1986 to test the legality of the continued detention of some 217 so-called
"political detainees arrested in the nine-year span of official martial rule and
committed to the New Bilibid Prisons in Muntinlupa. All had been made to stand
trial for common crimes before various courts martial; if any of these offenses
had any political color, this had neither been pleaded nor proved.
Of the 217 prisoners, 157 are civilians, and only 26 confirmed as
military personnel.

Issue: Whether or not military courts have jurisdiction over civilians

Ruling: No
As held in Olaguer: A military jurisdiction or tribunal cannot try and
exercise jurisdiction, even during the period of martial law, over civilians for
offenses allegedly committed by them as long as the civil courts are open and
functioning, and that any judgment rendered by such body relating to a civilian is
null and void for lack of jurisdiction on the part of the military tribunal
concerned
The fact cannot be ignored, however, that crimes appear to have been
committed, and there are accusations against herein petitioners for those
offenses. Olaguer cannot and does not operate to absolve the petitioners of
these charges, or establish that the same are baseless, so as to entitle them to
immediate release from detention. It is not to be forgotten that the victims in
offenses ascribed to the petitioners have as much interest as the State has to
prosecute the alleged authors of the misdeeds. Justice will be better served if
the detention of such of the petitioners as are not hereby ordered released or
excepted, is continued until their cases are transferred to the ordinary courts
having jurisdiction, and the necessary informations have been filed against them
therein, as has already been done in the case of petitioners Imperial D. Usman
and Samu Gumal. The State should be given a reasonable period of time to
accomplish this transfer, at which time the petitioners may apply for bail for
their temporary release.
The Solicitor General not unreasonably anticipates questions to arise as to
the availability of certain defenses to the petitioners upon their prosecution
before the civil courts. It seems evident, however, that no breach of the
constitutional prohibition against twice putting an accused in jeopardy of
punishment for the same offense would result from the retrial of the
petitioners" cases, for the simple reason that the absence of jurisdiction of the
courts martial to try and convict the petitioners prevented the first jeopardy
from attaching. Valid previous proceedings are required in order that the
defense of double jeopardy can be raised by the accused in the second
prosecution.

G. Bills of attainder-- Legislative adjudication of guilt

Bill of Attainder

A "bill of attainder" is a law which substitutes the legislative
determination of guilt for a judicial determination. Through a statute, the
legislature finds individuals or groups guilty, without the benefit of being proven
so in court.

A bill of attainder is of two kinds: (i) bill of attainder proper (legislative
imposition of the death penalty) and (ii) bill of pains and penalties (imposition of
a lesser penalty).

In People v. Ferrer, 48 SCRA 382 (1972), the Anti-Subversion Law (RA
1700) which declared the Communist Party of the Philippines a clear and present
danger to Philippine security, and thus prohibited membership in such
organization, was contended to be a bill of attainder. The SC, however,
dismissed the contention, holding that although the law mentions the CPP in
particular, its purpose is not to define a crime but only to lay a basis or to justify
the legislative determination that membership in such organization is a crime
because of the clear and present danger to national security.


People v. Ferrer, 48 SCRA 382 (1972)

F: Posed in issue in these two cases is the constitutionality of the Anti-
Subversion Act, which outlaws the Communist Party and other "subversive
associations", and punishes any person who "knowingly, willfully and by overt
acts affiliates himself, with, becomes or remains a member," of the Party and of
any other similar "subversive" organization.

ISSUE: W/N this law is a bill of attainder.

HELD: NO
A bill of attainder is a legislative act which inflicts punishment without
trial. Its essence is the substitution of a legislative for a judicial determination of
guilt. The constitutional ban against bill of attainder serves to implement the
principle of separation of powers by confining legislatures to rule-making and
thereby forestalling legislative usurpation of the judicial function.
When the Act is viewed in its actual operation, it will be seen that it does
not specify the Communist Party of the Phils (CPP) of the members thereof for
the purpose of punishment. What it does is simply to declare the Party to an
organized conspiracy for the overthrow of the Government for the purposes of
the prohibition against membersip in the outlawed organization. The term
"CPP" is used solely for definition purposes. In fact the Act applies not only to
the CPP but to "any other organizatuiion having the same purposes and their
successors". Its focus is not on individuals but on conduct.
Indeed, were the Anti-Subversion Act a bill of attainder, it would be
totally unnecessary to charge Communists in court, as the law alone, without
more would suffice to secure their punishement. But the undeniable fact is that
their guilt still has to be judicially established. The Government has yet to prove
at the trial that the accused joined the Party knowingly, willfully and by overt
acts, and that they joined with the specific intent to further its basic objectives.

H. Right to a speedy disposition of cases

Art. III, Sec. 16. All persons shall have the right to a speedy disposition
of their cases before all judicial, quasi-judicial, or administrative bodies.



The right to a speedy disposition of cases complements the right to a
speedy trial. After the case has been submitted for decision, so that technically
the trial stage is terminated, the Constitution mandates that the judicial, quasi-
judicial or administrative body or tribunal must decide the case consistent with
the right of the accused to a speedy disposition of his case.

To carry out this mandate, the Constitution in several other places
provides periods for deciding a case:

The Supreme Court has to decide cases within 24 months from the date of
submission of the case for decision which is the date of filing of the last pleading
[Art. VIII, Sec. 15 (1).]
III. SUBSTANTIVE RIGHTS UNDER THE DUE PROCESS CLAUSE


CONSTI TUTI ONAL LAW 2 ZPG & ASSOCI ATES

A. What acts cannot be criminalized

1. Mere beliefs and aspirations

Art. III, Sec. 18. (1) No person shall be detained solely by reason of his
political beliefs and aspirations.


2. Debts and civil obligations

Art. III. Sec. 20. No person shall be imprisoned for debt or non-payment
of a poll tax.


What the law prohibits is imprisonment for non-payment of a contractual
obligation.

When one is convicted of estafa and sent to prison, the imprisonment is
not for the non- payment of debt but for the deceit or abuse of confidence
employed by the convict.

Thus, in Lozano v. Martinez, 146 SCRA 123 (1986), the SC again upheld
Batas Blg. 22 (Bouncing Checks Law) as not unconsitutional for being violative of
the rule against non- imprisonment for debt. It is true that under this law deceit
is not necessary. It is, however, a valid exercise of the State of its power to
determine what acts constitute a crime.

What the Consitution further prohibits is imprisonment for non-payment
of poll tax, which is a tax imposed on certain persons regardless of their property
or business. The prohibition does not apply to non-payment of property taxes
and taxes on privilege.


Lozano v. Martinez, 146 SCRA 323 (1986)

F: BP 22 punishes any person "who makes or draws and issues any
check on account or for value, knowing at the time of issue that he does
not have sufficient funds in or credit with the drawee bank for the
payment of said check in full upon presentment, which check is
subsequently dishonored by the drawee bank for insufficiency of funds
xxx" Petitioners challenged the constitutionality of BP 22 on the
following grounds: 1) It offends the constitutional provision prohibiting
imprisonment for debt; 2) it impairs freedom of contract; 3) it
contravenes the equal protection clause; 4) it unduly delegates
legislative and executive powers; and 5) its enactment is flawed
because the Interim Batasan prohibited amendment of the bill on 3rd
reading.

HELD: The gravamen of the offense punished in BP 22 is the act of
making and issuing a worthless check or a check that is dishonored
upon its presentation for payment. It is not the non- payment of an
obligation which the law punishes. The law punishes the act not as an
offense against property but as an offense against public order. Recent
statistics show that one third of the entire money supply of the country
consists of currency in circulation. These demand deposits in the banks
constitute the funds against which commercial papers are drawn. The
amount concerned justifies the legitimate concern of the state in
preserving the integrity of the banking system.

3. Acts which when done were innocent

Art. III, Sec. 22. No ex post facto law or bill of attainder shall be enacted.

Ex Post Facto Law

An "ex post facto law" is a law that seeks to punish an act which, when
committed, was not yet a crime or was not as heavily punished. It is a law that
retroacts to the day of the act so as to cause prejudice to the person performing
the act. Its unfairness consists in the fact that the person could not have known
the act was criminal, and thus could not have avoided the crime. When a law is
more favorable to the accused, however, it is allowed to retroact.

In re Kay Villegas Kami, Inc., 35 SCRA 428

F: This petition for declaratory was filed by Kay Villegas Kami Inc., claiming
to be a duly recognized non-stock and non-profit corporation created under the
laws of the land, and praying for the detremination of the validity of Sec. 8, RA
6132 and a declaration of petitioner's right s and duties thereunder. Petitioner
claims that the challenged provision constitutes an ex post facto law.

ISSUE: W/N it is an ex post facto law.

HELD: NO
An ex post facto law is one which:
1. Makes criminal an act done before the passage of the law which was
innocent when done, and punishes such an act;
2. Aggravates a crime , or makes it greater than it was, when committed;
3. Changes the punishment and inflicts a greater punishment than the
law annexed to the crime when committed;
4. Alters the legal rules of evidence, and authorizes conviction upon less
or different testimony than the law required at the time of the commission of
the offense;
5. Assuming to regulate civil rights and remedies only, in effect imposes
penalty or deprivation of a right for something which when done was lawful; and
6. Deprives a person accused of a crime of some lawful protection to
which he has become entitled, such as the protection of a former conviction or
acquittal, or a proclamation of amnesty. [Quoting Mekin v. Wolfe, 2 Phil. 74
(1902)]
This constitutional prohibition refers only to criminal laws which are
given retroactive effect.
While it is true that Sec. 18 penalizes a violation of any provisin of RA
6132 including Sec. 8(a) thereof, the penalty is imposed only for acts committed
after the approval of the law and not those perpetrated prior thereto. There is
nothing in the law that remotely insinuates that its provisions shall apply to acts
carried out prior to its approval.

B. What punishments cannot be imposed

1. Involuntary servitude

Art. III, Sec. 18 (2) No involuntary sevitudes in any form shall exist,
except as a punishment for a crime whereof the party shall have been
convicted.


2. Excessive fines

Art. III, Sec. 19. (1) Excessive fines shall not be imposed. nor cruel,
degrading or inhuman punishment inflicted. Neither shall the death penalty
be imposed, unless for compelling reasons involving heinous crimes, the
Congress hereafter provides for it. Any death penalty already imposed shall be
reduced to reclusion perpetua.


3. Cruel, degrading and inhuman punishments

Art. III, Sec. 19. (1) Excessive fines shall not be imposed. nor cruel,
degrading or inhuman punishment inflicted. Neither shall the death penalty
be imposed, unless for compelling reasons involving heinous crimes, the
Congress hereafter provides for it. Any death penalty already imposed shall be
reduced to reclusion perpetua.
Id., Sec. 12. xxx
(2) No torture, force, violence, threat, intimidation, or any other means
which vitiate the free will shall be used against him. Secret detention places,
solitary, incommunicado, or other similar forms of detention are prohibited.


Assuming that judgment has been rendered and the accused has been
convicted the Constitution now further prescribes certain standards as to the
punishment that can be meted out. After all, due process prohibits barbaric and
disproportionate penalties.

The employment of physical, psychological or degrading punishment
against any prisoner or detainee, or the use of substandard or inadequate penal
facilities under subhuman conditions, shall be dealt with by law. [Art. III, Sec. 19
(2).]

In 1935, the prohibition was against "cruel and unusual" penalty, in 1973;
it was against "cruel or unusual " penalty; in 1987, the prohibition is against
"cruel, degrading or inhuman" punishment. The purpose in changing the
phraseology is to allow for experimentation, and not to fix the concept of what is
cruel to the standards of the present civilization, or those of antiquity. This
notion is supposed to expand and grow, so that what today is considered as
acceptable may in the next generation be deemed as cruel penalty.

Whether the cruelty of a punishment depends on its form or whether it
depends on its severity has been ambivalently answered by the SC:

In People v. dela Cruz, 92 Phil. 900 (1953) the SC ruled that it was the
form of punishment as fixed in antiquity (pillory desembowelment, etc.) and not
its severity, that constituted "cruel and unusual" penalty under the 1935
Constitution. Thus a disproportionate penalty (10 years imprisonment for theft)
is not cruel or unusual because it is only a matter of severity of an acceptable
form of punishment (imprisonment).

The SC spoke in a different way in People v. Borja 91 SCRA 340 (1979),
Borja was sentenced and he served at the national penitentiary for 20 years
before the case came to the SC. The Court said that Borja had been living in the
shadow of death. Although the sentence was initially valid, it had become cruel
CONSTI TUTI ONAL LAW 2 ZPG & ASSOCI ATES

by the lapse of time. And yet, this was a form of penalty that was neither cruel
nor unusual.


People v. Munoz, 170 SCRA 107 (1989)

F: The accused are four of the 11 bodyguards of a mayor who killed three
persons on suspicion that they were cattle rustlers. They were found guilty of
murder. Three appealed to the SC which found them equally liable for the killing.
The penalty for murder under the RPC is reclusion temporal to death. The
question concerns the penalty to be imposed in view of Art. III, sec. 19 which
provides that "Neither shall the death penalty be imposed, unless for compelling
reasons involving heinous crimes, Congress provides for it. Any death penalty
already imposed shall be reduced to reclusion perpetua."

HELD: Art. III, section 19 does not change the periods of the penalty prescribed
by Art. 248 of the RPC except insofar as it prohibits the imposition of the death
penalty adn reduces it to reclusion perpetua. The range of medium and
minimum penalties remain the same. VV.


People v. Lubreo, 200 SCRA 11 (1991)

F: A complaint for homicide was filed with MTC of Del Carmen, Surigao del
Norte, charging Remelito Lubreo along with crime of Homicide in connection
with the killing of Mamerto Sanico. Judge Gorgolon of said court conducted
both the preliminary investigation and preliminary examination. Thereafter, he
forwarded the records of the case to the Office of Provincial Fiscal. The fiscal
conducted his own PI and on the basis thereof, he filed an information for
murder not only against remelito but also against Lucresio Lubreo. Trial Court
find them guilty of the crime charged.

ISSUE: W/N the constituional presumption of innocence in favor of Lucrecio has
been overturned by the prosecution

HELD: NO.
An accused is presumed innocent until the contrary is proved. The
burden of proof is upon the prosecution and until such burden is sufficiently
discharged , the accused continues to enjoy the presumption of innocence. In
the instant case, the lower court convicted Lucrecio on the basis of its
conclusion that he was positively identified by witnesses Nenita Monter and
Epifanio Pangatungan as one of the assailants, and that therefore, his defense of
alibi would not prosper. Unfortunately, the testimonies of the abovementioned
witnesses did not categorically stated or proved that Lucrecio took part in
hacking the victim.
Though Monter categorically stated in her direct examination that she
saw the accused Lucresio hacking the victim, in the "re-enactmment", she
however candidly informed the court Lucresio was just standing by and she
could not remmenber as to who actually hacked the victim. From her version,
the participation of Lucrecio is at one enveloped inserious doubt. It is worse in
the case of Pangatungan. While he stated that "Lucrecio abetted in hacking as if
they will come one after the other in hacking his mind (sic) and the neck", he
never elaborated as to what "abetted in hacking " means. He could not even
specify the part of the body of Mamerto which was hit by Lucrecio.
There is evidently insufficient evidence to show the actual participation of
Lucresio in teh crime. There being no evidence of conspiracy, he cannot be held
for the acts of his co- appellant.

4. Secret detention places, solitary, incommunicado and other forms of
detention and the use of substandard or inadequate penal facilities


Art. III, Sec. 12. xxx
(2) No torture, force, violence, threat, intimidation, or any other means
which vitiate the free will shall be used against him. Secret detention places,
solitary, incommunicado, or other similar forms of detention are prohibited.

Id., Sec. 19. xxx
(2) The employment of physical, psychological, or degrading
punishment against any prisoner or detainee or the use of substandard or
inadequate penal facilities under subhuman conditions shall be dealt with by
law.

5. Indefinite Imprisonments

People v. Dacuycuy, 173 SCRA 90 (1989), supra.

C. The protection against double jeopardy

Art. III, Sec. 21. No person shall be twice put in jeopardy of punishment
for the same offense. If an act is punished by a law and an ordinance,
conviction or acquittal under either shall constitute a bar to another
prosecution for the same act.


Elements of double jeopardy, (Rule 117, Sec 7; People v. Obsania, 23 SCRA 249
(1968):

(1) Court of competent jurisdiction;
(2) A Complaint or Information sufficient in form and substance to sustain
a conviction;
(3) Arraignment and plea by the Accused;
(4) Conviction, acquittal, or dismissal of the case without the express
consent, of the accused.

Subsequent prosecution is barred for the following:

(1) Same offense
(2) Attempt of the same offense
(3) Frustration of the same offense
(4) Offense necessarily included in the 1st offense (All the elements of the
2nd constitute some of the elements of the 1st offense)
(5) Offense that necessarily includes the 1st offense (All the elements of
the 1st constitute some of the elements of the 2nd offense)

Exceptions to no. 5:

(1) The graver offense developed die to "supervening facts" arising from
the same act or omission constituting the former charged.

Thus, in Melo v. People, 85 Phils. 766 (1950), the SC allowed the
amnedment of the information from its original cahrge of frustrated homicide,
because after the filing of the information, the victim died.

(2) The facts constituting the graver charge became known or were
discovered only after the filing of the former complaint or information.

This overrules People v. Yorac, where the SC disallowed the amendment
of the information from slight physical injuries to frustrated murder after the
prosecution subjected the victim to another medical examination and found a
wound, that it was the fault of the prosecution if they had an incompetent
medical examination.

(3) The plea of guilty to the lesser offense was made without the consent
of the fiscal and the offended party.

Identity of offenses and identity of act

When an act gives rise to two or more offense which are punished by the
same authority, and an individual is convicted, acquitted, or the case dismissed
without his consent, of one of these offense (Crime A), there is no double
jeopardy if he is charged of another offfense (Crime B) flowing from the same
act. Double jeopardy arises only when he is again charged of that same offense
(Crime A). Thus, this is called double jeopardy by "identity of offenses".

But when an act which give rise to two or more offenses is punished by
two different authorities (a law and an ordinance), then if an individual is
convicted, acquitted, or the case dismissed without his consent, of any of these
offenses punished by one authority (Crime A by law), even if he is charged of
another offense which is punished by the other auhtority (Crime B by
ordinance), there is double jeopardy, because both offenses, one punished by a
law and the other punished by an ordinance, flowed from the same act. Thus,
this is called double jeopardy by "identity of act."

Sum: If only a law in involved, there is double jeopardy only when there is
an identity of offenses. But is a law and an ordinance are involved, there is
double jeopardy when there is an identity of act.

Identity of Offenses:

If a married man maintains as concubine a married woman not his wife,
the man is guilty of both concubinage and adultery. From the same act
(cohabiting with the married woman), two offenses arise. And yet he can be
prosecuted for both because, the two offenses coming from the same authority,
there is no identity of offenses.

Identity of Act:

People v. Relova, 48 SCRA 292 (1987), Relova was prosecuted under an
ordinance of Batangas City for the use of wiring to tap electricity without
permission from the local authorities, but the case was dismissed because the
crime has prescribed. So the fiscal filed a case for theft of electricity under the
RPC. The SC ruled there was double jeopardy already, and so the second case
could no longer be filed. For although the offenses were different, both flowed
from the same act. And in this case, the act was punished by a law and an
ordinance.

Loss of Jurisdiction: No double jeopardy

CONSTI TUTI ONAL LAW 2 ZPG & ASSOCI ATES

If the court has no jurisdiction, or was ousted of its jurisdiction beccause it
violated the right to due process of the parties, the decision is null and void, the
accused may again be charged.

In People v. Bocar, 138 SCRA 166 (1985), the SC, held that the move by
the trial court of summarily dismissing a criminal case for theft on the ground
that it merely involved a question of ownership deprived the prosecution of due
process by denying it the chance to introduce its evidence. This ousted the court
of its juridsiction.

In Galman v. Sandiganbayan, 144 SCRA 43 (1986), the SC declared the
criminal prosecution of the 26 accused in the Aquino-Galman double murder
case a "mistrial" after the SC commission found that the Sandiganbayan justices
and the Tanodbayan prosecutors had been summoned by the President and
instructed on how to conduct the trial. Due process is a right not only of the
accused but also of the State. Once the court deprives either party, which in this
case is the State, of a fighting chance, then it is ousted from its jurisdiction, and
double jeopardy would not apply. Thus, the accused were ordered retried.

1. Two situations contemplated

People v. Relova 148 SCRA 292 (1987)

F: Manuel Opulencia was charged wiht violation of Ordinance No. 1 series of
1974 of Batangas City prohibiting the installation of electric wiring devices
without authority from the city government. He admitted installing the electric
wiring devices found by the police in order to decrease the readings of electric
current. The case was however dismissed on the ground that the offense had
prescribed. Fourteen days later, the City Fiscal filed another case for theft
against him. The court also dismissed this case on the ground of double
jeopardy. The prosecution appealed contending the offense was different.

HELD: The contention has no merit. The first sentence of Art. III, sec. 21 states
the general rule: the constitutional protection against double jeopardy is not
available where the second prosecution is for an offense that is different from
the offense charged in the first or prior prosecution, although both may be
based from the same facts. The second sentence provides an exception: that the
protection against double jeopardy is available although the prior offense
charged under an ordinance be different from the offense charged subsequently
under the national statute such as the RPC provided that both offenses spring
from the same act or set of acts. VV.


People v. City Court of Manila, Branch VI, 154 SCRA 175 (1987)

F: Agapito Gonzales, together with Roberto Pangilinan, was accused of
violating Sec.7, in relation to Sec. 11 RA 3060 and Art. 201(3) of the RPC, in two
separate informations filed with the City Court of Manila. Upon arraignment,
accused Gonzales pleaded not guilty to both charges. The other accused,
Pangilinan, was not arraigned as he is still at large. Gonzales filed a motion to
quash the informations in the 2 cases on the ground that said informations did
not charge an offense. Motion denied. Later, he again moved to quash the
information in one of the Criminal case on the ground of duble jeopardy, as
there was according to him, also pending aginst him another criminal case,
where the informatin allegedly contain the same allegations as the information
in the first criminal case. Court granted the motion.

ISSUE: W/N there is double jeopardy.

HELD: NO
It is a settled rule that to raise the defense of double jeopardy, 3
requisites must be present: (1) a first jeopardy must have attached prior to the
second; (2) the first jeopardy must have been validly terminated; and (3) the
second jeopardy must be for teh same offense, or the second offense includes or
is necessarily included in the offense charged in the first information, or is an
attempt to commit the same or a frustration thereof. All these requisites do not
exist in this case,
The 2 informations with which the accused was charged , do not make
only one offense, contrary to private repondent's allegation. In other words, the
offense defined in Sec. 7 of the RA 3060 punishing the exhibition of motion
pictures not duly passed by the Board of Censors for Motion Pictures does not
include or is not included inthe offense defined in Art 201 (3) of the RPC
punishing the exhibition of indecent and immoral motin pictures.
The elements of the 2 offenses are different. The gravamen of the
offense defined in RA 3060 is the public exhibition of any motion pictures which
has not been previously passed by the Board of Censors for Motion Pictures.
The motion picture may be indecent or immoral but if it has not been previously
approved by the Board, its public showing constitutes a crimnal offense. On the
other hand, the offense punished in Art 201(3) of the RPC is the public showing
os indecent or immoral plays, scenes, acts, or shows, not just motion pictures.
The nature of both offenses also differs. The crime punished in RA 3060 is
malum prohibitum in wh criminal intent need not ber proved because it is
presumed, while the offense punished in Art. 201(3) of the RPC is malum in se,
which criminal intent is an indispensable ingredient. Suzette.


2. Rules of Court provisions

Rule 117, Sec. 7. Former conviction of acquittal; double jeopardy.--
When an accused has been convicted or acquitted, or the case against him
dismissed or otherwise terminated without his express consent by a court of
compentent jurisdiction, upon a valid complaint or information or other formal
charge sufficient in form and substance to sustain a conviction and after the
accused had pleaded to the charge, the conviction or acquittal of the accused
or the dismissal of the case shall be a bar to another prosecution for the
offense charged, or for any attempt to commit the same or frustration thereof,
or for any offense which necessarily includes or is necessarily included in the
offense in the former complaint of information.
However, the conviction of the accused shall not be a bar to another
prosecution for an offense which necessarily includes the offense charged in
the former complaint or information under any of the following instances:
(a) the graver offense developed due to supervening facts arising from
the same act or omission consituting the former charge;
(b) the facts constituting the graver charge became known or were
discovered only after the filing of the former complaint or information; or
(c) the plea of guilty to the lesser offense was made without the
consent of the fiscal and of the offended party.
In any of the foregoing cases, where the accused satisfied or serves in
whole or in part the judgement, he shall be credited with the same in the
event of conviction for the graver offense.


Melo v. People, 85 P 776 (1950)

F: Conrado Melo was charged in the CFI, Rizal with frustrated homicide , for
having allegedly inflicted upon Obillo, with a kitchen knife and with intent to kill,
several serious wounds on different parts of the body, requiring medical
attendance for a period of more than 30 days, and incapacitating him from
performing his habitual labor for the same period of time. On Dec. 29, 1949, at
8 am, accused pleaded not guilty to the offense chargde. At 10:15 am of the
same day, Obillo died from his wounds. An amended information was filed
charging accused with consummated homicide. Accused filed a motion to
quash the amended information alleging double jeopardy. Motion denied.

ISSUE: W/N there is double jeopardy.

HELD: NO
Double jeopardy means that when a person is charged with an offense
and the case is terminated either by acquittal or conviction or in any other
manner without the consent of the accused, the latter cannot again be charged
with the same or identical offense. The phrase "the same offense" has always
been construed to mean not only that the second offense charged is exactly the
same as the one alleged in the first information, but also that the two offenses
are identical. There is identity between the two offenses when the evidence to
support a conviction for one offense would be sufficient to warrant a conviction
for the other.
This rule of identity however does not apply, however, when the second
offense was not in existence at the time of the first prosecution, for the simple
reason that in such case there is no possibility for the accused, during the first
prosecution, to be convicted for an offense that was then inesistent. Thus,
where the accused was charged with physical injuries and after conviction the
accused dies, the charge for homicide against the same accused does not put
him twice in jeopardy.
Accordingly, an offense may be said to necessarily include or to be
necessarily included in another offense, for the purpose of detremining the
existence of double jeopardy, when both offenses were in existence during the
pendency of the first prosecution, for otherwise, if the second offense was then
inexistent, no jeopardy could attach therefor during the first prosecution, and
consequently a subsequent charge for the same cannot constitute a second
jeopardy. Suzette.


People v. City Court of Manila, Branch XI, 121 SCRA 637 (1983)

F: This is a petition to review the order of the City Court of Manila Branch XI,
dismissing the information for homicide thru reckless imprudence filed against
Gapay, in a criminal case on the ground of double jeopardy. Respondent court
held that the accused having been previously tried and convicted of serious
physical injuries thru reckless imprudence for the resulting death of the victim
would place the accused in double jeopardy.

ISSUE: W/N a person who has been prosecuted for serious physical injuries thru
reckless imprudence and convicted thereof may be prosecuted subsequently for
homicide thru reckless imprudence if the offended party dies as a result of the
same injuries.

HELD: YES
Well settled is the rule that one who has been charged with an offense
cannot be charged again with the same or identical offense though the latter be
lesser or greater than the former. However as held in the MELO case, the rule
CONSTI TUTI ONAL LAW 2 ZPG & ASSOCI ATES

of identity does not apply when the second offense was not in existence at the
time of teh first prosecution , for the reason that in such case there is no
possibility for the accused during the first prosecution, to be convicted for an
offense that was inexistent.
The victim Diolito de la Cruz died on the day the information was filed ,
and the accused was arraigned 2 days after or on October 20, 1972 . When the
information for homicide thru reckless imprudence was, therefore, filed on
October 24, 1972, the accused was already in doubly jeopardy. Suzette.


People v. Yorac, 42 SCRA 230 (1971)

F: Accused Yorac was charged with slight physical injuries before the City
Court of Bacolod, the offended party being Lam
Hock who, according to the medical cerificate issued by Dr. Rogelio Zulueta, was
confined since April 8 1968 up to the present time for head injury in Occidental
Negros Provincial Hspital. Accused pleaded guilty on April 16, 1968 resulting in
his being penalized to suffer 10 days for arresto menor. On April 18, 1968, the
provincial fiscal filed an information charging the same defendant with
frustrated murder arising from the same act against the aforesaid victim Lam
Hock for upon further diagnosis, the healing period for the injuries caused to
accused was found to be longer. A motion to quash was filed by the accused
on the ground of double jeopardy.

ISSUE: W/N the defendant, who had already been convicted of slight physical
injuries for injuries inflicted on Lam Hock , and had served sentence therefor,
may be prosecuted anew for frustrated murder for the same act committed
against the same person

HELD: NO.
In order not to violate the constitutional prohibition on double jeopardy,
there is the indispensable requirement of the existence of a new fact which
supervenes for which the defendant is responsible changing the character of the
crime imputed to him and together with the facts existing previously
constituting a new and distinct offense.
In this case, there is no supervening fact which occurred to justify the
non-existence of double jeopardy. The wound causing the delay in the healing
of the injuries caused to the victim was already in existence at the time of the
first examination of the doctor. Said delay was caused by the very superficial
and inconclusive examination then made resulting to a later finding of fracture.
Suzette.

Barlongay: When defense of double jeopardy available.-- (1) Dismissal based
on isufficiency of evidence; (2) dismissal bec. of denial of accused's right to
speedy trial; (3) accused is discharged to be a state witness.

When defense of double jeopardy not available.-- When the case is dismissed
other than on the merits upon motion of the accused personally, or through
counsel, such dismissal is regarded as w/ express consent of the accused, who is
therefore deemed to have waived the right to plea double jeopardy.

Yap v. Lutero, April 30, 1959

F: Yap was charged with reckless driving in violation of a city ordinance.
Later he was charged again in another criminal case in the same court with
serious physical injuries through reckless imprudence. Yap moved to quash the
latter information. Meanwhile, petitioner was acquitted in the first case.

ISSUE: W/N there was double jeopardy.

RULING: YES. From the viewpoint of Criminal Law, as distinguished from
Constitutional or Political Law - the offenses with which petitioner was charged
constitute, strictly different offenses, although, under certain conditions, one
offense may include the other, and accordingly, once placed in jeopardy for one,
the plea of double jeopardy may be in order as regards the other.
Thus, if the injuries mentioned in the second information were not
established by the evidence, petitioner could be convicted in the first case of the
very same violation of municipal ordinance charged in the first case, unless he
pleaded double jeopardy. Charo.


Galman v. Sandiganbayan, 144 SCRA 43

F: The petitioners filed an action to nullify the proceedings on the trial of the
Aquino-Galman duble murder case alleging that respondents Tanodbayan and
Sandiganbayan committed serious irregularities constituting mistrial and
resulting in miscarriage of justice and gross violation of the constitutional rights
of the petitioners and the sovereign people of the Philippines to due process of
law. The SC dismissed. Meanwhile, the Sandiganbayan rendered its decision
acquitting all the accused of the crime charged. Respondents submitted that in
view of the SB decision, the case has become moot and academic. Petitioners
filed a motion for reconsideration of the SC ruling. The SC created the Vasquez
Commisssion to look into petitioners' allegations.

RULING: The report of the Commission revealed that Pres. Marcos used the
overwhelming resources of the Government and his authoritarian powers to
corrupt and make a mockery of the judicial process in this case. The unwholly
scenario for the acquittal of the accused after the rigged trial would accomplish
the two principal objectives of satisfying the public clamor for the suspected
killers to be charged in court and of giviing them, through their acquittal, the
legal shield of double jepardy.
However, double jeopardy does not attach where a criminal trial was a
sham. A dictated, coerced and scripted verdict of acquittal such as in this case is
a void judgment. In legal contemplation, it is no judgment. It neither binds nor
bars anyone. The criminal collusion as to the handling and treatment of the
cases by public respondents completely disqualified them and voided ab initio
the SB verdict. DJ cannot be invoked where the prosecution, which represents
the sovereign people in crimnal cases is denied due process.


People v. Obsania, 23 SCRA 249

F: The information filed by the fiscal alleged that through violence and
intimidation, Obsania had carnal knowledge of one Erlinda Dollente against the
latter's will. Later, the fiscal amended the complaint to allege therein that the
offense was committed with lewd designs. The accused after pleading not guilty
moved for the dismissal of the case on the ground that the first information was
fatally defective for failing to allege "lewd desiigns," and that the amended
information did not cure the jurisdictional infirmity. The motion of the defense
was sustained by the judge. Hence this appeal by the fiscal.

RULING: The failure of the prosecution to allege "lewd designs" in the first
information does not affect the sufficiency in substance of the information, for
unchaste motives are deemed inherent in the very act of rape itself. In any case,
the lower court erred in dismissing the case by failing to distinguish between the
concept of jurisdiction and insufficiency in substance of an indictment.
As to the question of double jeopardy, the following requisites must have
been obtained to invoke the constitutional protection against it:
(1) a valid complaint or information;
(2) a court of competent jurisdiction;
(3) the defendant had pleaded to the charge; and
(4) the defendant was acquitted, or convicted, or the case against him
was dismissed or otherwise terminated without his express consent.

The only remaining and decisive issue in this case seems to be as to
whether or not the case was dismissed without the prior consent of the accused.
The SC ruled that as a general rule, when the case is dismissed, other than
on the merits, upon motion of the accused, such dismissal is to be ragarded as
with the express consent of the accused and consequently he is deemed to have
waived his right to plead double jeopardy and/or he is estopped from claiming
such defense on appeal by the Government or in another indictment for the
same offense.
The exception to this is where the dismissal is sought by the accused on
the ground that they were denied their right to a speedy trial and that the
government failed to prosecute; in which case double jeopardy will set in. The
case of herein accused falls under the general rule.

D. The privilege of the writ of habeas corpus

Art. III, Sec. 15. The privilege of the writ of habeas corpus shall not be
suspended except in cases of invasion or rebellion, when the public safety
requires it.


In case of invasion or rebellion, when the public safety requires it, the
President may, for a period not exceeding 60 days, suspend the privilege of the
writ of habeas corpus...

The suspension of the privilege of the writ shall apply only to persons
judicially charged for rebellion or offenses inherent in or directly connected with
invasion.

During the suspension of the privilege of the writ, any person thus
arrested or detained shall be judicially charged within 3 days, otherwise he shall
be released. (Art. VII, Sec. 18.)

A "writ of heabeas corpus" is a writ directed to the person detaining
another, commanding him to produce the body of the detainee at a designated
time and place, and to show cause why he should continue to be detained.

The "privilege of the writ" is the right to have the immediate
determination of the legality of the deprivation of physical liberty.

What is suspended is the privilege of the writ, and not the writ itself. The
writ will always issue as a matter of course. But when the privilege of the writ is
suspended, all the detaining office needs to do when he receives the writ of
habeas corpus is to show to the court that the detainee is being detained for an
offense covered by the suspension, and the court cannot inquire any further to
find out if the detention is legal. Under the Conmstitution, this is so only for 3
CONSTI TUTI ONAL LAW 2 ZPG & ASSOCI ATES

days. After 3 days, the Court can now require the detaining officer to produce
the body of the detainees and show cause why he should not be released.

The suspension of the privilege of the writ applied only to crimes related
to invasion or rebellion. An extensive discussion was made under the
Commander-in- Chief clause of the President, supra. This rest of the section will
be confined to habeas corpus as a remedy in all other offenses.

In general as already noted above, the privilege of the writ is an
extraordinary remedy to question the illegality of the arrest or detention, or any
other restraint to liberty. When all else is lost, it is the last recourse to get
someone out of his illegal detention.

1. Functions of the writ

Villavicencio v. Lukban, 39 P 778 (1919)

Habeas corpus is available not only for those who are in actual detention
but even for those whose liberty is merely restrained. Thus, in Moncupa v.
Enrile, 141 SCRA 233 (1986), the SC granted habeas corpus to petitioner who,
though temporarily released, could not travel outside Metro Manila, could not
change his residence, could not be interviewed by media, and had to report to
the military.


2. The writ of habeas corpus as a post-conviction remedy


In Chavez v. Court of Appeals, supra, habeas corpus was the remedy of
one whose confinement was the result of a void judgnment of conviction arrived
at after the judge violated due process by compelling him to take the stand and
testify against himself.

Chavez v. Court of Appeals, 24 SCRA 633 (1986), supra.


In Gumabon v. Director of Prison, 37 SCRA 420 (1971), some persons who
were charged with the complex crime of rebellion with homicide, rape, or other
common crimes, did not appeal their conviction and so were sentenced
accordingly. The other accused, however, appealed their conviction, resulting in
a new ruling in People v. Hernandez to the effect that there can be no complex
crim of rebellion with homicide, rape, etc., for these common crimes are
absorbed by rebellion. As a result, while those who appealed were now free,
those who did not remained in jail. The SC ruled that those who conrtinued to
languish in jail could avail of habeas corpus to question the legality of their
continued detention pursuant to the ruling in People v. Hernandez.

3. Suspension of the privilege

Art. VII, Sec. 18.
Lansang v. Garcia, 42 SCRA 488 (1971)

E. Affirmative rights

1. Free access to the courts

Art. III, Sec. 11. Free access to the courts and quasi-judicial bodies and
adequate legal assistance shall not be denied to any person by reason of
poverty.

2. Protection and enforcement of constitutional rights

Art. III, Sec. 12. xxx
(4) The law shall provide for penal and civil sanctionsfor violations of
this section as well as compensation to and rehabilitation of victims of torture
or similar practices, and their families.

3. Compensation to, and rehabilitation of, victims of tortures

Art. III, Sec. 12. xxx
(4) The law shall provide for penal and civil sanctions for violations of
this section as well as compensation to and rehabilitation of victims of torture
or similar practices, and their families.




IV. FREEDOM OF EXPRESSION


Art. III, Sec. 4. No law shall be passed abridging the freedom of speech,
of expression, or of the press, or the right of the people peaceably to assemble
and petition the Government for redress of grievance.

Id., Sec. 18. (1) No person shall be detained solely by reason of his
political beliefs and aspirations.
xxx

A. Philosophical Basis of Guarantees

Free Market Place of Ideas

1. For the discovery of political truth

When men have realized that time has upset many fighting faiths, they
may come to believe even more than they believe the very foundations of their
own conduct that the ultimate good desired is better reached by free trade in
ideas-- that the best test of truth is the power of the thought to get itself
accepted in the competition of the market, and the truth is the only ground
upon which their wishes safely can be carried out. (Justice Holmes, Abrams v.
United States, 250 U.S. 616. (1919)

The theory behind freedom of expression is the principle that ours is a
democratic society, and so the only way to rule ultimately is by, means of public
opinion, which is possible only when everyone can speak their minds out and
compete in the free market place of ideas.

2. For self government

United States v. Bustos, 37 P 731 (1918)


Burgos v. Chief of Staff, 133 SCRA 800 (1984), supra

HELD: As a consequence of the search and seizure, the premises of the
"Metropolitan Mail" and "We Forum" were padlocked and sealed, with the
further result that the printing and publication of said newspapers were
discontinued. Such closure is in the nature of previous restraint or censorship
abhorrent to the freedom of the press guaranteed under the fundamental law
and constitutes a virtual denial of petitioner's freedom to express themselves in
print. This state of being is patenly anathematic to a democratic framework
where a free, alert and even militant press is essential for the political
enlightenment and growth of the citizenry.

New York Times v. Sullivan, 380 US 51 (1964)

3. For individual protection

B. Prior Restraints

Thus any system of prior restraints of expression comes to the Court
bearing a heavy presumption against its constitutionality, giving the government
a heavy burden to show justification for the imposition of such restraint. (New
York v. United States (1971); also in New York Times v. Pentagon and Bantam
Books v. Publication of Pentagon Papers).


Sanidad v. COMELEC, 181 SCRA 529 (1990)

Subsequent Punishment

And even subsequent punishment is tempered by the greater interest of
promoting free public opinion. The most significant expression is the law on
libel.

We consider this case against the background of a profound national
commitment to debate on public issues being uninhibited, robust and wide-
open, and that it may well include vehement, caustic, and sometimes
unpleasantly sharp attacks on government and public officials. The falsity of
some of the factual statements and alleged defamations do not qualify the role.
And just as factual error afforded no warrant for repressing speech that would
otherwise be free, the same is true of injury to official reputation. (New York
Times v. Sullivan, 380 U.S. 51 (1964)

The interest of society and good government demands a full discussion of
public affairs. Whether the law is wisely or badly enforced is a fit subject for
proper comment. Public policy, welfare of society, and the orderly
administration of government have demanded protection for public opinion.
The inevitable and incontestable result has been the development and adoption
of the doctrine of privilege. [Justice Malcom, United States v. Bustos, 731
(1918).]

While, uncer the Revised Penal Code, any defamatory statement is
presumed to be malicious (malice-in-law), when the defense proves that the
communication is privileged, such a presumption of malice does not arise
because of the greater public interest involved.

If the communication is absolutely privileged (as in parliamentary
freedom of speech), the prosecution cannot even prove malice-in-fact.
CONSTI TUTI ONAL LAW 2 ZPG & ASSOCI ATES


If the communication is only qualifiedly privileged (Art. 354 enumerates
the 2 instances: fair and true reporting of an official proceeding; legal moral or
social duty), the burden is shifted on the prosecution to prove malice-in-fact,
which the defense can overcome by proving the truth of the defamatory
statement (which in the case of public officials may or may not constitute a
crime, so long as related to the conduct of his office) and good motive.

C. Content-Based Restrictions

1. Test of validity of content-based restrictions

The U.S. Supreme Court and, by haphazard imitation, the Philippine
Supreme Court, have evolved certain tests to regulate the contents of speech.

Dangerous Tendency Test: When the legislative body has determined
generally, in the exercise of its discretion, that utterances of a certain kind
involve such danger of a substantive evil that they may be punished, the
question whether any specific utterance coming within the prohibited class is
likely, in and itself, to bring the substantive evils, is not open to consideration. In
such cases, the general provision of the statute may be constitutionally applied
to the specific utterance if its natural and probable effect was to bring about the
substantive evil which the legislative body might prohibit. [Gitlow v. New York,
268 US 652 (1925).]

Example: Art. 142. Inciting to sedition. When the legislature has decided
that one who advocates a certain conduct is guilty of a crime, the court cannot
intrude. As it evolved, this test was supposed to apply when there is a statute, in
contrast to the clear and present danger rule which applies when the speech is
not prohibited by statute.

Clear and Present Danger Test: The question in every case is whether the
words used are used in such circumstances and are of such a nature as to create
a clear and present danger that they will bring about the substantive evils that
Congress has a right to prevent. It is a question of proximity and degree.
[Schenck v. United States, 249 US 47 (1919).]

The emphasis of the test is the nature of the circumstances under which it
is uttered. The speech itself may not be dangerous. As Holmes said: "Many
things that might be said in time of peace are such a hindrance to its effort that
their utterance will not be endured so long as men fight." Or saying "Fire" in a
crowded movie house.

Grave-but-improbable danger: Whether the gravity of the evil,
discounted by its improbability, justifies such an invasion of free speech as is
necessary to avoid the danger. [Dennis v. United States, 341 US 494 (1951),
quoting Judge Learned Hand.]

This test was meant to supplant the clear and present danger. They both
emphasize the circumstances of the speech, but this latter test consider the
weighing of values.

Direct Incitement Test: The consitutional guarantees of free speech and
press do not permit a State to forbid or proscribe advocacy of the use of force or
of law violation, except where such advocacy or peech is directed to inciting or
producing imminent lawless action, and is likely to incite or produce such action.
[Brandenburg v. Ohio, 395 U.S. 444 (1969), cited in Salonga v. Cruz Pano, 134
SCRA 438 (1985).]

The test emphasizes the very words uttered: (a) What words did he
utter? (b) What is the likely result of such utterance? It criticizes the clear and
present danger test for being top dependent on the circumstances. Speaker
may, when tested show no incitement but you know the speaker is inciting to
sedition.

Balancing of Interest Test: The court must undertake the delicate and
difficult task of weighing the circumstances and appraising the substantiality of
the reasons advanced in support of the regulation of the free enjoyment of
rights. [American Communication Ass'n v. Douds, 339 US 383 cited in Gonzales
v. COMELEC, 27 SCRA 835 (1969A)]

The test applied when two legitimate values not involving national
secuirty crimes compete. Involves an appoint of the competing interest.
(Gonzales v. Comelec)

In Aver v. Capulong and Enrile, for instance, it is a question of balancing
the freedom of expression of the producer and the right to privacy of Enrile.

(not in VV's revised outline)
Balancing of Factors Test: The truth is theat the clear-and-present danger
test is over- simplified judgement unless it takes into account also a number of
other factors: (1) the relative seriousness of the danger in comparison with the
value of the occasion for speech or political activity, (2) the availability of more
moderate controls than those the State has imposed, and perhaps (3) the
specific intent with which the speech is launched. (Freund, quoted in Dennis v.
United States in the concurring opinion of Justice Frankfurter).


2. Applications of tests in various contexts

a. Freedom of expression and national security

Babst v. National Intelligence Board 132 SCRA 316 (1984)

F: Petitioners are journalists and columnists. On different dates in July 1980,
they were summoned by military authorities for interrogation regarding their
work, feelings, sentiments, beliefs, associations and even private lives. In
addition, one of them was charged with libel by a General who sought to recover
P10 million in damages. They brought an action for prohibition to stop the NIB
from questioning them and from filing libel suits on matters that had been the
subject of inquiry by the NIB.

HELD: The petition has become moot and academic. Be that as it may, it is not
idle to note that, while ordinarily, an invitation to attend a hearing and answer
some questions is not illegal or constitutionally objectionable, under certain
circumstances, however, such an invitation can easily assume a different
appearance as when it comes from a powerful group composed predominantly
of ranking military officers and the designate interrogation site is a military
camp.

b. Freedom of expression and criticism of official conduct: The Test of "Actual
Malice"

Read Revised Penal Code, Articles 353-354 and 361-362

Freedom of expression and libel

Freedom of speech versus right to reputation. Libel is the most common
form of subsequent punishment. Although one cannot be prevented from
saying something before he actually says it, one can be held liable for what one
has said if it causes damage to the rights of others.


Soliven v. Makasiar; Beltran v. Makasiar, 167 SCRA 393 (1988)

F: The President of the Philippines filed a complaint for libel against the
petitioners, who were the publisher and columnist of the Philippine Star, based
on the following statement in Beltran's column of Oct. 12, 1987 totle "The
Nervous Officials of the Aquino Administration": "If you recall, during the August
29 coup attempt, the President hid under her bed while the firing was going on -
perhaps the first Commander-in-Chief to do so." Beltran did not submit a
counter affidavit and instead, moved to dismiss the complaint. The fiscal denied
his motion. Thus, this petition for certiorari.

HELD: xxx
(3) As regards the contention of petitioner Beltran that he could not be
held liable for libel bec. of the privileged character of the publication, the Court
reiterates that it is not a trier of facts and that such a defense is best left to the
trial court to appreciate after receiving the evidence of the parties. As to
petitioner Beltran's claim that to allow the libel case to proceed would produce a
"chilling effect" on the press freedom, the Court finds no basis at this stage to
rule on the point. VV.

Manuel v. Cruz-Pano, 172 SCRA 225 (1989)

Libel suits based on official criticisms should be dismissed outright unless made in
bad faith

F: Petitioner wrote the Chairman of the Anti-Smuggling Action Center
denouncing abuses allegedly committed by ASAC agents against petitioner's
clients. Petitioner said the agents subjected Ng Woo Hay to indignities and took
her necklace and bracelet and her son's wristwatch plus HK$ 70. But the agents
were exonerated so petitioner filed criminal charges of robbery. Petitioner found
prosecutors unsympathetic so he filed a civil action for damages against the
agents. Later, the Bulletin Today published a news item based on petitioner's
letter to ASAC. This became the basis of an action for libel brought against
petitioner and his clients. Petitioner moved to quash the case but his motion was
denied.

HELD: From the viewpoint of procedural and substantive law, the charge is
defective. The letter constitutes privileged communication. It was sent by
petitioner in his capacity as lawyer in the discharge of his legal duty to his clients.
He could also invke his civic duty as a private individual to expose anomalies in
the public service. The complaint was addressed to the official who had
authority over them and could impose proper disciplinary sanctions. As an index
of good faith, the letter was sent privately, directly to the addressee without any
funfare nor publicity. As for the news report, it is difficult to believe that the
petitioner, an ordinary citizen without known ties to newspaper, could have by
himself caused the publication. It does not appear either that the report was
CONSTI TUTI ONAL LAW 2 ZPG & ASSOCI ATES

paid for like an advertisement. At any rate, the news item is a true and fair
report of a judicial proceeding, made in good faith and without comments or
remarks. VV.


Newsweek Inc. v. IAC 142 SCRA 171 (1986)

F: Petitioner was sued for libel in connection with the publication in the Feb.
23, 1981 issue of Newsweek of the article "An Island of Fear." The plaintiffs,
sugar planters of Bacolod, complained that the article portrayed them as
exploiters of sugar workers. Petitioner moved to dismiss the complaint on the
ground that the article was not libelous since it did not single any particular
individual. The trial court denied the motion and petitioner filed a petition for
certiorari in the IAC which was dismissed. Thus, this appeal to the SC.

HELD: Where the defamation is alleged to have been directed at a group or
class, it is essential that the statement must be so sweeping or all-embracing as
to apply to every individual in that group or class, or sufficiently specific so that
each individual in the class or group can prove that the defamatory statement
specifically pointed to him, so that he can bring the action separately if need be.
The disputed portion which refers to plaintiff Sola never singled out Sola. The
news report merely stated that the victim had been arrested by members of a
special police unit brought into the area by Sola, the mayor. Hence, the report
referring as it does to an official act is within the realm of privileged and is
protected by the constitutional guarantees of free speech and press. VV.

Notes: Since the Newsweek artciles "Island of fear in the Visayas" did not
specify any individual, it cannot be libelous. An article must be sufficiently,
specific or at least sweeping as to apply to all members of a group, in order to be
deemed libelous.

Lopez v. Court of Appeals, 34 SCRA 116 (1970)

The pictures of a former mayor was inadvertently published and mistaken
for another man who was a sanitary inspector and fooled the authorities about
the Babuyan Islands, claiming of murders there, so they could go and he could
be rescued. An erratum was published by the This Week magazine. The SC,
quoting Quisumbing v. Lopez, however, found for plaintiff, but with reduced
damages, since the error in in this case could have been checked consideringing
that this was a weekly magazine and not a daily.

Quisumbing v. Fernando, 96 Phil 510 (1955)

Newspapers should be given leeway and tolerance to enable them to
courageously and effectively perform their important role in our democracy. In
the preparation of stories, press reporters and editors usually have to race to
their deadlines; and consistently with good faith and reasonable care, they
should not be held to account, to a point of suppression, for honest mistakes or
imperfection in the choice of words.
.

Mercado v. CFI of Rizal 116 SCRA 93 (1982)

F: Petitioner was accused of libel on the basis of a telegram which he sent to
the Secretary of Public Works requesting investigation of Mrs. Virginia Mercado
of the Public Service Commission "as we have reason to believe that she has
enriched herself thru corrupt practices xxx." He filed a motion to dismiss on the
ground that his communication was privileged, but his motion was denied. He
filed another motion which was also denied. Thus, this petition for certiorari,
mandamus and prohibition in the SC.

HELD: US v. Bustos is a landmark decision antedating by forty years a similar
decision of the US Supreme Court to the effect that a libel prosecution must
survive the test of whether or not the offending publication is within the
guarantees of free speech and free press. However, Justice Malcolm in US v.
Bustos was careful to point out that qualified privilege and this is one instance
may be "lost by proof of malice." What casts doubt on the good faith of
petitioner is his conduct, vis--vis private respondent. The tenacity with which
petitioner had pursued a course of conduct on its face would seem to indicate
that a doubt could reasonably be entertained as the bona fides of petitioner.
The prosecution should be given a chance to prove malice.

c. Freedom of expression and the right to privacy

Lagunzad v. Gonzales, 92 SCRA 476 (1979)

F: Lagunzad filmed the Moises Padilla story based on a book written by
Rodriguez. xxx Nelly Amane who was a half-sister of Padilla objected to the
movie on the ground that it contained a portrayal of Padilla's private and family
life, including scenes about his mother, Maria Soto vda. de Gonzales, and a
certain "Auring" as Padilla's girl friend. Subsequently, Nelly Amante, together w/
her sister and mother, agreed to allow petitioner to "exploit, use and develope
the life story of Moises Padilla for purposes of producing the pictures," in
consideration of P20,000. Petitioner paid P5,000 but as he failed to pay the
balance agreed upon, he was sued. Judgement was rendered against him by the
trial court, w/c was affirmed by the CA. Petitioner appealed to the SC
contending that he was forced to enter into the agreement only to avoid
financial loss caused by delay in the showing of the movie and the relatives of
Padilla did not have a property right in the life of M. Padilla since Padilla was a
public figure.

HELD: Petitioner's averment is not well taken. Being a public figure does not
automatically destroy in toto a person's right to privacy. The right to invade a
person's privacy to disseminate public information does not extend to fictional
or novelized representation of a person, no matter how a public figure he or she
may be. In the case at bar, while it is true that petitioner exerted efforts to
present the true-to-life story of Moises Padilla, petitioner admits that he
included a little romance in the film bec. w/o it, it would be a drab story of
torture and brutality.
Freedom of expression, indeed, occupies a preferred position in the
hierarchy of civil liberties. It is not, however, w/o limitations. In the particular
circumstances presented and considering the obligations assumed by petitioner
under the agreement, the validity of such agreement will have to be upheld
particular bec. the limits of freedom of expression are reached when expression
touches upon matters of private concern. [In the agreement signed by him,
petitioner admitted that in the picture produced, he had "exploited the life story
of Moises Padilla for pecuniary gain, and other profit motives, and (had)
encroached upon the privacy of Moises Padilla's immediate family, and (had) in
fact included, in the PICTURE's case, persons portraying some of MOISES
PADILLA's kin..."]


Ayer Productions Pty. Ltd. v. Capulong April 29, 1988

F: Pivate respondent Juan Ponce Enrile filed an action in the RTC of Makati
to enjoin the petitioners from producing the movie "The Four Day Revolution," a
documentary of the EDSA Revolution in 1986 on the ground that it violated his
right to privacy. Petitioners contended that the movie would not involve his
private life not that of his family. But the trial court issued a writ of preliminary
injunction and ordered petitioners to desist from making the movie making
reference whatsoever to Ponce Enrile. This, this action for certiorari.

HELD: Freedom of speech and expression includes freedom to produce motion
pictures and to exhibit them. What is involved is a prior restraint by the Judge
upon the exercise of speech and of expression by petitioners. Because of the
preferred character of speech and of expression, a weighty presumption of
invalidity vitiates measures of prior restraint. The Judge should have stayed his
hand considering that the movie was yet uncompleted and therefore there was
no "clear and present danger." The subject matter of the movie does not relate
to the private life of Ponce Enrile. The intrusion is no more than necessary to
keep the film a truthful historical account. He is, after all, a public figure. The
line of equilibrium in the specific context of the instant case between freedom of
speech and of expression and the right of privacy may be marked out in terms of
a requirement that the proposed motion picture must be fairly truthful and
historical in its presentation of facts. There must be no showing of a reckless
disregard of truth.

Notes: Ayer sought to produce a movie on the 4-day revolution. Enrile,
who had previously been asked for the use of his character in the movie and had
refused the offer, sued to enjoin the filming because he did not want any
mention of his and his family's name. The SC lifted the injunction issued by the
lower court on the ground that it amounted to prior restraint, which is no better
if imposed by the courts than if imposed by administrative bodies or by
ecclesiatical officials.

In Ayer, the reference to Enrile is unavoidable because his name is part of
history and this cannot be changed or altered; thus his name can be used so long
as only his public life is dwelled only. But in Lagunzad, although Moises Padilla
was also a public figure, the movie dealth with both the public and private lives
of Moises Padilla.

d. Freedom of expression and administration of justice (contempt of court)

In re Ramon Tulfo, AM NO. 90-4-1545-0, April 17. 1990

Zaldivar v. Sandiganbayan, 170 SCRA 1 (1989)

Cabansag v. Fernandez, 102 Phil 152 (1957)

A contempt imposed by the court on the party who sent a letter to the
Presidential Action Committee complaining about the delay in the disposition of
the agrarian case, was lifted by the SC. It held that although such a letter should
have been sent to the SC and not the PAC, it was nevertheless a valid exercise of
speech which did not significantly destroy, the orderly administration of justice.

People v. Alarcon, 60 Phil 265 (1939)

A person can be held liable for making comments on a pending case (sub
judice) which have the tendency to impair or obstruct the orderly administration
CONSTI TUTI ONAL LAW 2 ZPG & ASSOCI ATES

of justistice. But if the case is not pending, such comment is a valid exercise of
the freedom of expression.

e. Symbolic Expression-- The Flag-burning case

Flag burning when done to express dissent is protected speech.

F: Respondent Johnson participated in a political demonstration where he
burned an American flag while protesters chanted. No one was physically
injured or threatened with injury, although several witnesses were seriously
offended by the flag burning. Johnson was convicted of desecration of a
venerated object in violation of a Texas statute which (1) prohibited the
desecration of, among other things, a state or national fl ag, and (2) defined
desecration as the physical mistreatment of such objects in a way which the
actor knows will seriously offend one or more persons likely to observe or
discover the act. A state court of appeals affirmed. The Court of Criminal
Appeals of Texas reversed, holding that the desecration statute as applied
violated the defendant's right to freedom of speech under the Federal Consti-
tution's First Amendment, because the statute (1) was too broad for First
Amendment purposes as it related to breaches of the peace, and (2) was not
adequately supported by the state's purported interest in preserving a symbol of
unity.

ISSUE: Whether the flag desecration statute is unconstitutional

HELD: YES. Decision Affirmed.
Johnson's conviction was inconsistent with the First Amendment under
the particular circumstances because (1) Johnson's conduct was sufficiently
imbued with elements of communication to implicate the First Amendment,
given that this flag burning was the culmination of a political demonstration and
that the state conceded that the protester's conduct was expressive; (2) the
state's interest in preventing breaches of the peace was not implicated on the
record in this case, since (a) no disturbance of the peace actually occurred or
threatened to occur because of the flag burning, (b) it cannot be presumed that
an audience which takes serious offense at a particular expression is necessarily
likely to disturb the peace, and (c) the flag burning does not fall within the small
class of "fighting words" that are likely to provoke the average person to
retaliation and thereby cause a breach of the peace; and (3) the state's asserted
interest in preserving the flag as a symbol of nationhood and national unity does
not justify the conviction, since (a) the attempted restriction on expression is
content-based, and thus subject to the most exacting scrutiny, given that the
flag desecration statute is aimed not at protecting the physical integrity of the
flag in all circumstances, but only against impairments that would cause serious
offenses to others and is aimed at protecting onlookers from being offended by
the ideas expressed by the prohibited activity, and (b) although the state has a
legitimate interest in encouraging proper treatment of the flag, it may not foster
its own view of the flag by prohibiting expressive conduct relating to it and by
criminally punishing a person for burning the flag as a means of political protest.

f. Movies Censorship

While prior restraint is the general rule, censorship in the movies is
tolerated because by the nature of the medium, it has a greater impact on the
audience and produces instant reaction for the ideas it presents, unlike
newspapers which are read by people separated by walls.


Gonzales v. Katigbak, 137 SCRA 356 (1985)

F: Petitioner was the producer of the movie Kapit sa Patalim which the
Board of Review for Motion Pictures and Televisions allowed on condition that
certain deletions were made and that it was shown on adults only. The
petitioner brought an action, claiming violation of their freedom of expression.

HELD: Motion pictures are important both as a method for the communication
of ideas and the expression of the artistic impulse. The power of the Board is
limited to the classification of films. For freedom of expression is the rule and
restrictions the exception. The power to impose prior restraint is not to be
presumed, rather the presumption is against its validity. Censorship is allowable
only under the clearest proof of a clear and present danger of a substantive evil
to public safety, public morals, public health or any other legitimate public
interest. The Board committed an abuse of discretion in subjecting petitioner to
difficulty and travail before the movie was classified as "For adults only" without
deletion. However there is not enough votes to consider the abuse of discretion
grave as it explained that there were reasons for its action because of the scenes
showing women erotically dancing naked and kissing and caressing each other
like lesbians. VV.


Notes: The movie involved in this case was "Kapit sa Patalim" which the
censors wanted to cut in some part and to label "For Adults". The SC rules that
movies are within the constitutional protection of freedom of expression, so that
censorship is presumed to be valid as constituting prior restraint. The only case
whe the Board of Censors can order a deletion is when there is a clear and
present danger of a substantive evil against national security or public morals or
other public interest. In all other cases, the Board can only classify.

But a different standard must be followed in television because of the
pervasive and intrusive influence of the medium on people who watch its
programs without having to pay anything.

On the issue of obscenity, the SC held that sex along is not necessarily
obscenity, the test being whether, using contemporary community standards,
the dominant appeal us to the prurient interest. (Miller v. California). Thus on
this score, it found abuse of discretion of the part of the Board for subjecting the
producer to difficulty and for entertaining a narrow view of obscenity, but it
lacked the votes to rules that the abuse was grave.

Tests of obscenity:
(1) Whether the average person, applying contemporary community
standards, would find that the work, taken as a whole, appeals to the prurient
interest.
(2) Whether the work depicts or describes, in a patently offensive way,
sexual conduct specifically defined by the applicable law.
(3) Whether the work, taken as a whole, lacks serious literary, artistic,
political or scientific value. (Miller v. California, 37 L. Ed. 2d 419.)

g. Radio Broadcast

In Eastern Broadcasting Corp. v. Dans, 137 SCRA 647, the SC held that radio
broadcast also enjoys the protection of the freedom of expression. If closed
down, the owners enjoy the rights to due process according to the standards set
in Ang Tibay v. CIR.

But radio deserves greater regulation than newspapers because it could
invade the privacy of everyone for no fee, and it is such that one is likely to listen
to what is being said.


Eastern Broadcasting Corp. (DYRE) V. Dans, 137 SCRA 647 (1985)

F: The petitioners filed this action to compel respondent government
officials to allow the reopening of Radio Station DYRE after it had been closed for
allegedly having been used to incite the people to sedition. The petitioner
contended that it was denied due process because no hearing was held and no
proof was submitted to establish a factual basis for the closure. However, before
the Court could promulgate its decision the petitioner filed a motion to
withdraw its action on the ground that it had sold the radio station to Manuel
Pastrana and that the National Telecommunications Commission had expressed
its willingness to grant the requisite license.

HELD: The case has been moot and academic. However, for the guidance of the
inferior courts and administrative bodies, the following guidelines must be
observed: 1) The cardinal primary requirements in administrative proceedings as
laid down in Ang Tibay v. CIR should be followed before a broadcast station may
be closed; 2) All forms of communication are entitled to the broad protection of
the freedom of expression clause. Necessarily, the freedom of television and
radio broadcasting is somewhat lesser in scope than the freedom accorded to
newspapers and print media. This limitation derives from the fact the broadcast
media have a uniquely pervasive presence in the lives of all Filipinos; 3) The
government has a right to be protected against broadcasts which incite listeners
to violently overthrow it; and 4) Broadcast stations deserve the special
protection given to all forms of media by the due process and freedom of
expression clauses of the Constitution.

h. Freedom of Information

Art. III, Sec. 7. The right of the people to information on matters of
public concern shall be recognized. Access to official records, and to
documents and papers pertaining to, official acts, transactions, or decisions, as
well as to government research data used as basis for policy development,
shall be afforded the citizen, subject to such limitations as may be provided by
law.

Baldoza v. Dimaano, 71 SCRA 14 (1976)

Access of official records (the docket book) for any lawful purpose (to look
into the criminal cases for a report on the peace and order situation of the
municipality) is guaranteed. But it is subject to reasonable conditions by the
custodian of the records.


Garcia v. BOI, 177 SCRA 374 (1989)


D. Content-Neutral Restrictions

O'brien test: A government regulation is sufficiently justified if it is within
the constitutional power of the government; if it furthers an important or
CONSTI TUTI ONAL LAW 2 ZPG & ASSOCI ATES

substantial governmental interest; if the governmental interest is unrelated to
the suppression of free expression; and if the incidental restriction on alleged
freedom of expression is no greater than is essential to the furtherance of that
interest. [US v. O'brien, 391 US 367 (1968), adopted in Adiong v. COMELEC, 207
SCRA 712 (1992)]

1. Regulation of political campaign

National Press Club v. COMELEC, 207 SCRA 1 (1992)

F: Petitioners herein were representatives of mass media which were
prevented from selling and donating space or air time for political
advertisements under RA 6646.

ISSUE: Whether or not RA 6646 constitutes a violation of the constitutional right
to freedom of expression.

RULING: NO. The Comelec has been expressly authorized by the Constitution to
supervise or regulate the enjoyment or utilization of the franchises or permits
for the operation f media of communication and information. The fundamental
purposes of such power are to ensure "equal opportunity, time, and space, and
the right to reply," as well as uniform and reasonable rates of charges for the use
of such media facilities, in connection with "public information campaigns and
forums among candidates."
Of course, the law limits the right of free speech and of access to mass
media of the candidates themselves. The limitation however, bears a clear and
reasonable connection with the objective set out in the Constitution. For it is
precisely in the unlimited purchase of print space and radio and television time
that the resources of the financially affluent candidates are likely to make a
crucial difference.

Adiong v. COMELEC, 207 SCRA 712 (1992)

F: Petitoner, Adiong, a 1992 senatorial candidate, assails Comelec
Resolution No. 2347 insofar as it prohibits the posting of decals and stickers on
mobile places, public or private, and limits their location or publication to
authorized posting areas.

ISSUE: Whether or not the resolution is constitutional.

RULING: NO. The prohibition unduly infringes on the citizen's fundamental right
of free speech. There is no public interest substantial enough to warrant the kind
of restriction involved in this case. The posting of decals amd stickers in mobile
places does not endanger any substantial government or public interest. Under
the clear and present danger rule, not only must the danger be patently clear
and pressingly present but the evil sought to be avoided, must be so substantive
as to justify a clamp over one's mouth or a writing instrument to be stilled.
Significantly, the freedom of expression curtailed by the prohibition is not
so much that of the candidate or the political party. The regulation strikes at the
freedoom of an individual to express his preference and, by displaying it on his
car, to convince others to agree with him. A sticker may be furnished by a
candidate but once the car owner agrees to have it placed on his private
vehichle, the expression becomes a statement by the owner, primarily his own
and not of anybody else.
Morever, The restriction is so broad that it encompasses even the citizen's
private property, which in this case is a privately owned vehicle. In consequence
of this prohibition, another cardinal right guaranteed under the Constitution is
violated which is that no person shall be deprived of his property without due
proocess of law.

2. Freedom of Assembly

Public Assembly Act of 1985 (Batas Blg. 580)

A permit to hold a rally must be filed with the Office of the Mayor at least,
five working days before the day of the rally.

But no permit from the mayor is required in case the rally is going to be
held in (i) freedom parks, (ii) inside a private property (provide with consent of
the owner), and (iii) campuses of state universities (which are left to university
authorities)

The application must be in writing and must include: (1) names of the
organizers and leaders, (2) date and time, place and street, (3) size (4)manner of
the use of the street, (5) sound system to be used (6)purpose. It must also have
a statement of the duties of the rallyists.

The written application is filed with the Office of the Mayor.
Acknowledgemet is given of its receipt. If the Mayor refuses to accept the
application, then it is enough for filing purposes if a copy is posted in the
premises.

The Mayor has 2 working days to act on the application. If he does not
act, it is deemed granted.

But if he thinks that the rally creates a "clear and present danger" to
public peace, order, health, etc., and he has proof of this, he should not deny the
application right away. He should hold a hearing during which the applicant can
be heard. If after hearing he is still not satisfied that no danger exists, then he
can deny the application.

The applicant can then go to any court other than the Supreme Court for
the review of the decision of denial of the mayor. The courts have 24 hours to
act on the petition. If the judgment is a reversal of the denial, or in any case if
the applicant is satisfied with the decision, the judgment becomes final and
executory immediately, and no appeal can be taken by the local authorities
anymore.

But if the decision is not satisfactory to the applicant, then he has 48
hours from receipt to appeal to the SC.

During the rally, the police must be limited to maintaining peace and
order and so must stay away by 100 meters from the rallyists. They must be in
full uniform, with their names visibly written. They can carry no firearm except a
nighstick, but they are allowed protective devices.

If they anticipate trouble, the police must call the attention of the leader
of the rallyists. When trouble actually erupts, the police must not disperse the
crowd right away but first give a warning. If violence persists, they must give a
second warning. If still violence continues, only then can they fight back.

If a rally does not have a permit, the police can disperse the crowd, but
they cannot use violence. Penalty is imposed only on the leaders and organizers.

Among the duties of the rallyists are: (a) to inform the members of their
duty under the law, (b) to police their own rank, and (c) to cooperate with local
authorities in maintaining peace and order.


Notes: The freedom to use public places to peaceably assemble is best
expressed thus: "Wherever the title or steets and parks may rest, they have
immemorially been held in trust for the use of the public and, time out of time
have been used for purposes of assembly, communicating thought betwee
citizens, and discussing public questions." (Justice Roberts. Hague v. CIO)

Although under a "permit system", before one can use a public place, one
must first obtain prior permit from the proper authorities, the principle has
always been that one has the right to a permit, subject only to reasonable
regulation. The validity of the permit system has been upheld by the Court,
provided, (a) it is concered only with the time, place and manner of assembly ad
(b) it does not vest on the licensing authority unfettered discretion in choosing
the groups which could use the public place and discriminate others.

As held by the SC in Primicias vs Fugoso, 80 Phil. 71, the City Ordinance of
Manila giving authority to the Mayor to issue permits for parades should be
construed to be limited to the time, place, and manner of the parades socially to
secure public order, convenience and welfare. Thus, denying the Nacionalista
Party a permit to hold a rally at the Plaza Miranda on the ground that passions
raised by the recent national election were still high and a rally to protest
election anomalies could only exacerbate the matter, was overturned by the
court.


Primicias vs Fugoso, 80 Phil. 71

F: This is an action for mandamus instituted by petitioner Primicias,
campaign manager of the Coalesced Minority Parties, to compel Mayor Fugoso
of the City of Manila to issue a permit for the holding of a peaceful public
meeting at Plaza Miranda for the purpose of petitioning the government for
redress of grievances. The Mayor denied the application on the ground that
passions still run high due to the recent election, and a rally to protest election
anomalies might threaten breaches of the peace and disruption of public order.

ISSUE: W/n the Mayor can refuse to grant the permit.

RULING: NO. The police power granted to the Mayor under the Ordinance
enacted by the Municipal Board pursuant to its authority under the Revised
Administrative Code which pertains to the use of streets and public places, can
be construed only to mean the power to regulate, which means and includes
the power to control, govern, and to restrain but cannot be construed as
synonymous with "suppress" or "prohibit."
The Court quoted with approval the decision in the American case Cox v.
State of New Hampshire, " a statute requiring persons using public streets for a
parade or procession to procure a special license therefor from the local
authorities is not an unconstitutional abridgement of the rights of assembly or of
freedom of speech and press, where, as the statute is construed by the state
courts, the licensing authorities are strictly limited, in the issuance of licenses, to
a consideration of the time, place, and manner of the parade or procession, with
a view to conserving the public convenience and of affording an opportunity to
CONSTI TUTI ONAL LAW 2 ZPG & ASSOCI ATES

provide proper policing, and are not invested with arbitrary discretion to issue or
refuse license..."


But under the same ordinance, the SC, in Navarro v. Villegas, 31 SCRA 730
(1970), upheld the mayor's refusal to grant permit to a group during weekdays,
on a finding that everytime there was an announced rally, stores closed and
business was gravely affected because of violent incidents. It found the policy of
the mayor to allow rallies only during weekends to be reasonable.


Navarro v. Villegas, 31 SCRA 730 (1970)

F: The petitioner, acting in behalf of the Movement for a Democratic
Philippines (MDP), an association of students, workers and peasants, applied for
a permit from the Mayor of Manila to hold a rally at Plaza Miranda. Respondent
Mayor denied the application to hold the rally on the date and time specified
by petitioners in view of the events that transpired during the last
demonstration held by them which ended in the destruction of public and
private property, loss of a few lives, injuries to a score of other persons and the
closing down of schools, offices and many stores. The Mayor suggested that the
MDP utilize the Sunken Gardens near Intramuros for its rally and that the rally
be held during weekends and earlier during the day so that it may end before
dark.
Petitioner challenged the action of the Mayor on the ground that the
same constitutes a violation of their right to freedom of assembly. Petitioner
contended that the right of the people to peaceful assembly and to petition the
government for redress of grievances may be exercised without the prior
necessity of securing a permit from the government and that such right cannot
be fully enjoyed without the corresponding right to use public places for that
purpose.

ISSUE: Whether or not the Mayor`s denial to issue a permit amounted to a
violation of petitioner`s right to freedom of assembly.

HELD: NO.
The respondent Mayor has not denied nor absolutely refused the permit
sought by petitioner. He has expressed willingness to grant the permit for the
peaceful assembly during certain days and time, and at a place when they would
not disrupt the normal activities of the community.
The respondent mayor possesses reasonable discretion to determine or
specify the streets or public places to be used for the assembly in order to
secure convenient use thereof by others and provide adequate and proper
policing to minimize the risks of disorder and maintain public safety and order.
Petitioner has failed to show a clear specific legal duty on the part of
respondent Mayor to grant their application for a permit unconditionally.
Experience in connection with present assemblies and demonstrations have
shown that they pose a clear and imminent danger of public disorders, breaches
of the peace, criminal acts, and even bloodshed as an aftermath of such
assemblies, which, petitioner has manifested, it has no means of preventing.
Charo.


In Ignacio v. Ela, 99 Phil. 346 (1956), the majority upheld the mayor's
denial of permit to members of the Jehovah's Witnesses sect for the use of a
klosk within the town plaza in order to avoid any untoward incident with
members of the Roman Catholic Church, whose tenets are opposed to those of
the petitioners, and whose church is very near the klosk.



Ignacio v. Ela, 99 Phil. 346 (1956)

F: The Mayor denied a permit to the members of the Jehovah's Witnesses to
use the kiosk in the town plaza for the purpose of holding a public lecture on the
ground that the permit, if granted, may give rise to disturbance of the religious
ceremonies being performed by the Catholic Church which was said to be within
hearing distance from the kiosk and which might lead to any untoward incident
with members of the rival denomination.

ISSUE: W/N the denial is valid.

In J.B.L. Reyes v. Bagatsing, 125 SCRA 553 (1983), the SC found no basis
for the denial of permit to the Anti-Bases Coalition to hold a march from Luneta
to the street fronting the U.S. Embassy. It affirmed the general rule that the use
of streets is free to all. It found the fear entertained by city authorities that the
rallyists might be agirated by provocateurs to be unfounded, given the report of
the NPD that adequate security measures were provided by the police.

The Court did not rule on the validity of the ordinance of Manila
prohibiting any rally within 200 meters from any foreign embassy as a means of
complying with the Geneva Convention that requires the host country to protect
the premises and personnel of the embassy.

Then it gave guidelines for the issuance of permits (now in BP 9801 (i) any
group which applies must do so within a sufficient time so the authority can
have time to act: (ii) if a disagreement arises over a denial of a permit, the
applicant can question the denial in the lower court, which can try questions of
fact and law, and (iii) appeal can be made to the SC on an expedited procedure.

J.B.L. Reyes v. Bagatsing, 125 SCRA 553 (1983)

F: Retired Justice JBL Reyes, on behalf of the Anti-Bases Coalition, sought a
permit from the City of Manila to hold a peaceful march and rally on Oct. 26,
1983 starting 2 p.m. from Luneta to the gates of the US Embassy. He filed this
petition because as of Oct. 20, there was yet no action on his request to hold a
rally.

HELD: Free speech, like free press, may be identified with the liberty to discuss
publicly and truthfully any matter of public concern without censorship or
punishment. There is to be no previous retraint whether in the form of libel
suits, prosecution for damages, or contempt proceedings unless there is a "clear
and present danger of a substantive evil that the State has a right to prevent."
There can be no legal objection, absent the existence of a clear and present
danger of a substantive evil to the holding of a peaceful rally at Luneta. Neither
can there be objection to the use of the streets up to gates of the US Embassy. A
statute requiring persons to secure a special license to use public streets for a
procession is not unconstitutional. The licensing of authorities are strictly limited
to the consideration of the time, place and manner and the authorities are not
invested with arbitrary discretion to issue or refuse a permit.

In German v. Barangan, 135 SCRA 514 (1985), the SC upheld the power of
the city authorities to close JP Laurel Street fronting Malacanang from all rallies
as a form of "area restriction", in order to protect the President and his family,
based on the incident in the early 70s when the gates of the palace were almost
stormed. The rallyists in this case purported to merely worship at St. Jude's.

In case a rally is held in a private place, no permit from the mayor is
required. However, the consent of the owner of the place must be acquired.

German v. Barangan 35 SCRA 514 (1985)

F: On Oct. 2, 1984 the petitioners who were businessmen, students and
employees, met on JP Laurel Street in Manila for the ostensible purpose of
hearing mass at the St. Jude Chapel which adjoins the Malacaang grounds. They
wore yellow T-shirts and, with clenched fists, marched on the street and shouted
anti-government invectives. They were stopped from proceeding to the chapel
by the Presidential Security Command. They brought an action for mandamus.

HELD: The yellow T-shirts worn by some of the marchers, their fists clenched
and chants of anti-government investives support the government's claim that
the petitioners purpose was not really to worship at the chapel but to hold an
anti-government demonstration close to the residence of the President. The
restricted use of JP Laurel Street is justified. The need to secure the safety of
heads of states cannot be overemphasized. The threat to their lives is constant
and felt throughout the world. The petitioners were not restrained in their
freedom of religion but only in the manner by which they had attempted to
translate the same into action.
In Malabanan v. Ramento, 129 SCRA 359 (1984) and Arreza v. GAUP, 13
SCRA 94 (1985), the SC upheld the right to expression of students who held a
rally in a private university. But since they held it beyond the time granted in a
place other than the one allowed by the administration, their suspension was
condoned.

Malabanan v. Ramento, 129 SCRA 359 (1984)

F: Petitioners were officers of the Supreme Student Council of the Gregorio
Araneta University Foundation. They were granted a permit to hold a meeting to
protest the merger of two units of the university. On the scheduled date, the
students continued their meeting beyond the scheduled time and held it in a
different place from that indicated in the permit. They expressed in a vehement
language their opposition to the merger and as a result, classes and office work
was disturbed. Petitioners were placed under preventive suspension. On appeal,
they were found guilt of holding an illegal assembly and oral defamation. They
were suspended for one academic year. They filed a petition for certiorari in the
SC.

HELD: The petititon may be considered moot and academic considering that the
TRO issued by the SC allowed the students to enroll. But there is a need to pass
squarely on the constitutional question. Respect for the constitutional rights of
peaceable assembly and free speech calls for the setting aside of the order of
suspension. Suspending them for one year is out of proportion considering that
the vigorous presentation of views was expected. The excitement of the
occasion, the propensity of speakers to exaggerate and the exuberance of the
youth should be taken into consideration.


Arreza v. GAUP, 13 SCRA 94 (1985)

CONSTI TUTI ONAL LAW 2 ZPG & ASSOCI ATES

F: Petitioners were officers and members of the Student Council of the
Gregorio Araneta University Foundation. They were refused enrollment for
having led a rally on Sept. 28, 1982.

HELD: As held in Malabanan v. Ramento: "If in the course of such
demonstration, with an enthusiastic audience goading them on, utterances,
extremely critical, at times even vitriolic, were let loose, that is quite
understandable. They would be ineffective if during the rally they speak in the
guarded and judicious language of the academe. At any rate, even a sympathetic
audience is not disposed to accord full credence to their fiery exhortations. They
take into account the excitement of the occasion, the propensity of speakers to
exaggerate, the exuberance of youth. xxx" The refusal of the university to enroll
the students is a highly disproportionate penalty.
Notes: Note that while the permit system is not allowed in the case of
publication, it is allowed in the case of assembly. In publication, censorship is
presumptively unconstitutional. There is very little possibility or justification for
the regulation of news. The remedy in this case is prosecution or subsequent
punishment.
But in assembly regulation is allowed because it is needed by the very
nature of the expression, when people use streets, they may deprive other
groups which want to use the streets too. So as long as only the incidents of
speech are regulated, the measure is constitutionally acceptable.


Nestle Phils. Inc. v. Sanchez 154 SCRA 541 (1987)

F: While these cases were pending in the SC, the labor unions involved
intensified the pickets they had been conducting in front of the Padre Faura gate
of the Court and set up picket quarters, at times obstructing access to and egress
from the Court's premises. When required to show cause why they should not
be held in contempt of court, their lawyer apologized and assured that the
above incident would not be repeated.

HELD: The Court will not hesitate in future similar situations to apply the full
force of the law and punish for contempt those who attempt to pressure the
Court into acting one way or the other in any case pending before it. Grievances
must be ventilated in the proper channels, i.e. through appropriate petitions or
pleadings in keeping with the respect due the courts as impartial administrators
of justice. Moreover, "parties have a constitutional right to have the causes tried
fairly in court by an impartial tribunal, uninfluenced by publication or public
clamor xxx" The acts of respondents are not only an affront to the dignity of this
Court but equally a violation of the above-stated right of the adverse parties and
the citizenry at large.

3. Freedom of Association and the right to strike in the public sector

Art. III, Sec. 8. The right of the people, including those employed in the
public and private sectors, to form unions, associations, or societies for
purposes not contrary to law shall not be abridged.

The inclusion of the right to unionize in this article is ill-advised because
while the right to unionize is an economic and labor right, the right to
association in general is a civil- political right.
Discussed elsewhere is the argument why public employees cannot
engage in collective bargaining and strike.


SSS Employees Assn vs CA, 175 SCRA 686 (1989)

F: SSS filed w/ the RTC-QC a complaint for damages w/ a prayer for a writ of prel
inj. against petitioners SSSEA, alleging that the officers and members of the
latter staged an illegal strike and barricaded the entrances to the SSS building
preventing non-striking employees from reporting to work and SSS members
from transacting business w/ SSS. The Public Sector Labor-Management Council
ordered the strikers to return to work but the strikers refused to do so. The
SSSEA went on strike bec. SSS failed to act on the union's demands.
Petitioners filed a motion to dismiss the complaint for lack of
jurisdiction, w/c motion was denied. The restraining order w/c was previously
issued was converted into an injunction after finding the strike illegal.
Petitioners appealed the case to the CA. The latter held that since the
employees of SSS are govt employees, they are not allowed to strike.

HELD: Employees in the Civil Service may not resort to strikes, walkouts and
other temporary work stoppages, like workers in the private sector, in order to
pressure the Govt. to accede to their demands. As now provided under Sec. 4,
Rule III of the Rules and Regulations to Govern the Exercise of the Right of Govt.
EEs to Self-Organization which took effect after the initial dispute arose, the
terms and conditions of employment in the Govt, including any political
subdivision or instrumentality thereof and govt. owned and controlled
corporations with original charters, are governed by law and employees therein
shall not strike for the purpose of securing changes thereof.
The statement of the court in Alliance of Govt Workers v. Minister of
Labor and Employment (124 SCRA 1) is relevant as it furnishes the rationale for
distinguishing bet. workers in the private sector and govt employees w/ regard
to the right to strike?

Since the terms and conditions of govt. employment are
fixed by law, govt. workers cannot use the same weapons
employed by workers in the private sector to secure
concessions from their employers. The principle behind
labor unionism in private industry is that industrial peace
cannot be secured through compulsion of law. Relations
bet. private employers and their employees rest on an
essentially voluntary basis. Subject to the minimum
requirements of wage laws and other labor and welfare
legislation, the terms and conditions of employment in the
unionized private sector are settled through the process of
collective bargaining. In govt employment, however, it is
the legislature and, where properly given delegated
power, the administrative heads of govt w/c fix the terms
and conditions of employment. And this is effected
through statutes or administrative circulars, rules, and
regulations, not through CBA's

E. Academic Freedom

Garcia v. Faculty of Admission, 68 SCRA 277 (1975)

F: The FAC of the Loyola School of Theology refused to readmit petitioner,
Garcia, in its M.A. program because they felt that "her frequent questions and
difficulties were not always pertinent and had the effect of slowing down the
progress of the class;" that it would be "to the best interest (of the petitioner) to
work with a faculty that is more compatible with her orientation. Garcia assailled
her expulsion for being unreasonable; that the reasons given therefor were
invalid for nowhere did it appear that her conduct constituted a violation of the
school's regulations and grave misconduct.

ISSUE: Whether or not the FAC can be compelled by mandamus to readmit
petitioner.

RULING: NO. The Constitution recognizes the enjoyment by institutions of higher
learning of the right to academic freedom. The school decides for itself its aims
and objectives and how best to attain them. It is free from outside coercion or
interference save possibly when the overriding public welfare calls for some
restraint. It has a wide sphere of autonomy certainly extending to the choice of
the students.
The collective liberty of an organization is by no means the same thing as
the freedom of the individual members within it. In considering the problems of
academic freedom, one must distinguish between autonomy of the university, as
a corporate body, and the freedom of the individual university teacher.
The personal aspect of the freedom consists of the right of each university
teacher to seek and express the truth as he personally sees it, both in his
academic work and in his capacity as a private citizen. This status of the
individual teacher is as important as the status of the institution to which he
belongs and through which he disseminates learning.
On other hand, the internal conditions for academic freedom in a
university are that the academic staff should have de facto control of the
following functions: (a) admission and examination of students; (b) curricula for
courses of study; (c) appointment and tenure of office of academic staff; and (d)
allocation of income among the different categories of expenditure. It is the
business of a university to proviide that atmosphere which is most conducive to
speculation, experiment and creation. It is an atmosphere in which the four
essential freedoms of a university prevail - to determine for itself who may
teach, what may be taught, how it shall be taught, and who may be admitted to
study.
For the above reason, mandamus is not available for the petitioner.
There is no duty on the part of the School to admit her to study since the School
clearly has the discretion to turn down even qualified applicants due to
limitations of space, facilities, professors and optimum classroom size and
component considerations. There are standards to meet and policies to pursue.
What a student possesses is a privilege rather than a right.

UP v. Ayson, 176 SCRA 647 (1989)

F: In 1972, the UP BOR approved the establishment of the UPCB Highshool
to serve, among others, "as a laboratory and demonstration school for
prospective teachers - provided that UPCBHS must be self-supporting."
However, the Dept of Professional Education in Baguio was never organized. So,
the BOR decided to phase out UPCBHS for failing to attain the conditions for its
creation. The UPCBHS Foundation Inc. sought to restrain the University from
phasing out the UPCBHS.

ISSUE: Is secondary public education demandable in an institution of higher
learning such as the UP?

RULING: NO. UP invokes its exercise of academic freedom. Private respondent
invokes the right to quality education and to free secondary education.
The rights invoked by private respondent may be asserted only as against
the Government through the DECS. UP was created under its charter to provide
CONSTI TUTI ONAL LAW 2 ZPG & ASSOCI ATES

advanced tertiary education. An institute of higher learning cannot be compelled
to provide for secondary education.
It is beyond cavil that UP as an institution of higher learning enjoys
academic freedom. UPCBHS was established subject to a number of
conditionalities. Failing on such conditions, UP can order its abolition on
academic grounds. Charo.


UP v. CA, Feb. 9, 1993

F: Former PANAMIN Minister Manuel Elizalde and the Tasaday
representative filed a complaint for damages and declaratory relief against UP
Professors Jerome Bailen and Zeus Salazar who disputed the authenticity of the
Tasaday find and made a proposition in various conferences attended by them
that Elizalde merely fabricated the discovery of the Tasadays.
UP intervened, aaserting its duty to protect the respondents as faculty
members for acts and utterances made in the exercise of academic freedom.
The lower court denied UP's motion to dismiss for failure to state a cause of
action. Hence this petition.

RULING: With respect to the prayer of the complaint for "judgment declaring the
Tasadays to be a distinct ethnic community, the lower court is cautioned that
the same is akin to a prayer for a judicial declaration of Philippine citizenship
which may not be granted in a petition for declaratory relief. The complaint was
filed mainly to vindicate plaintiff's dignity and honor.
Indeed, it is beyond the province of the court to make pronouncements
on matters beyond its ken and expertise. To be sure, in resolving the complaint
for damages, the court may find congruence in what is justiciable and what falls
within the field of the sciences. Still, it is best to keep in mind that its proper role
and function is the determination of legal issues.

V. FREEDOM OF RELIGION

Art. III, Sec. 5. No law shall be made respecting an establishment of
religion; or prohibiting the free exercise thereof. The free exercise and
enjoyment of religious profession and worship, without discrimination or
preference, shall forever be allowed. No religious test shall be required for the
exercise of civil or political rights.

A. Non-Establishment Clause

The clause prohibits excessive government entanglement with,
endorsement or disapproval of religion [Vicoriano v. Elizalde Rope Workers
Union, 59 SCRA 54 (1974); Lynch v. Donnelly, 465 US 668 (1984) (O'Connor, J.,
concurring); Allegheny County v. Greater Pittsburg ACLU, 492 US 574 (1989).]

The clause prohibits the State from establishing a religion. In assessing
the validity of the law, the questions to be asked are:
a. Is the purpose of the law religious, or is it secular?
b. Does it or does it not inhibit or advance religion?
c. Is its effect to promote or to avoid an excessive entaglement between
the State and religious matters in religion?

The Non-Establishment clause is violated when the State gives any
manifest support to any one religion, even if nothing is done against the
individual.

It is likewise violated if the State favors all religions, for there may be
atheists who are not so favored.

1. Operation of sectarian schools

While the ownership, creation and management of educational
institutions must be in the hands of Filipinos or 60% Filipino-owned
corporations, sectarian schools and those run by religious groups and missions
board are exempted from these requirements, provided the administration is in
the hands of Filipinos, who could be sectarian. [Art. XIV, Sec. 4(2).]


2. Religious instruction in public schools


Provided it is upon the written petition of the parents and it is at no cost
to the State (although this is not entirely possible, because the use of classrooms
and electricity are costs in the State), religious instruction in public elementary
and secondary schools during class hours, by one approved by the authorities of
the religion of the child or ward is allowed. [Art. XIV, Sec. 3(3).] Religion can
even be integrated in the school curriculum. [Civ. Code, 359 (1).]

3. Anti-evolution laws

In Epperson v. Arkansas, 393 U.S. 97 (1968), the SC held that the teaching
of the Darwinian theory of evolution cannot be prohibited from public shools by
parents whose religions finds the theory offensive.

4. Prayer and Bible-reading in public schools

In Engel v. Vitale, 370 U.S. 421 (1967), the SC disallowed the conducting of
an interdenominational prayer before the start of classes in public schools as,
violative of the Non- Establishment clause.

Engel v. Vitale, 370 U.S. 421 (1967)

F: The respondent Board of Education upon the recommendation of the
State Board of Regents, directed the School's District principal to cause the
recitation in public schools of a brief, denominationally neutral prayer. Its
observance on the part of the students was voluntary.

RULING: The Court ruled that the State of New York, by using its public school
system to encourage the recitation of the Regent's prayer has adopted a practice
wholly inconsistent with the Establishment Clause. The prayer was composed by
govt officials as part of a governmental program to further religious beliefs. The
constitutional prohibition against laws respecting an establishment of religion
means at least that it is not part of the business of the government to compose
official prayers for any group to recite as part of a religious program carried on
by the govt.
The clauses of the 1st Amendment which prohibit laws respecting an
establishment of religion and abridging the free exercise thereof, although
overlapping in certain instances, forbids two diff kinds of governmental
encroachment upon religious freedom. The stablishment clause, unlike the free
exercise clause, does not depend upon any showing of direct governmental
compulsion and is violated by the enactment of laws which establish an official
religion, whether or not those laws operate directly to coerce non-observing
individuals. It rests on the belief that a union of govt and religion tends to
destroy govt and to degrade religion, and upon an awareness of the historical
fact that governmentally established religion and religious persecutions go hand
in hand.

In Abington School District v. Schemp, 374 U.S. 203 (1963), it likewise
disallowed the reading of a passage from the bible without comment in public
schools as contrary to the Non- Establishment clause.

Abington School District v. Schemp, 374 U.S. 203 (1963)

The issue was whether the establishment clause was violated by a Pennsylvania
Statute or a rule of the Board of Commissioners of Baltimore adopted pursuant
to statutory authority requiring the reading without comment, at the opening of
each school day, of verses from the Bible and the recitation of the Lord's prayer
by the students in unison. The students and parents may refuse to participate in
the school exercises. These exercises were prescribed as part of the curricular
activities of students who are required by law to attend school and held in
school buildings under the supervision and participation of teachers employed in
those schools.

RULING: YES, the establishment clause was violated.
The establishment clause prohibits a state from placing official support
behind the tenets of one or all orthodoxies and the free exercise clause
guarantees the right of every person to freely choose his own course with
reference to religious training, teaching and observance, free from any
compulsion from the State.
The test in determining whether a legislative enactment violates the
Establishment clause which withdraws all legislative power respecting religious
belief or the expression thereof, is the PURPOSE and the PRIMARY EFFECT of the
enactment. If either is the advancement or inhibition of religion, then the
enactment exceeds the scope of legislative power as circumscribed by the First
Amendment. To withstand the strictures of the establishment clause, there must
be a secular legislative purpose and a primary effect that neither advances nor
inhibits religion.
The place of the Bible as an instrument of religion cannot be gainsaid. This
is particularly so where the State's recognition of the pervading religious
character of the exercise is evident from the rule's specific permission of the
alternative use of the Catholic Douay version of the Bible as well as from a
recent amendment permitting non-attendance at the exercises, none of those
factors being consistent with the contention that the Bible is used either as an
instrument for non-religious moral inspiration or as a reference for the teaching
of secular subjects.

5. Tax exemption

Art. VI, Sec. 28. xxx
(3) Charitable institutions, churches, parsonages or convents
appurtenant thereto, mosques, non-profit cemeteries, and all lands, buildings
and improvements, actually, directly, and exclusively used for religious,
charitable or educational purposes shall be exempt from taxation.


The ruling in Bishop of Nueva Segovia v. Provincial Board, 51 Phil. 352
(1927) is modified to the extent now that the property must be "actually,
directly and exclusively" used for religious purposes to be exempt.

CONSTI TUTI ONAL LAW 2 ZPG & ASSOCI ATES

If not for religious purposes, educational purposes.

Bishop of Nueva Segovia v. Provincial Board, 51 Phil. 352 (1927)

F: The plaintiff, the Roman Catholic Apostolic Church, represented by the
Bishop of Nueva Segovia, is the owner and occupant of a parcel of land in San
Nicolas, Ilocos Norte. On the south siide is a part of the church yard, the convent
and an adjacent lot used as vegetable garden. In the center is the remainder of
the churchyard and the church. On the north side is an old cemetery and the
base of what was once a tower. The Prov. Board imposed a tax on the whole
land.

ISSUE: Whether or not the taxation is legal.

RULING: NO. The exemption in the payment of the land tax mandated in the
Constitution in favor of the religious entities refers to the home of the priest
who presides over the church and who has to take care of himself in order to
discharge his duties. It therefore must include not only the land actually
occupied by the church but also the adjacent ground destined for the ordinary
incidental uses of man.
Except in large cities where the density of the population and the
development of commerce require the use of large tracts of land for buildings, a
vegetable garden belongs to a house and, in the case of a convent, its use is
limited to the necessities of the priest. Therefore, which comes under the tax
exemption.
As to the lot which was formerly the cemetery, while it is no longer used
as such, neither is it used for commercial purposes and, accdg to the evidence, is
now being used as a lodging house by the people who participate in religious
festivities. The same constitutes an incidental use in religious functions. It also
comes within the exemption.

6. Public aid to religion

The payment or use of public money or property for any rel igious
institution or clergy is not allowed; except in those cases provided in the
Constitution: priests assigned in the AFP, penal institution, government
orphanage, or leprosarium. [Art. VI, Sec. 29 (2)]

But in Aglipay v. Ruiz, 64 Phil. 201 (1937), the SC held that the stamp
printed by the government showing the map of the Philippines with a rosary to
commemorate the 33rd International Eucharistic Congress to be held in Manila
did not violate the Non-Establishment clause because its main purpose, was to
call the world's attention to Manila as the site of an international congress, and
whatever benefit it gave the Catholic Church was only incidental.




Aglipay v. Ruiz, 64 Phil. 201 (1937)

F: The petitioner, Mons. Aglipay, Head of the Phil. Ind. Church, sought to
restrain respondent Director of Posts from issuing and selling postage stamps
commemorative of the 33rd International Eucharistic Congress. The Director
issued the stamps under the provisions of Act 4052 which appropriates public
funds for the cost of the plates and printing of the stamps. Petitioner alleged
that the issuance of the stamps was done in violation of the Constitutional
provision that no public money or property shall be appropriated for the use,
benefit or support of any sect or religion.

ISSUE: W/N petitioner's contention is tenable.

RULING: NO. Act 4052 contemplated no religious purpose in view. What it gave
the Director of Posts was the discretionary power to determine when the
issuance of special postage stamps would be advantageous to the government.
The purpose in issuing the stamps was to advertise the Philippines and
attract more tourists to this country. The officials concerned merely took
advantage of an event considered of international importance to give publicity
to the country and its people. The stamp contained a map of the Philippines and
the location of Manila, and an inscription as follows: "Seat XXXIII International
Eucharistic Crusade." What was emphasized was not the event but Manila.
It was obvious that while the stamps may be said to be inseparably linked
with an event of a religious character, the resulting propaganda received by the
Roman Catholic Church was merely incidental and was not the aim and purpose
of the government.

In Ignacio v. Ela, supra, the dissenting opinion of Justice Concepcion
pointed out that the mayor disapproved the application for a permit not so
much because he was afraid that breach of peace would ensue but because he
wrongly though the kiosk should be used for public purposes only and not for
religious purposes. When the Jehovah's Witness members use the public
squares, they are no different from ordinary pedestrians or promenaders who
use the street: that they are performing religious acts is only incidental. So long
as the use of public property is only incidentalally and temporarilly for religious
purposes and so long as the use is such as to be reasonably compatible with the
use to which other members of the community are similarly entitled, then the
non-establishment clause is not violated. The tests then are (1) Is the use of the
public facility compatible with general use? (2) Is the resulting benefit to the
religious group only incidental.

B. Free Exercise Clause

1. Flag Salute

Ebranilag v. Division Superindentent of Schools of Cebu, 219 SCRA 256 (1993)

Conscientious Objectors cannot be compelled to salute the flag.

F: All the ptetitioners in these cases were expelled from their classes by
the public school authorities in Cebu for refusing to salute the flag, since the
national anthem and recite the patriotic pledge as required by RA 1265 and by
Dept. Order No. 8 dated July 21, 1955 of the DECS making the flag ceremony
compulsory in all educational institutions.

ISSUE: W/N school children who are members of a religious sect known as
Jehovah's Witnesses may be expelled from school (both private and public), for
refusing, on account of their religious beliefs, to take part in the flag ceremony
which includes playing (by a band) or singing the Phil. National Anthem, saluting
the Phil. flag and reciting the patriotic pledge.

HELD: NO.
The idea that one may be compelled to salute the flag, sing the national
anthem, and recite the patriotice pledge, during a flag ceremony on pain of
being dismissed from one's job or of being expelled from school, is alien to the
conscience of the present generation of Filipinos who cut their teeth on the Bill
of Rights w/c guarantees their rights to free speech and the free exercise of
religious profession and worship.
xxx
xxx Forcing a small religious group, through the iron hand of the law, to
participate in a ceremony that violates their religious beliefs, will hardly be
conducive to love of country or respect for duly constituted authorities.
xxx
The sole justification for a prior restraint or limitation on the exercise of
religious freedom is the existence of a grave and present danger of a character
both grave and imminent, of a serious evil to public safety, moral, health or any
other legitimate public interest, that the state has a right and duty to prevent.
Absent such a threat to public safety, the expulsion of the pets. from the schools
is not justified.
xxx
Although petitioners do not participate in the compulsory flag ceremony,
they do not engage in external acts or behavior that would offend their
countrymen who believe in exercising their love of country through the
observance of the flag ceremony. They quietly stand at attention during the
ceremony to show their respect for the right of those who choose to participate
in the solemn proceedings. As there is no disruption, expulsion is unwarranted.
However, if they should commit breaches of peace by action that offend
the sensibilities, both religious and patriotic, of other persons, the school
authorities have the power to discipline them.


Compare West V. Board of Education v. Barnette, 319 US 624 (1943)

F: The State Board required public school pupils to salute the flag of the
United States while reciting a pledge of allegiance under penalty of expulsion
entailing liability of both pupil and parents to be proceeded against for unlawful
absence. Appellees, members of the Jehovah's Witnesses, consider the flag as a
graven image which they are forbidden to salute under their religious beliefs.
The State asserts the power to condition access to public education.

ISSUE: W/N the compulsory flag salute is valid.

RULING: NO.
In connection with pledges, the flag salute is a form of utterance. It
requires an affirmation of a belief and an attitude of mind. It is now a
commonplace that censorship or suppression of expression of opinion is
tolerated by the Constitution only when the expression presents a clear and
present danger of action of a kind the State is empowered to prevent and
punish. Here the power of compulsion is invoked without any allegation that
remaining passive during a flag salute ritual creates a clear and present danger
that would justify an effort even to muffle expression.
To sustain the compulsory flag salute, we are required to say that a Bill of
Rights which guards the individual's right to speak his mind left it open to public
authorities to compel him to utter what is not in his mind.
The Court applies the limitations of the Constitution with no fear that
freedom to be intellectually and spiritually diverse or even contrary will
disintegrate the social organization. To believe that patriotism will not flourish if
patriotic ceremonies are voluntary and spontaneous instead of a compulsory
routine is to make an unflattering estimate of the appeal of our institutions to
free minds.

2. Freedom to propagate religious doctrines
CONSTI TUTI ONAL LAW 2 ZPG & ASSOCI ATES


American Bible Society v. City of Manila, 101 P 386 (1957)

F: Plaintiff is engaged in the distribution and sale of bibles and religious
articles. The City Treasurer of Manila informed the plaintiff that it was
conducting the business of general merchandise without securing the necessary
license and paying the requisite fee in violation of the City ordinance. Plaintiff
protested against this requirement as constituting a restraint upon the exercise
of religion. It claimed that it is not engaged in business which necessitates the
securing of a license as it never made any profit from the sale of its bibles.

ISSUE: Whether or not the ordinance as applied to petitioner is unconstutional
for being in restraint of petitioner's right to free exercise of religion.

HELD: YES. The power to tax the exercise of the privilege is the power to control
or suppress its enjoyment. Those who can tax the exercise of religious practice
can make its exercise so costly as to deprive it of the resources necessary for its
maintenance. It is true that the price asked for the religious articles was in some
instances a little bit higher than the actual cost of the same, but this cannot
mean that plaintiff was engaged in the business or occupation of selling said
"merchandise" for profit. The mark up can only be treated as contributions by
the faithfuls to the religious cause. The Ordinance CANNOT be applied to
plaintiff society, for in so doing, it would impair its free exercise and enjoyment
of its religious profession and worship, as well as its right to disseminate
religious beliefs.

3. Exemtion from union shop

Victoriano v. Elizalde Rope Workers Union, 59 SCRA 54 (1974)

F: Benjamin Victoriano is an employee of the Elizalde Rope Factory. In 1962,
he resigned from the respondent labor union on the ground that the Iglesia ni
Kristo of which he is a member prohibits union membership. As the union
demanded his dismissal from employment pursuant to a closed shop agreement,
Victoriano brought this action for injunction. The CFI ruled in his favor exempting
from the closed-shop contracts members of religious sects which prohibit
affiliation of their members in any labor organization. The union appealed.

HELD: The statute does not violate the rights of association. It does not impair
the obligation of contracts for not only are existing laws read into contracts in
order to fix the obligation of the parties but the reservation of essential
attributes of sovereign power is also read into such contracts. Neither does the
law constitute an establishment of religion. It has been held that in order to
withstand objections based on this ground, the statute musr have a secular
purpose and that purpose must not directly advance or diminish the interest of
any religion. Congress acted merely to relieve persons of the burden imposed by
union security agreements.

4. Disqualification from local government office

Pamil v. Teleron 86 SCRA 413 (1978)

F: In 1971, Fr. Margarito Gonzaga was elected mayor of Albuquerque, Bohol.
A petition was filed against him on the basis of section 2175 of the Revised
Administrative Code providing that "in nocase shall there be elected or
appointed to a municipal office ecclesiastics, soldiers in active service, persons
receiving salaries from provincial funds, or contractors for public works." The CFI
dismissed the petition on the ground that the ineligibility has been impliedly
repealed by section 23 of the 1971 Election Code.

HELD: The voting of the SC was inconclusive. Seven justices held that section
2175 is no longer operative. Justice Fernando held that section 2175 imposed a
religious test on the exercise of the right to run for public office contrary to Art.
III of the 1935 Constitution. Justice Teehankee held that section 2175 had been
repealed by the Election Code. Five justices held that section 2175 is
constitutional.


VI. LIBERTY OF ABODE AND OF TRAVEL

Art. III, Sec. 6. The liberty of abode and of changing the same within the
limits prescribed by law shall not be impaired except upon lawful order of the
court. Neither shall the right to travel be impaired except in the interest of
national security, public safety, or public health, as may be provided by law.


Salonga v. Hermoso 97 SCRA 121 (1980)

Right to travel

This is not the first time petitioner Jovito Salonga came to the SC by way
of a mandamus proceeding to compel the issuance to him of a certificate of
eligibility to travel. In the first case, Salonga v. Madella, the case became moot
and academic. The present petition is likewise moot and academic. In the
motion to dismiss filed by the Solicitor General, it was stated that the certificate
of eligibility to travel had been granted petitioner.
Nonetheless, in view of the likelihood that this Court may be faced again
with the same situation, it is desirable that respondent Travel Processing Center
should exercise the utmost care to avoid the impression that certain citizens
desirous of exercising their constitutional right to travel could be subjected to
inconvenience or annoyance. The freedom to travel is one of the most
cherished. xxx


Marcos v. Manglapus, 177 SCRA 668 & 178 SCRA 760 (1989)

F: This petition for mandamus and prohibition asks the Court to order the
respondents to issue travel documents to Mr. Marcos and the immediate
members of his family and to enjoin the implementation of the President's
decision to bar their return to the Philippines. The case for petitioners is founded
on the assertion that the right of the Marcoses to return to the Philippines is
guaranteed under the provisions of the Constitution respecting one's liberty of
abode and right to travel. Respondents argue the primacy of the right of the
State to national security over individual rights.

RULING: The right involved in this case is not the right to travel from the
Philippines to other countries or within the Philippines. Essentially, the right
involved is the right to return to one's country, a totally distinct right under
international law, independent from although related to the right to travel.
The right to return to one's country is not among the rights specifically
guaranteed in the Bill of Rights, which treats only of the liberty of abode and the
right to travel, but it is the Court's well considered view that the right to return
may be considered as a generally accepted principle of international law, and
under our Constitution, is part of the law of the land. However, it is distinct and
separate from the right to travel and enjoys a different protection under the Intl.
Covenant of Civil and Political Rights, i.e. against being arbitrarily deprived
thereof.
The request or demand of the Marcoses to be allowed to return to the
Philippines cannot be considered in the light solely of the constitutional
provisions guaranteeing liberty of abode and the right to travel. It must be
treated as a matter that is appropriately addressed to those residual unstated
powers of the President which are implicit in and correlative to the paramount
duty residing in that office to safeguard and protect general welfare. In that
context, such request or demand should submit to the exercise of a broader
discretion on the part of the President to determine whether it must be granted
or not.

You might also like